Vous êtes sur la page 1sur 204

Electricité et Magnétisme

V. de Coulon, gymnase Auguste Piccard, août 2014


Table des matières

1 Charges électriques 1

2 Conducteurs et isolants 2

3 Loi de Coulomb 3

4 Champ électrique 4

5 Flux et loi de Gauss relative au champ électrique 8

5.1 Notion de flux . . . . . . . . . . . . . . . . . . . . . . . . . . . . . . . . . . . . . . 8

5.2 Loi de Gauss relative au champ électrique . . . . . . . . . . . . . . . . . . . . . 12

5.3 Loi de Gauss et loi de Coulomb . . . . . . . . . . . . . . . . . . . . . . . . . . . 13

5.4 Champ électrique proche de la surface d’un conducteur . . . . . . . . . . . . . 14

5.5 Compteur Geiger-Müller . . . . . . . . . . . . . . . . . . . . . . . . . . . . . . . 16

5.6 Effet cage de Faraday . . . . . . . . . . . . . . . . . . . . . . . . . . . . . . . . . 17

6 Tension électrique 18

6.1 Définition et propriétés . . . . . . . . . . . . . . . . . . . . . . . . . . . . . . . . 18

6.2 Lien entre tension et travail . . . . . . . . . . . . . . . . . . . . . . . . . . . . . 20

6.3 Oscilloscope . . . . . . . . . . . . . . . . . . . . . . . . . . . . . . . . . . . . . . . 22

7 Potentiel électrique 23

8 Condensateurs 26

8.1 Capacité d’un condensateur . . . . . . . . . . . . . . . . . . . . . . . . . . . . . 26

8.2 Condensateur plan . . . . . . . . . . . . . . . . . . . . . . . . . . . . . . . . . . . 28

8.3 Groupement de condensateurs . . . . . . . . . . . . . . . . . . . . . . . . . . . . 30

I
8.4 Energie d’un condensateur chargé . . . . . . . . . . . . . . . . . . . . . . . . . 32

8.5 Remarque concernant les canons à électrons . . . . . . . . . . . . . . . . . . . 33

9 Courant, loi d’Ohm 35

9.1 Courant . . . . . . . . . . . . . . . . . . . . . . . . . . . . . . . . . . . . . . . . . 35

9.2 Loi d’Ohm . . . . . . . . . . . . . . . . . . . . . . . . . . . . . . . . . . . . . . . . 37

10 Puissance, générateur, récepteur 40

10.1 Puissance électrique . . . . . . . . . . . . . . . . . . . . . . . . . . . . . . . . . . 40

10.2 Générateur et récepteur . . . . . . . . . . . . . . . . . . . . . . . . . . . . . . . . 41

11 Groupement de résistances, lois de Kirchhoff, circuits 44

11.1 résistances en parallèle . . . . . . . . . . . . . . . . . . . . . . . . . . . . . . . . 45

11.2 résistances en série . . . . . . . . . . . . . . . . . . . . . . . . . . . . . . . . . . 46

11.3 Lois de Kirchhoff . . . . . . . . . . . . . . . . . . . . . . . . . . . . . . . . . . . . 46

11.4 Exemples de circuit : décharge et charge d’un condensateur . . . . . . . . . . 48

12 Instruments de mesure 50

13 Champ magnétique et force de Lorentz 52

14 Charges en mouvement dans un champ magnétique 54

14.1 Effet Hall . . . . . . . . . . . . . . . . . . . . . . . . . . . . . . . . . . . . . . . . 55

14.2 Spectromètre de masse . . . . . . . . . . . . . . . . . . . . . . . . . . . . . . . . 55

14.3 Cyclotron . . . . . . . . . . . . . . . . . . . . . . . . . . . . . . . . . . . . . . . . . 56

14.4 Aurores boréales . . . . . . . . . . . . . . . . . . . . . . . . . . . . . . . . . . . . 58

15 Force de Laplace 59

II
15.1 Force entre deux courants parallèles . . . . . . . . . . . . . . . . . . . . . . . . 60

15.2 Galvanomètre à cadre mobile . . . . . . . . . . . . . . . . . . . . . . . . . . . . 61

15.3 Principe du moteur électrique à courant continu . . . . . . . . . . . . . . . . . 62

15.4 Courants de Foucault . . . . . . . . . . . . . . . . . . . . . . . . . . . . . . . . . 64

15.5 Moment magnétique . . . . . . . . . . . . . . . . . . . . . . . . . . . . . . . . . . 65

16 Champ magnétique, loi d’Ampère et loi de Biot-Savart 69

16.1 Loi d’Ampère . . . . . . . . . . . . . . . . . . . . . . . . . . . . . . . . . . . . . . 70

16.2 Champ magnétique d’un long fil rectiligne parcouru par un courant . . . . . 71

16.3 Champ magnétique à l’intérieur d’un solénoïde . . . . . . . . . . . . . . . . . . 73

16.4 Loi de Biot-Savart . . . . . . . . . . . . . . . . . . . . . . . . . . . . . . . . . . . 74

16.5 Bobine et barreau aimanté . . . . . . . . . . . . . . . . . . . . . . . . . . . . . . 78

17 Loi de Gauss relative au champ magnétique 79

18 Loi d’induction de Faraday 80

19 Générateur, moteur et alternateur 85

19.1 Exemple 1 : le générateur de courant continu . . . . . . . . . . . . . . . . . . . 85

19.2 Exemple 2 : le moteur à courant continu . . . . . . . . . . . . . . . . . . . . . . 87

19.3 Exemple 3 : l’alternateur . . . . . . . . . . . . . . . . . . . . . . . . . . . . . . . 89

19.4 Courant sinusoïdal produit par une dynamo . . . . . . . . . . . . . . . . . . . 92

20 Auto-induction 94

20.1 Enclenchement du courant dans une bobine . . . . . . . . . . . . . . . . . . . . 95

20.2 Energie d’une bobine traversée par un courant . . . . . . . . . . . . . . . . . . 96

20.3 Surtension dans une ampoule . . . . . . . . . . . . . . . . . . . . . . . . . . . . 97

III
21 Induction mutuelle : le transformateur 99

22 Oscillations propres 100

22.1 Masse suspendue à un ressort . . . . . . . . . . . . . . . . . . . . . . . . . . . . 100

22.2 Circuit LC . . . . . . . . . . . . . . . . . . . . . . . . . . . . . . . . . . . . . . . . 103

23 Oscillations forcées - circuits alternatifs 104

23.1 Circuit RL (série) en régime alternatif . . . . . . . . . . . . . . . . . . . . . . . 104

23.2 Circuit RC (série) en régime alternatif . . . . . . . . . . . . . . . . . . . . . . . 105

23.3 Circuit RLC (série) en régime alternatif . . . . . . . . . . . . . . . . . . . . . . 106

23.4 Puissance consommée par un appareil en régime alternatif . . . . . . . . . . 108

24 Circuits alternatifs : généralisation 109

24.1 Tensions aux bornes : UL , UR et UC . . . . . . . . . . . . . . . . . . . . . . . . . 109

24.2 Représentation de Fresnel . . . . . . . . . . . . . . . . . . . . . . . . . . . . . . 110

24.3 Notation par les nombres complexes . . . . . . . . . . . . . . . . . . . . . . . . 112

24.4 Généralisation aux autres types de circuits . . . . . . . . . . . . . . . . . . . . 113

25 Applications 115

25.1 Expérience de lévitation d’un anneau au-dessus d’une bobine parcourue par
un courant alternatif . . . . . . . . . . . . . . . . . . . . . . . . . . . . . . . . . 115

25.2 Transport de l’électricité et moteur (a)synchrone . . . . . . . . . . . . . . . . . 117

25.3 Filtre passe-haut, filtre passe-bas . . . . . . . . . . . . . . . . . . . . . . . . . . 124

25.4 Récepteur radio à modulation d’amplitude . . . . . . . . . . . . . . . . . . . . 124

26 Equations de Maxwell 125

27 Annexe I : Les nombres complexes 127

IV
28 Annexe II : Méthode des moindres carrés 131

28.1 Problématique . . . . . . . . . . . . . . . . . . . . . . . . . . . . . . . . . . . . . 131

28.2 Régression linéaire . . . . . . . . . . . . . . . . . . . . . . . . . . . . . . . . . . . 133

28.3 Modèles linéaires à M paramètres . . . . . . . . . . . . . . . . . . . . . . . . . 135

28.4 Applications . . . . . . . . . . . . . . . . . . . . . . . . . . . . . . . . . . . . . . . 136

29 Exercices 139

29.1 Série 1 : charge, loi de Coulomb, champ électrique . . . . . . . . . . . . . . . 139

29.2 Série 2 : flux, tension, potentiel . . . . . . . . . . . . . . . . . . . . . . . . . . . 140

29.3 Série 3 : condensateurs . . . . . . . . . . . . . . . . . . . . . . . . . . . . . . . . 142

29.4 Série 4 : courants, circuits . . . . . . . . . . . . . . . . . . . . . . . . . . . . . . 144

29.5 Série 5 : puissance, générateur, récepteur . . . . . . . . . . . . . . . . . . . . . 146

29.6 Série 6 : ampèremètres, voltmètres . . . . . . . . . . . . . . . . . . . . . . . . . 148

29.7 Série 7 : particules dans un champ magnétique . . . . . . . . . . . . . . . . . 148

29.8 Série 8 : force de Laplace . . . . . . . . . . . . . . . . . . . . . . . . . . . . . . . 150

29.9 Série 9 : champ magnétique produit par des courants . . . . . . . . . . . . . 154

29.10 Série 10 : flux, induction . . . . . . . . . . . . . . . . . . . . . . . . . . . . . . 155

29.11 Série 11 : auto-induction . . . . . . . . . . . . . . . . . . . . . . . . . . . . . . 159

29.12 Série 12 : circuits RLC . . . . . . . . . . . . . . . . . . . . . . . . . . . . . . . 160

30 Solutions des exercices 165

30.1 Solutions Série 1 . . . . . . . . . . . . . . . . . . . . . . . . . . . . . . . . . . . . 165

30.2 Solutions Série 2 . . . . . . . . . . . . . . . . . . . . . . . . . . . . . . . . . . . . 167

30.3 Solutions Série 3 . . . . . . . . . . . . . . . . . . . . . . . . . . . . . . . . . . . . 171

30.4 Solutions Série 4 . . . . . . . . . . . . . . . . . . . . . . . . . . . . . . . . . . . . 174

V
30.5 Solutions Série 5 . . . . . . . . . . . . . . . . . . . . . . . . . . . . . . . . . . . . 176

30.6 Solutions Série 6 . . . . . . . . . . . . . . . . . . . . . . . . . . . . . . . . . . . . 178

30.7 Solutions Série 7 . . . . . . . . . . . . . . . . . . . . . . . . . . . . . . . . . . . . 180

30.8 Solutions Série 8 . . . . . . . . . . . . . . . . . . . . . . . . . . . . . . . . . . . . 182

30.9 Solutions Série 9 . . . . . . . . . . . . . . . . . . . . . . . . . . . . . . . . . . . . 186

30.10 Solutions Série 10 . . . . . . . . . . . . . . . . . . . . . . . . . . . . . . . . . . . 188

30.11 Solutions Série 11 . . . . . . . . . . . . . . . . . . . . . . . . . . . . . . . . . . . 192

30.12 Solutions Série 12 . . . . . . . . . . . . . . . . . . . . . . . . . . . . . . . . . . . 193

31 Quelques constantes 197

VI
Remarque préliminaire

Une partie des figures et certains passages de ce cours sont adaptés des ouvrages cités en
référence.

1 Charges électriques

Dès l’Antiquité, les grecs savaient qu’un morceau d’ambre (résine) frotté exerce une at-
traction sur de petites brindilles de paille. Le mot électron (particule élémentaire, un des
constituants des atomes) vient du grec ≪ êlektron ≫ qui signifie ambre.

En frottant différents objets l’un contre l’autre, il arrive que ces objets soient dans un état
particulier : ils exercent les uns sur les autres des forces d’attraction ou de répulsion. Ils
sont dits électrisés. Un corps qui n’est pas électrisé est dit neutre.

Prenons un objet électrisé quelconque A . Tout autre objet électrisé est ou attiré ou re-
poussé par A . Ce critère permet de partager l’ensemble des objets électrisés en deux caté-
gories. L’expérience montre que deux corps de la même catégorie se repoussent toujours et
qu’un corps d’une catégorie attire n’importe quel autre corps de l’autre catégorie. L’objet A
fait donc partie de la catégorie des corps qu’il repousse. Nous en concluons qu’il y a deux
espèces d’électrisation et qu’il n’y en a que deux (fait expérimental). L’une est appelée po-
sitive (+) et l’autre négative (−) (on parle aussi de charge positive et de charge négative).
Par convention (due à Benjamin Franklin), on dit que le verre frotté (par un morceau de
tissu) est chargé positivement et que le plastique frotté est chargé négativement.

Les charges de même signe se repoussent. Les charges de signes contraires s’attirent.

On sait maintenant depuis moins d’un siècle que les atomes sont constitués de particules
des deux types d’électricité. Les protons (+) et les électrons (−). Les neutrons, eux, ne sont

1
pas électriquement chargés. Autrement dit, le noyau des atomes (formé de protons et de
neutrons) est positivement chargé ; la charge négative de l’atome (portée par les électrons)
est située autour du noyau (un peu comme un nuage étendu). D’autre part, l’essentiel
de la masse d’un atome est portée par le noyau (un proton a environ la même masse
qu’un neutron). En revanche, un électron est bien plus ≪ léger ≫ qu’un proton (environ 2′ 000
fois plus léger). Lorsqu’on électrise un corps par frottement, on arrache ou l’on donne des
électrons à ce dernier (les électrons sont très ≪ volatiles ≫ ). Les noyaux, eux, ne sont pas
déplacés.

Un atome électriquement neutre a autant de protons que d’électrons. Un corps chargé po-
sitivement contient davantage de protons que d’électrons (il est chargé par défaut d’élec-
trons). De même, un corps chargé négativement possède davantage d’électrons que de
protons (il est chargé par excès d’électrons).

Loi de conservation de la charge : dans un système isolé, la charge totale reste constante.

Même dans les cas où la matière ne se conserve pas, la charge, elle, se conserve. Par
exemple, diverses réactions nucléaires peuvent donner naissance à un positron (particule
de même masse que l’électron, mais de charge opposée). Lorsqu’un positron rencontre un
électron, les deux particules se combinent et se détruisent l’une l’autre. La matière dispa-
raît (elle s’est transformée en énergie) et il apparaît deux rayons gamma (γ) (particules de
lumière qui sont électriquement neutres). La charge du système est donc nulle dans l’état
initial comme dans l’état final. La loi de conservation de la charge est donc satisfaite.

L’unité de charge : dans le système international d’unités, l’unité de charge est le coulomb
noté [C]. Sa définition se déduit de celle de l’ampère (voir plus loin dans le cours). Il nous
suffit de dire ici que le coulomb est la charge portée par un ensemble de 6, 24 · 1018 protons.
Nous désignerons la charge du proton par e et la charge de l’électron par − e.

charge du proton = e = (6, 24 · 1018)−1 [C]=+1, 602 · 10−19[C ]


charge de l’électron = − e = −(6, 24 · 1018)−1 [C]=−1, 602 · 10−19 [C]

Remarque : lorsqu’on frotte un corps, on place sur ce dernier une charge qui, en valeur
absolue, est de l’ordre d’une dizaine de [n C] (nano-coulomb) par cm2 de surface frottée
(10−8 C/cm2 ). Cela représente un transfert d’environ 6 · 1010 (60 milliards) électrons par
cm2 .

2 Conducteurs et isolants

Dans certains types de matériaux, par exemple le métal, l’eau du robinet et le corps hu-
main, une partie de la charge négative se déplace librement au sein du matériau. Dans
ces types de matériaux, certains électrons se déplacent très facilement. On les appelle les
électrons libres ou les électrons de conduction (même s’ils ne sont pas totalement
libres). Ces matériaux sont appelés des conducteurs. Par exemple le cuivre est un ex-
cellent conducteur. Dans un morceau de cuivre, chaque atome contribue en moyenne pour

2
un électron libre parmi les 29 électrons qu’il possède.

A l’intérieur d’autres matériaux, notamment le verre, l’eau pure et le plastique, les élec-
trons se déplacent plus difficilement. On parle alors d’isolants. Un isolant ne possède
donc pas d’électrons libres.

Une tige de cuivre neutre est isolée de son environnement lorsqu’on la suspend à un fil
non-conducteur. En approchant une tige de plastique négativement chargée, les électrons
libres du cuivre vont être repoussés dans la partie droite de la tige de cuivre (région la plus
éloignée de la charge négative portée par la tige de plastique). De ce fait, la partie gauche
de la tige de cuivre est positivement chargée (par défaut d’électrons). On dit que la tige de
cuivre est polarisée. Une extrémité de la tige porte une charge +, l’autre une charge −.
Comme la charge − de la tige de plastique est plus proche du côté + du cuivre que du côté
−, on observe une force nette attractive et la tige de cuivre se met à tourner.

3 Loi de Coulomb

On suppose deux particules chargées (ou charges ponctuelles) séparées d’une distance r .
Notons q 1 et q 2 les charges (exprimées en coulombs) des deux particules respectivement.
~ qu’exerce une particule
L’intensité de la force électrostatique (attractive ou répulsive) F
sur l’autre est donnée par

| q1| | q2|
F=k loi de Coulomb (1785) (1)
r2
où k est une constante de proportionnalité appelée constante de Coulomb. Ce dernier a
établi cette loi à partir d’expériences réalisées en 1785.

3
Dans chaque cas, la force agissant sur une particule est de grandeur égale à la force agis-
sant sur l’autre, mais sa direction est opposée en vertu de la troisième loi de Newton
(action/réaction).

La valeur numérique de la constante k vaut

k ≃ 8, 99 · 109 [N m2 /C2 ]

Calculons à titre d’exemple la force de répulsion électrostatique qu’exerce un proton sur


un autre proton tous deux situés à l’intérieur d’un noyau d’atome et séparés d’une distance
de l’ordre de 4 · 10−15 m.
F = ke2 / r 2 = 8, 99 · 109 (1, 602 · 10−19 )2 /(4 · 10−15 )2 = 14 [N]. Comparons maintenant cette
force avec la force d’attraction gravitationnelle qu’exerce un proton sur l’autre :
F = Gm2p / r 2 = 6, 67 · 10−11 (1, 67 · 10−27)2 /(4 · 10−15)2 = 1, 2 · 10−35 [N]. Cette dernière force est
beaucoup plus faible (1036 fois plus faible). L’interaction gravitationnelle est négligeable
devant les forces électrostatiques entre particules élémentaires chargées.

4 Champ électrique

La notion de champ, déjà introduite au cours lors de l’étude de la gravitation, se révèle


être un concept très pertinent et utile pour décrire les forces qui s’exercent à distance
entre plusieurs corps.

4
Le champ électrique est un champ vectoriel, il est constitué par une distribution de
vecteurs, un pour chacun des points de la région entourant un objet chargé, par exemple
une tige. On définit le champ électrique en un point P comme suit. On pose d’abord une
charge positive q 0 (appelée charge d’essai) sur le point P . On mesure ensuite la force
~ qui s’exerce sur la charge d’essai. Pour terminer, on définit le champ
électrostatique F
~ produit par l’objet chargé (ici la tige) au point P par :
électrique E

~
~= F
E ~ : [N/C]
unité de E
q0

Par conséquent, la grandeur du champ électrique E ~ au point P est : E = F / q 0 et la direction


de E~ est celle de la force F
~ qui s’exerce sur la charge d’essai positive. On représente le
champ électrique au point P par un vecteur dont l’origine se trouve en P . Pour déterminer
le champ électrique à l’intérieur d’une zone, il faut répéter cette opération pour tous le
points de cette zone.

Bien qu’on emploie une charge d’essai positive pour définir le champ électrique d’un objet
chargé, ce champ existe malgré la charge d’essai. Le champ en P existait avant qu’on y
dispose la charge d’essai, tout comme il continue d’exister si on la retire. On suppose que
pour la définition du champ électrique, la présence de la charge d’essai ne modifie pas
la distribution de la charge sur l’objet chargé (ici la tige) et que, par conséquent, elle ne
change pas le champ électrique que l’on cherche à définir. Pour cette raison, on suppose
que la charge d’essai q 0 est faible.

D’autre part, l’expérience montre que la force qui s’exerce sur la charge d’essai q 0 est
proportionnelle à q 0 (généralisation de la Loi de Coulomb à une distribution de charges).
Par conséquent, la définition de E~ est indépendante de la valeur de la charge d’essai. Il
s’agit donc bien d’une propriété du corps chargé (la tige) dans le voisinage de ce dernier.

Pour déterminer le champ électrique E ~ en un point P produit par plusieurs charges, il


suffit de faire la somme vectorielle des champs que produit en P chacune des charges (en
faisant abstraction des autres). Notons par E ~ 1 le champ produit en P par une charge q 1
et par E~ 2 celui produit en P par une charge q 2 . Le système constitué par les deux charges

5
~ qui est simplement
q 1 et q 2 produit au point P un champ E

~ =E
E ~1 + E
~2

Si un point se déplace dans un champ électrique en suivant toujours la direction et le


sens indiqués par le vecteur-champ, il décrit une ligne qu’on appelle ligne de champ. On
peut aussi définir une ligne de champ comme une ligne qui est en tout point tangente au
vecteur-champ.

Les figures ci-dessous montrent quelques lignes de champ pour trois systèmes différents :
une sphère chargée négativement, un ensemble de deux charges ponctuelles positives et
négatives respectivement.

Les lignes de champ électrique s’éloignent d’une charge positive (d’où elles proviennent)
et elles se rapprochent d’une charge négative (où elles aboutissent).

L’exemple suivant montre une portion d’un plan infiniment grand où une charge posi-
tive est répartie de manière uniforme. Si l’on disposait une charge d’essai positive à un
quelconque endroit à proximité du plan (figure a), la force électrostatique résultante qui
s’exercerait sur la charge d’essai serait perpendiculaire au plan. En effet, les forces exer-
cées dans les autres directions se compenseraient en raison de la symétrie. Le vecteur
champ électrique à un endroit quelconque de l’espace se trouvant d’un côté ou de l’autre
du plan est également perpendiculaire à ce dernier et il est dirigé vers l’extérieur (figure
b et c). Comme la charge est répartie uniformément sur le plan, les vecteurs champ ont
tous la même grandeur. Un tel champ électrique, dont la grandeur et la direction sont les
mêmes en tout point, est appelé un champ électrique uniforme.

6
Exemple : Calculons le champ électrique à l’origine du système de coordonnées produit par
les trois charges q 1 , q 2 et q 3 disposées comme dans la figure ci-dessous. Admettons que
q 1 = +2Q , q 2 = −2Q et q 3 = −4Q où Q désigne une charge positive connue (par exemple
1µC). Les trois charges se trouvent toutes à une distance d connue de l’origine. Le vecteur-
champ E ~ vaut E
~ =E~1 + E ~2 + E~ 3 . Tout d’abord, déterminons le champ électrique d’une des
trois charges, disons q 1 . Plaçons une charge test q 0 à l’origine et calculons, par la loi de
Coulomb, l’intensité F1 de la force qu’exerce q 1 sur q 0 . La norme du vecteur-champ E ~1
produit par q 1 à l’origine est alors, par définition E 1 = F1 / q 0 . On a donc pour E 1 (norme
de E~ 1) :

F1 1 | q 1 || q 0 | | q1|
E1 = = k 2
=k 2
q0 q0 d d

Comme déjà mentionné, E 1 est bien indépendant de la charge test. En procédant de la


sorte pour les deux autres champs, il vient

2Q 2Q 4Q
E1 = k E2 = k et E3 = k
d2 d2 d2
~ 1 et E
Les champs E ~ 2 ont même direction et même sens. Par conséquent :

~ 2 k = E 1 + E 2 = k 4Q
~1 + E
kE
d2
~ 3 et E
On constate que les deux vecteurs E ~ 1+E
~ 2 ont même norme et qu’ils forment le même
~ est dirigé selon l’axe des x et sa
angle de 30◦ par rapport à l’axe des x. Ainsi, le champ E
norme vaut :
Q
E = 2 E 3 x = 2 E 3 cos 30◦ = 8 k cos 30◦
d2

Le champ électrique d’un conducteur chargé a des propriétés particulières lorsque les
charges sont au repos. On parle alors de champ électrostatique. Pour un tel conducteur,

7
le champ électrostatique est nul à l’intérieur de ce conducteur. En effet, si un champ exis-
tait, les électrons libres se mettraient en mouvement en contradiction avec l’état d’équi-
libre supposé.

Par ailleurs, les lignes de champ sont perpendiculaires au voisinage immédiat de la surface
du conducteur. Pour les mêmes raison, si une composante du champ parallèle à la surface
existait, les électrons libres se mettraient en mouvement. Le conducteur ne serait alors
pas en état d’équilibre.

La figure ci-dessous montre ce phénomène. D’autre part, les charges se concentrent à la


surface du métal dans les régions à faibles rayons de courbure (vers les pointes). Dans
ces régions, une charge ≪ ressent ≫ moins la présence des autres, le métal faisant office
≪ d’écran ≫ vis-à-vis du champ électrostatique créé par les autres charges.

On appelle cet effet, l’effet de pointe. Le paratonnerre par exemple utilise cet effet pour
favoriser la décharge électrique au travers de la pointe métallique plantée dans le sol que
constitue ce paratonnerre (plutôt que la sapin situé à côté de la maison et susceptible de
s’écraser sur elle).

5 Flux et loi de Gauss relative au champ électrique

5.1 Notion de flux

Pour introduire la notion de flux associé à un champ de vecteurs, considérons l’exemple


d’un liquide en mouvement (une rivière par exemple). En chaque point est définie la vi-
tesse ~
v du liquide en ce point. L’ensemble de tous les vecteurs vitesse constitue un champ
vectoriel. Les lignes de courant d’eau, dont les vecteurs vitesse sont tangents, forment les
lignes de champ.

Le flux du champ des vitesses dans notre exemple a une signification concrète : c’est le

8
débit du liquide (mesuré en m3 /s) à travers une surface. Voyons comment calculer un tel
débit.

a) Dans un écoulement où le vecteur ~ v est partout le même, plaçons une surface d’aire S ,
perpendiculairement aux lignes du champ. Il s’agit d’une surface ≪ mathématique ≫ que
nous plaçons par la pensée dans le fluide et non un objet matériel. On veut calculer le
débit d’eau (noté par D , en m3 /s ) qui traverse la surface d’aire S .

Pendant une durée ∆ t, l’eau qui traverse la surface a effectué une distance égale à v ∆ t.
Ainsi, le volume d’eau qui a traversé la surface pendant l’intervalle de temps ∆ t est égal
à S v ∆ t (volume d’un parallélépipède de base d’aire S et de hauteur v ∆ t). Le débit D est
donc
S v ∆t
D= =v S
∆t

b) Dans le cas où la surface est oblique par rapport aux lignes du champ du cas a), le calcul
est simple aussi (figure ci-dessous).

Toute l’eau qui passe à travers la surface S a passé à travers la surface S ′ qui, elle, est
perpendiculaire aux lignes de champ de vitesses. Le débit vaut donc (voir cas a) D = v S ′ .
Or, l’aire S ′ est plus petite que l’aire S . On voit sur la figure que S ′ = S cos α, où α est
l’angle entre les deux surfaces S et S ′ . La formule pour calculer le débit est donc, dans ce
cas

D = v S cos α

On peut donner une forme vectorielle à cette expression. Pour cela il faut représenter la
surface à l’aide d’un vecteur. Nous représenterons cette surface par un vecteur S~ dont la
direction est perpendiculaire à la surface (on parle aussi de vecteur normal à la surface)

9
et dont la norme kS ~ k est l’aire de la surface que l’on avait notée S . Le sens du vecteur S ~
est choisi arbitrairement. Le choix du vecteur S ~ dans la figure est tel que l’angle entre les
vecteurs ~ ~ vaut α. On reconnaît dans la formule du débit le produit scalaire des deux
v et S
vecteur ~ ~
v et S (~ ~ = v S cos α). Ainsi la formule pour calculer le débit s’écrit simplement
v·S

D =~ ~
v·S produit scalaire de ~ ~
v et S
~ , l’angle aurait été (180◦ − α) et la formule du débit
Si l’on avait choisi l’autre sens pour S
serait devenue

D = −~ ~
v·S

puisque cos(180◦ − α) = − cos α. Ainsi, au signe près, le débit D se calcule par ~ ~.


v·S

c) Dans un écoulement quelconque, introduisons par la pensée une surface quelconque.


Partageons-la en surfaces d’aires ∆S 1 , ∆S 2 ,..., ∆S n suffisamment petites pour qu’on puisse
les considérer comme planes et que sur chacune d’elles le champ soit pratiquement uni-
forme. Désignons par ~v i la vitesse sur l’élément de surface portant l’indice i et par ∆S ~i
~
le vecteur représentant cette surface. Les vecteurs ∆S i pointent tous du même côté de la
surface.

Le débit total D est la somme de tous les débits élémentaires à travers les petites surfaces.
~ i (voir la figure), le débit est
Avec le sens choisi pour les vecteurs ∆S
n
~i
X
D= ~
v i · ∆S
i=1

~ i aurait conduit à D = − P i ~
L’autre choix pour les vecteurs ∆S ~i
v i · ∆S

Définition : On appelle flux d’un champ de vecteurs ~


v (quelconque) à travers une surface
(quelconque), la grandeur notée Φ ( phi ) et définie par
≪ ≫

n
~i
X
Φ= ~
v i · ∆S flux
i=1

Remarque 1 : Le flux est donc défini au signe près. Il dépend du sens donné aux vecteurs
~ i . Le débit d’un liquide est donc, au signe près, le flux du champ de vitesse à travers
∆S
une surface donnée.

10
Remarque 2 : Plus exactement, le flux est défini par la limite de la somme ci-dessus,
~ i tendent vers zéro (leur nombre tend alors vers l’infini). Il s’agit alors
lorsque tous les ∆S
d’intégrales de surfaces (voir cours de mathématiques de 3ème année) que l’on note
Z
Φ= ~ ~
v · dS

Une intégrale est ≪ une somme infinie de termes infinitésimaux ≫ (ce n’est pas une défi-
nition mathématique, mais elle suffira pour nous !). les vecteurs d S ~ sont associés à une
portion infinitésimale de la surface en question. Ils sont perpendiculaires à cette surface.
La norme dS du vecteur d S ~ est l’aire (infinitésimale) de la portion de surface considérée.

La figure de gauche ci-dessous représente les lignes de champ de vitesses de notre rivière.
Imaginons une surface fermée, une sorte d’ellipsoïde par exemple, à l’intérieur de la rivière
(c’est l’ellipse dans la figure). Les petits vecteurs qui pointent vers l’extérieur de la surface
représentent les vecteurs éléments de surface ∆S ~ i dont on vient de parler. L’eau entre à
l’intérieur de la surface par la gauche et ressort par la droite. Le débit d’eau entrant est
égal au débit d’eau sortant. Il n’y a pas de perte : tout ce qui entre doit ressortir.

Dans la somme
n
~i
X
Φ= v i · ∆S
~
i=1

la somme des termes négatifs est le débit entrant (l’angle entre ~ ~ i est plus grand que
v i et ∆S
90◦ , le produit scalaire ~ ~ i est donc négatif). De même, la somme des termes positifs
v i · ∆S
est le débit sortant. On a donc nécessairement :

Φ=0 pour une surface fermée, figure de gauche

Dans la figure du milieu ci-dessus, il y a une source d’eau au sein de la rivière. Les lignes du
champ de vitesses jaillissent ou émergent du point représentatif de la source. Admettons
par exemple que le débit de cette source soit de 3 m3 /s. Il y a donc davantage de lignes de
champ de vitesses qui sortent d’une surface fermée quelconque (représentée par l’ellipse
ou le petit cercle entourant la source). Comme toute l’eau qui provient de la source doit
quitter l’intérieur de la surface (l’enceinte), le débit d’eau qui sort de l’enceinte est plus
grand, en valeur absolue, que le débit d’eau qui entre dans l’enceinte. La différence est
exactement égale aux 3 m3 /s. On a donc

Φ = 3m3 /s > 0 surface fermée contenant la source, figure du milieu

11
Similairement, la figure de droite représente un puits dans lequel l’eau s’engouffre et
quitte la rivière (disparaît en quelque sorte). Les lignes de champ arrivent contre le point
représentatif de ce puits. Admettons que l’eau quitte la rivière à un taux de 2 m3 /s. Cette
fois, le flux total Φ est négatif et vaut exactement :

Φ = −2m3 /s < 0 surface fermée contenant le puits, figure de droite

Dans le cas général où une surface fermée entoure plusieurs sources et puits, le flux (Φ) du
champ de vitesses sur cette surface est exactement égal à la somme des débit des sources
et des puits en comptant positivement les premiers et négativement les seconds.

Ce qu’il y a de remarquable, c’est que le flux est le même indépendamment de la surface


fermée, pour autant qu’elle entoure les mêmes sources et les mêmes puits.

5.2 Loi de Gauss relative au champ électrique

~ produit par un ensemble quelconque de charges électriques a des


Le champ électrique E
propriétés semblables à celles que nous venons de discuter pour le champ de vecteurs ~
v
d’un liquide en mouvement.

Nous appelons surface de Gauss une surface fermée quelconque qui peut ou non conte-
nir un certain nombre de charges électriques. Nous orientons les vecteurs ∆S ~ i vers l’ex-
térieur de la surface. Avec ce choix d’orientation, le flux Φ du champ électrique est bien

12
défini.
n
~i
~ i · ∆S
X
Φ= E
i=1

L’expérience permet d’énoncer la loi suivante :

Loi de Gauss : le flux du champ électrique à travers une surface de Gauss (donc fermée)
est proportionnel à la somme algébrique des charges situées à l’intérieur de cette surface.
Nous noterons :

Q enf ~
Φ= Loi de Gauss relative à E
ǫ0

avec Q enf , la charge totale enfermée (contenue à l’intérieur de l’enceinte définie par la
surface de Gauss) et ǫ0 une constante fondamentale en électricité appelée permittivité du
vide ou constante d’influence. Elle vaut :
C
ǫ0 ≃ 8, 86 · 10−12
Vm
On ne peut pas démontrer la loi de Gauss. Mais, comme ses nombreuses conséquences
sont vérifiées par l’expérience, on peut l’admettre comme un axiome. C’est une des lois
fondamentales de l’électricité.

Du point de vue des propriétés du flux, rappelons l’analogie entre le champ de vitesses
d’un liquide en mouvement et le champ électrique produit par des charges électriques :

charges (+) ⇔ sources charges (−) ⇔ puits ~ ⇔~


E v

5.3 Loi de Gauss et loi de Coulomb

Comme application de la loi de Gauss, calculons le champ électrique produit par une
charge ponctuelle positive q. Vu la symétrie sphérique, le champ électrique est le même en
norme pour tous les points situés à une distance r de la charge. De plus ce champ est ra-
dial (les lignes de champ sont des droites passant par la charge). Choisissons alors comme
surface de Gauss, une sphère de rayon r centrée sur la charge et calculons le flux Φ.

13
~ i est colinéaire au vecteur ∆S
Le vecteur E ~ i . On a
~ i = E i ( r ) ∆S i = E ( r )
~ i ( r ) · ∆S ∆ S i = E ( r ) 4π r 2
X X X
Φ( r ) = E
i i i

La loi de Gauss dit que ce flux doit être proportionnel à la charge contenue dans la sphère,
c’est-à-dire q. Ainsi
q
E ( r ) 4π r 2 =
ǫ0
On en déduit pour la norme du champ à la distance r :
1 q
E (r) =
4πǫ0 r 2
On constate que le champ électrique décroît en 1/ r 2 .

Si l’on place une charge d’essai q 0 à la distance r de q, la force F que q exerce sur q 0 vaut
donc
1 q q0
F = q0 E (r) =
4πǫ0 r 2
On retrouve bien la loi de Coulomb vue plus haut, avec Le facteur numérique :
1
= k ≃ 8, 99 · 109 [N m2 /C2 ]
4πǫ0

5.4 Champ électrique proche de la surface d’un conducteur

A partir de la loi de Gauss, il est facile de déterminer le champ électrique qui se trouve
immédiatement à l’extérieur de la surface d’un conducteur. On considère une portion de
la surface du conducteur assez petite pour qu’on puisse la considérer comme plane. On
imagine ensuite une minuscule surface de Gauss cylindrique qui est enchâssée dans cette
portion (voir figure). L’une des extrémités du cylindre se trouve entièrement à l’intérieur
du conducteur, alors que l’autre se situe complètement à l’extérieur. L’axe du cylindre est
choisi perpendiculaire à la surface du conducteur, afin d’exploiter la symétrie du champ.

14
On a vu que le champ électrique, proche de la surface d’un conducteur, doit être perpen-
diculaire à cette surface. D’autre part, les charges que l’on met sur le conducteur (ici des
charges +, on a arraché quelques électrons), viennent se placer à la surface du conducteur.
On a également vu que le champ électrique est nul à l’intérieur des conducteurs (lorsque
les charges sont immobiles).

On peut ainsi calculer immédiatement le flux Φ du champ E ~ à travers cette surface de


Gauss. Le flux est nul sur la surface latérale du cylindre (E ~ ⊥ dS~ ) et sur le ≪ couvercle
≫ situé dans le conducteur ( E~ = 0). Il ne reste que le couvercle extérieur où le flux vaut E S
~ ), S désigne l’aire du couvercle du cylindre.
~ ∥ dS
(car E

Φ=E S

On appelle densité superficielle de charge la quantité de charge par unité de surface.


Elle se désigne par σ et se mesure en C/m2 . La charge enfermée à l’intérieur de la surface
de Gauss est donc Q enf = σ S . Ainsi, si l’on connaît la densité superficielle de charge placée
sur la portion de conducteur considérée, on peut calculer le champ par la loi de Gauss :

Q enf σ S σ
E S= = ⇒ E= proche d’un conducteur
ǫ0 ǫ0 ǫ0

Champ électrique au voisinage d’une longue tige isolante chargée

La figure montre une portion d’une très longue tige isolante porteuse d’une charge positive
uniformément distribuée le long de la tige. On appelle densité linéaire de charge la
quantité de charge par unité de longueur. Elle se désigne par λ et se mesure en C/m. On la
suppose connue. Par symétrie, le champ électrique est radial et perpendiculaire à la tige.

Pour appliquer la loi de Gauss avec profit, on choisit une surface de Gauss qui a la symétrie
cylindrique. On considère un cylindre de hauteur h et de rayon r . On veut déterminer
l’intensité du champ électrique à une distance r de la tige. Sur les deux couvercles du
cylindre le flux est nul (E~ ⊥ dS~ ). Sur la surface latérale du cylindre, E est constant et
~ ~
E ∥ d S . Comme l’aire latérale vaut 2πrh, le flux Φ à travers la surface de Gauss vaut

Φ = E 2πrh

15
En appliquant la loi de Gauss il vient, sachant que la charge enfermée est Q enf = λ h,
Q enf λ h λ
E 2πrh = = ⇒ E= tige uniformément chargée
ǫ0 ǫ0 2πǫ0 r
On constate que le champ électrique décroît en 1/ r pour ce système-là.

5.5 Compteur Geiger-Müller

Il s’agit d’un détecteur de particules ionisantes (qui provoquent l’ionisation des atomes).
Il est constitué d’un fil central mince porteur d’une charge positive qu’entoure un cylindre
métallique, circulaire et concentrique porteur d’une charge négative qui, en valeur abso-
lue, est identique à celle du fil. Par conséquent, un puissant champ électrique radial est
établi à l’intérieur du cylindre. Ce dernier contient un gaz inerte à basse pression. Lors-
qu’une particule ionisante entre dans le détecteur par la paroi du cylindre, elle ionise
quelques-uns des atomes de gaz. Cette réaction produit des électrons libres vers le fil posi-
tif. Toutefois, le champ électrique est d’une telle intensité que ces électrons possèdent une
énergie cinétique suffisante pour ioniser à leur tour d’autres atomes de gaz. L’avalanche
d’électrons qui s’ensuit se transmet au fil. Un signal est alors émis, qui sert à enregistrer
le passage de la particule ionisante originale.

Typiquement, le rayon du fil central est de 25µm, le rayon du cylindre de 1, 4 cm et la


longueur du tube de 16 cm. Le champ électrique sur la paroi intérieure du cylindre est
d’environ 2, 9 · 104 N/C. En exploitant le résultat de la section précédente, on peut détermi-
ner la charge positive que porte le fil.

Calculons la densité linéaire de charge positive sur le fil

λ = 2πǫ0 r E = 2π 8, 86 · 10−12 1, 4 · 10−2 2, 9 · 104 = 2, 26 · 10−8 C/m

Comme la longueur du fil est de h = 16 cm, la charge q qu’il porte vaut q = λ h = 2, 26 ·


10−8 16 · 10−2 = 3, 62 · 10−9 C = 3, 62 nC. Par conséquent, la charge négative que porte le
cylindre est de −3, 62 nC.

16
Connaissant λ, on peut calculer l’intensité du champ électrique au voisinage immédiat du
fil (à 25µm du centre) :

λ 2, 26 · 10−8
E= = = 1, 62 · 107 N/C
2πǫ0 r 2π 8, 86 · 10−12 25 · 10−6

C’est-à-dire, un champ électrique environ 1000 fois supérieur à celui situé sur la paroi
intérieure du cylindre.

5.6 Effet cage de Faraday

Plaçons des charges sur un conducteur possédant une cavité. Lorsque l’équilibre électro-
statique est atteint (quasi instantanément), le champ électrique est nul dans la matière du
conducteur. Est-il aussi nul à l’intérieur de la cavité ? Les charges placées sur le conduc-
teur sont-elles réparties sur ses deux surfaces ou seulement sur sa surface extérieure ?

Considérons une surface fermée entourant la cavité et située entièrement dans la matière
du conducteur (en pointillé sur le dessin). Sur toute cette surface, le champ électrique est
nul. Le flux qui la traverse est nul. D’après la loi de Gauss, la charge qui y est enfermée
est nulle. La surface limitant la cavité ne porte donc pas de charge. La surface intérieure
du conducteur ne porte donc pas de charges et le champ à l’intérieur de la cavité est nul.

Cette propriété trouve une application pratique dans la cage de Faraday. A l’intérieur
d’une cage métallique, on protège des personnes ou des instruments de toute action élec-
trique venant de l’extérieur, même si la cage est chargée d’une manière importante.

Pour cette raison, il est donc difficile de téléphoner avec un portable situé à l’intérieur
d’un bâtiment fait de béton armé. Une personne située à l’intérieur d’une voiture est bien
protégée de la foudre.

17
6 Tension électrique

6.1 Définition et propriétés

La définition de la tension est très proche de celle du travail (vue au cours de mécanique).
Considérons deux points A et B d’un certain champ électrique. Joignons-les par une courbe
Γ. Partageons la courbe Γ en petits tronçons, suffisamment petits pour être considérés
comme rectilignes et pour que sur chacun d’eux, le champ électrique soit pratiquement
uniforme. Orientons chaque tronçon dans le sens de parcours de A vers B. Nous obtenons
~ i le champ
ainsi des petits vecteurs déplacement ∆~r i (pour i = 1, 2, .., n). Désignons par E
électrique (moyen) sur le i ème tronçon.

Par définition, la tension électrique entre les points A et B par le chemin Γ est la gran-
deur, notée U ou (U AB ) suivante :
~ i · ∆~r i
X
U AB = E
i

~ i · ∆~r i ) le long du chemin Γ.


On somme tous les produits scalaires élémentaires (E

Unité de la tension : dans le système international d’unités (MKSA), l’unité de la tension


est appelée le volt noté [V] (ou V sans les [..]), du nom de Alessandro Volta (1745-1827),
physicien italien qui découvrit la pile électrique (1800).
N J
1V = 1 · 1m ou 1V = 1
C C
On déduit de là que : 1 NC
= 1mV
. Ainsi l’unité N/C qui sert à mesurer le champ électrique
est équivalente à l’unité V/m.

De façon plus rigoureuse, la tension est la limite de la somme lorsque la longueur de tous
les tronçons tend
R vers 0. On obtient alors une intégrale du champ le long du chemin que
l’on note U = E~ · d~r .

Remarque : la tension U dépend en règle général du chemin qui joint les points A et B. On
ne l’indique généralement pas pour ne pas alourdir l’écriture.

18
La tension jouit des propriétés suivantes (comme pour le travail en mécanique)

U AB = −UBA

Le long d’un chemin AC sur lequel se trouve le point B

U AC = U AB + UBC

Comme pour le travail en mécanique associé à un champ de force conservatif, on a, en


électricité, une propriété semblable :

Les tensions sont indépendantes des chemins dans tout champ électrostatique (créé par
des distributions quelconques de charges au repos).

Voyons ce que signifie cette dernière propriété dans le cas d’une pile de 1, 5 [V].

Les lignes de champ électrique produites à l’extérieur d’une pile sortent de la borne posi-
tive (appelée anode) et rentrent par la borne négative (cathode) selon l’illustration. Consi-
dérons un chemin qui part de l’anode (+) et qui aboutit à la cathode (−). Effectuons le
P ~
calcul i E i · ∆~
r i (il peut être difficile à faire dans ce cas). Le résultat que l’on trouve est :
1, 5 [V]. En répétant cette même opération mais sur un autre chemin joignant l’anode à la
cathode, le résultat reste le même (1, 5 [V]). Ce résultat caractérise la pile et est indépen-
dant du chemin qui joint ses bornes. On l’appelle la tension aux bornes de la pile. Une
pile (générateur ou source de tension) est symboliquement représentée par :

La grande barre représente la borne + (anode) et l’autre la cathode (on ne mentionne


souvent plus les signes + et −).

Dans le montage ci-dessous, une pile dont la tension aux bornes est U AB (par exemple
U AB = 1, 5 [V]) est reliée par deux fils conducteurs (métal) à deux corps métalliques repré-
sentés par les deux cercles. On verra plus tard comment ce que l’on fait pour mesurer la
tension aux bornes d’une pile ou d’une autre source de tension.

19
Il y a un mouvement de charges lorsqu’on relie les deux corps métalliques aux bornes de la
pile. Après un temps très bref, les charges sont au repos et on a un champ électrostatique
dont les lignes partent du corps + et arrivent vers le corps −. L’allure de ces lignes de
champ est bien différente que dans la situation où la pile est seule. Toutefois, la tension
entre les points C et D est exactement la même qu’aux bornes de la pile. En effet, dans les
conducteurs (fils + corps) le champ électrique est nul (car plus de mouvement de charges).
Ainsi

UCD = UC A + U AB + UBD = 0 + U AB + 0 = U AB

La tension entre deux points d’un même conducteur est toujours nulle en électrostatique
(UC A = UBD = 0). Ce n’est plus vrai lorsque les électrons libres se déplacent dans les
conducteurs (présence d’un courant électrique). On le verra plus loin.

6.2 Lien entre tension et travail

Dans une région où règne un champ électrique E ~ , considérons deux points A et B et un


chemin quelconque Γ qui les joint. Une particule de charge q (positive ou négative) subit
alors une force électrique. Evaluons le travail entre A et B, le long de Γ, de cette force
électrique. Par définition du travail, il vient
~ i · ∆~r i = q E
~ i · ∆~r i = q U AB ⇒ WAB = q U AB
X X
WAB = F
i i

avec F~i = q E
~ i la force électrique sur le ième tronçon. Le travail est donc proportionnel à
la tension électrique. Le coefficient de proportionnalité est la charge q de la particule. On
peut dire aussi que la tension est un travail par unité de charge.

Remarque : le lien entre le travail et la tension WAB = q U AB dépend en toute généralité


du chemin Γ considéré. Cependant, dans la majorité des cas que nous considérerons, le
champ électrique sera conservatif. C’est le cas, en particulier, pour le champ électrique
produit par une distribution de charges au repos. Dans ce cas WAB (et U AB ) ne dépend pas
du choix du chemin Γ joignant A et B. Ce chemin peut être la trajectoire de la particule,
mais pas nécessairement.

Considérons le cas d’un champ électrique produit par une distribution de charges au repos
(électrostatique). Sous l’effet de ce champ électrique, une particule de charge q et de masse
m suit une certaine trajectoire. Elle passe à un certain endroit A puis, un peut plus tard,
à un endroit B. Si l’on connaît la tension électrique entre les points A et B, on peut déter-
miner la vitesse de la particule en B si l’on connaît sa vitesse en A . On utilise le théorème

20
de l’énergie cinétique (voir cours de mécanique) :

1 1 2 q U AB
mv2B − mv2A = WAB = q U AB ⇒ v2B = v2A +
2 2 m
Remarque : Lorsqu’on applique le théorème de l’énergie cinétique, le chemin Γ est toujours
une portion de la trajectoire de la particule en question.

Comme exemple, traitons le cas de deux grandes plaques conductrices parallèles reliées à
notre pile de 1, 5 [V]. La tension entre un point de la plaque + (noté A ) et un point B de la
plaque − est aussi de U AB = 1, 5 [V] (voir avant) et ceci quelque soit l’écartement entre les
plaques. Plaçons un proton (de charge e = 1, 602 · 10−19 C et de masse m p = 1, 67 · 10−27 kg)
entre les plaques proche de l’anode (+). Ce proton est repoussé par l’anode et attiré par la
cathode. Il va donc être accéléré en direction de cette dernière. Evaluons la vitesse vB du
proton juste avant qu’il ne s’écrase sur la cathode.
s s
2 e U AB 2 · 1, 602 · 10−19 · 1, 5
vB = v2A + = 0+ ≃ 16964 m/s ≃ 17 km/s
mp 1, 67 · 10−27

C’est déjà une vitesse considérable... et rien qu’avec une pile de 1, 5 [V] ! Reprenons un
exemple similaire en plaçant cette fois un électron (de charge − e = −1, 602 · 10−19 C et
de masse m e = 9, 11 · 10−31 kg) proche de la cathode (B) (plaque −). Ce dernier va donc
accélérer en direction de l’anode ( A ). En appliquant le théorème de l’énergie cinétique à
cette situation, il vient (sachant que UBA = −U AB = −1, 5 [V]) :
s s
2 ( − e ) U BA 2 (−1, 602 · 10−19) (−1, 5)
v A = v2B + = 0+ ≃ 7, 26 · 105 m/s = 726 km/s
me 9, 11 · 10−31

L’accélération de l’électron est beaucoup plus grande que celle du proton, vu que sa masse
est environ 1800 fois plus faible que celle du proton.

Il peut paraître surprenant que la vitesse obtenue ne dépende pas de la distance entre les
plaques, mais uniquement de la tension entre ces dernières. La raison vient de ce que le
champ électrique est plus intense entre les plaques si elles sont proches que si elles sont
éloignées (mais la tension, elle, reste la même). La pile transfert davantage d’électrons (de
l’anode vers la cathode) lorsque les plaques sont proches et de moins en moins lorsqu’on
les éloigne (voir la figure).

U AB

− − ~
E
+ + + −
+ − + −
+ − + −
+ − + −

b
A B b


+ + − +
+ −
+ − + − + −
d

21
Pour un petit écartement des plaques, le champ électrique entre ces dernières est uniforme
(ou homogène), si l’on néglige les effets de bord. Désignons par d la distance entre les
plaques. On a :

~ =E d
~ · AB U AB
U AB = E ⇒ E=
d
Le champ électrique en norme est donc inversement proportionnel à la distance d et, donc,
l’accélération de la particule également.

6.3 Oscilloscope

L’oscilloscope, ou oscillographe cathodique, est un instrument qui permet de mesurer les


tensions, spécialement les tensions qui varient très rapidement. Le principe de fonction-
nement est le même que le tube cathodique qui se trouve dans les TV (pas à écran plat).

Un filament porté à incandescence par le passage d’un courant électrique constitue la ca-
thode de l’instrument. La température étant élevée, l’agitation thermique est assez consi-
dérable pour que des électrons libres soient projetés hors du filament. A quelque distance
de la cathode se trouve l’anode, ayant la forme d’un tube. L’anode et la cathode sont re-
liées à un générateur qui établit entr elles une tension U de l’ordre de 2000 V. Le géné-
rateur place des charges sur la cathode et sur l’anode, et que ces charges produisent un
champ électrique dont les lignes vont de l’anode à la cathode. Les électrons éjectés su-
bissent une force orientée vers l’anode. Ils la traversent, puisqu’elle a la forme d’un tube,
et sont ainsi lancés dans l’espace à grande vitesse. Ils constituent ce que l’on appelait
autrefois des ≪ rayons cathodiques ≫ . Le système est placé sous vide de sorte que les élec-
trons ne heurtent pas les molécules d’air et ne sont donc ainsi pas freinés. A l’extrémité
du tube, ils viennent frapper un écran fluorescent. Ils rendent lumineux pour un instant
le point de l’écran qu’ils ont frappé. On peut, et c’est là l’intérêt de l’oscilloscope, faire dé-
vier le faisceau électronique. On y parvient au moyen de plaques de déflexion. Ce sont des
plaques conductrices parallèles disposées de part et d’autre du faisceau. Si on les relie à
un générateur dont la tension est U ′, des charges y sont amenées et créent un champ élec-
trique uniforme. En passant dans ce champ, les électrons subissent une force constante
qui courbe leur trajectoire. Etudions la déflexion de la trajectoire des électrons en fonction
de la tension U ′ appliquée aux plaques.

La vitesse v0 des électrons à leur arrivée à l’anode est, en faisant l’hypothèses qu’ils
quittent la cathode à vitesse nulle : (cette vitesse n’est pas nulle, mais négligeable devant

22
v0 )
s
1 2 eU
mv02 − 0 = − e(−U ) ⇒ v0 =
2 m

où m et − e sont la masse et la charge d’un électron respectivement.

Admettons que l’électron conserve sa vitesse v0 après avoir quitté le canon et avant d’en-
trer entre les plaques de déflexion (voir la remarque sur les canons à électrons à la fin
de la section traitant des condensateurs). Calculons l’angle de déflexion ϕ d’un électron.
Soient d la distance entre les plaques, L leur longueur et U ′ la tension qui règnent entre
elles. L’intensité du champ uniforme est donc E = U ′ / d (comme à la section précédente).
L’intensité de la force que l’électron subit est F = eE . Son accélération est constamment
perpendiculaire aux plaques et a pour grandeur

F eE eU ′
a= = =
m m md
Cette accélération est constante. L’électron subit donc un mouvement balistique.

La durée du passage t de l’électron est t = L/v0 puisque le mouvement horizontal de l’élec-


tron est un MRU. En revanche, la composante verticale de la vitesse à la sortie des deux
plaques est v = at. On a donc (voir figure)

at aL eU ′ L U ′L
tan ϕ = = 2 = =
v0 v0 mdv02 2U d

Sur l’écran, la déflexion de la tache lumineuse est sensiblement proportionnelle à tan ϕ.


Elle est, par conséquent, proportionnelle à la tension U ′ appliquée aux plaques.

7 Potentiel électrique

De manière similaire à l’énergie potentielle que l’on définit en mécanique dans le cadre
de champs de force conservatifs, on va définir le potentiel électrique à partir de la tension
électrique. Le potentiel électrique ne peut être défini que dans un champ électrique où les
tensions sont indépendantes des chemins (champ conservatif). De tels champs ayant cette
propriété sont : les champs électrostatiques (qui sont créés par des charges au repos) et les
champs dans lesquels il n’y a que des courants constants (voir plus loin dans le cours).

23
Dans un champ électrique satisfaisant à la condition mentionnée, choisissons un point
de référence O. La tension entre un point quelconque P et le point O est univoquement
déterminée, puisqu’elle ne dépend pas du chemin. C’est donc une fonction du point P . On
l’appelle potentiel de P . Avec les notations utilisées pour la tension, le potentiel de P ,
qui se note VP , est défini par :

VP = UPO

Le point O est choisi arbitrairement, mais une fois choisi, il est fixe et on considère la
tension entre tous les points P de l’espace et le même point O.

Comme les tensions se mesurent en volts, l’unité de potentiel est aussi le volt.

Calculons la différence de potentiel entre deux points P et Q . On a :

VP − VQ = UPO − UQO = UPO + UOQ = UPQ

Ainsi, la différence de potentiel entre deux points est égale à la tension entre ces points.

VP − VQ = UPQ

Pour deux points P et Q situés sur une même ligne de champ électrique, comme dans la
figure, la tension UPQ est positive. D’après ce que l’on vient de voir, le potentiel de P est
supérieur à celui de Q . On en déduit :

Si l’on suit une ligne de champ dans son propre sens, on rencontre des potentiels décrois-
sants. Dans notre exemple : VP > VQ .

Tous les points P qui sont au même potentiel définissent une surface dans l’espace. Cette
surface s’appelle surface équipotentielle. Un potentiel différent caractérise une autre
surface équipotentielle.

La tension entre deux points d’une même surface équipotentielle est toujours nulle (puisque
la différence de potentiel est nulle). Considérons un point P quelconque et envisageons
différents petits déplacements. Ceux qui donnent lieu à des tensions nulles sont perpendi-
culaires aux lignes de champs. Nous en déduisons :

Les surfaces équipotentielles sont partout perpendiculaires aux lignes de champs.

Les figures ci-dessous illustrent cette propriété. Elles présentent les lignes de champ élec-
trique et les coupes transversales des surfaces équipotentielles pour une charge ponctuelle
et pour un dipôle électrique (système de deux charges de signes contraires).

24
Considérons maintenant un conducteur chargé quelconque. Le champ électrique dans un
conducteur étant nul (si les charges sont au repos), il s’en suit que la tension entre deux
de ses points quelconques est également nulle. Ainsi :

Tous les points d’un même conducteur sont au même potentiel. La surface du conducteur
caractérise donc une surface équipotentielle. Ainsi :

Les lignes de champ sont perpendiculaires à la surface des conducteurs, dans le voisinage
de ceux-ci.

On peut donc parler sans ambiguïté du potentiel d’un conducteur ou de la différence de


potentiel entre deux conducteurs. On en déduit également qu’une ligne de champ ne peut
pas avoir son origine et son extrémité sur le même conducteur.

La figure de gauche montre l’allure des lignes de champ électrique lorsqu’une boule conduc-
trice neutre est placée à côté d’une boule conductrice chargée. La boule neutre ≪ se charge
≫ par influence (une partie des électrons libres se déplace en direction de la boule chargée)

laissant un défaut d’électrons sur sa partie droite. Chaque point de la boule neutre est
au même potentiel. De même, chaque point de la boule chargée est au même potentiel,
mais ce dernier est plus élevé que celui de l’autre boule. Il existe donc une différence de
potentiel (tension) entre ces deux boules.

La figure de droite montre une situation analogue. Un conducteur neutre est suspendu
dans un champ électrique externe. Le champ électrique est nul à l’intérieur du conducteur.

25
Les lignes de champ partent ou arrivent perpendiculairement à la surface du conducteur
(qui est une surface équipotentielle).

Potentiel électrique créé par une charge ponctuelle

Considérons une charge ponctuelle positive Q > 0 située à l’origine des coordonnées et
calculons le potentiel dans le voisinage de cette charge. Choisissons, comme c’est géné-
ralement l’usage, le point de référence O situé à l’infini (là où le champ électrique est
nul). Désignons par P un point repéré par le vecteur position ~r . Vu la symétrie sphérique
du champ électrique dans cette situation, le potentiel V (~r ) à l’endroit P est le même en
chaque point qui se trouve à la distance r ≡ k~r k de la charge. Autrement dit, les équipoten-
tielles sont des sphères centrées sur la charge. Ainsi, le potentiel ne dépend que de r (la
norme du vecteur ~r ). Nous le désignerons simplement par V ( r ). Nous devons donc calculer
ZO Z∞ Z∞
~ 1 Q Q 1
V ( r ) = UPO = E ( r ) · d~r = E ( r ) dr = 2
dr =
P r 4πǫ0 r r 4πǫ0 r
Pour effectuer le calcul, nous avons considéré un chemin rectiligne qui suit une ligne de
champ de P à l’infini (vu que le champ est conservatif). Si la charge Q est négative, le
~ ( r ) · d~r devient −E ( r ) dr sous l’intégrale. D’où
terme E

Q 1
V (r) = pour une charge ponctuelle Q > 0 ou Q < 0
4πǫ0 r

Le potentiel d’une charge ponctuelle tend vers 0 comme 1/ r lorsque r tend vers l’infini. On
obtient ainsi un résultat semblable à celui obtenu, dans le cours de mécanique, pour le
calcul de l’énergie potentielle dans le champ gravitationnel d’un astre.

8 Condensateurs

8.1 Capacité d’un condensateur

Un condensateur est un des nombreux dispositifs servant à emmagasiner de l’énergie


électrique. On trouve par exemple un condensateur dans un appareil photo portatif avec
flash fonctionnant à piles. Le condensateur sert à accumuler assez lentement la charge
entre deux utilisations, renforçant ainsi un champ électrique à l’intérieur de l’appareil. Il
conserve ce champ et l’énergie associée jusqu’à ce que celle-ci soit libérée rapidement sous
forme d’éclair lumineux. Les condensateurs sont des composants essentiels des circuits de
syntonisation des transmetteurs et récepteurs radio. Des condensateurs microscopiques
forment également les blocs mémoire des ordinateurs.

Nous avons déjà rencontré un condensateur qu’on appelle ≪ condensateur plan ≫ formé de
deux plaques conductrices reliées à une source de tension (ici une pile). Nous avions re-
marqué qu’en éloignant une plaque de l’autre, les quantités de charges portées par chaque
plaque diminuaient. Notre intention est de déterminer le lien qui existe entre la charge
portée par une plaque et la tension entre les plaques.

26
De manière générale, on appelle condensateur un ensemble de deux conducteurs (pas né-
cessairement des plaques), appelés armatures, isolés l’un de l’autre et portant des charges
opposées. Lorsque nous parlons de la charge Q d’un condensateur, il ne s’agit pas de la
charge totale (qui est généralement nulle), mais de la charge portée par une des arma-
tures. Sauf mention expresse, Q est choisi positif. De même nous comptons positivement
la tension U entre les armatures (U est donc la tension calculée à partir d’un chemin dont
l’origine part de l’armature (+) et aboutit à l’armature (−), le résultat étant bien entendu
indépendant du chemin choisi).

Considérons un ensemble de charges produisant un certain champ électrique. Remplaçons,


par la pensée, chaque charge par une charge k fois plus grande. Toute surface fermée
imaginable renferme k fois plus de charges et, d’après la loi de Gauss, le flux qui la traverse
doit être k fois plus grand. Cela n’est possible que si, en chaque point de l’espace, le champ
est k fois plus grand. En résumé : l’intensité du champ électrique est proportionnelle aux
charges qui le produisent.

Si, en tout point de l’espace, le champ est k fois plus grand, toute tension est également
multipliée par k. En résumé, la tension est proportionnelle au champ électrique.

Appliquons ces conclusions à un condensateur portant une charge Q . En tout point, le


champ électrique est proportionnel à Q . Comme la tension entre les armatures est propor-
tionnelle au champ, on en déduit que cette tension est proportionnelle à Q . Ce qui s’écrit :

Q=C U

Le coefficient de proportionnalité C est appelé capacité du condensateur. On peut aussi


dire :

La capacité d’un condensateur est le quotient de sa charge par la tension entre ses arma-
tures.

Dans l’exemple de la figure de la section (6.2), on peut donc dire que la capacité du conden-
sateur de gauche (lorsque les plaques sont proches) est plus élevée que la capacité du
condensateur de droite (plaques éloignées), puisque la tension entre les armatures est
toujours la même (c’est la tension aux bornes de la pile) et que la charge du premier
condensateur est plus grande que le second.

La capacité C d’un condensateur dépend de la géométrie de ce dernier (distance entre les


armatures, grandeur des armatures,...).

Dans le système international d’unités, la capacité se mesure en [C/V]. Cette unité s’ap-
pelle le farad (abréviation F).

coulomb
1 farad = 1
volt
Les condensateurs utilisés dans la technique ont des capacités beaucoup plus petite que
1 farad. On utilise les sous multiples de cette unité, par exemple : 1 microfarad = 1 µF =
10−6 F.

27
8.2 Condensateur plan

Calculons la capacité C d’un condensateur plan formé de deux armatures planes paral-
lèles, identiques, séparées d’une distance d petite par rapport aux diamètres des plaques.
L’aire d’une plaque est S . Nous utilisons à cet effet deux notions : la loi de Gauss et la
définition de la tension.

La surface de Gauss choisie enferme la charge Q portée par la plaque positive. Sa forme
(≪ boîte à chaussures ≫ ) tient compte de la symétrie du champ électrique (voir la figure). Le
champ électrique est considéré comme homogène entre les plaques (on néglige les effets de
bord). Seule la face située entre les armatures contribue au flux du champ, qui vaut donc
Φ = E S . En effet, sur la partie de la surface de Gauss située à l’intérieur de l’armature,
le champ est nul. Sur les faces latérales il l’est également si l’on néglige les effets de bord.
On a donc
Q
ES =
ǫ0

La tension U (positive) entre les plaques s’exprime en termes de l’intensité du champ E et


de d . Appliquons la définition de la tension en choisissant un chemin qui va de l’armature
(+) à l’armature (−) (par exemple celui de la figure). Il vient

U=E d

Donc

Q ǫ0 ES S
C= = ⇒ C = ǫ0
U Ed d

Cette relation montre que la capacité est d’autant plus grande que l’aire des plaques est
grande et que la distance qui les sépare est petite.

Cette formule n’est pas valable seulement pour un condensateur plan, mais pour un conden-
sateur de forme quelconque (cylindrique, sphérique, etc.) pourvu que la distance entre ses
armatures soit constante et très petite.

Si l’on place un diélectrique, c’est-à-dire un isolant entre les plaques d’un condensateur
portant une charge Q , le champ électrique subit une diminution. Indiquons brièvement
pourquoi. Les électrons et les noyaux du diélectrique subissent des forces de sens opposé.

28
Les atomes en sont légèrement déformés (voir figure). Le centre de gravité des charges
positives ne coïncide plus avec celui des charges négatives. Ce phénomène est appelé po-
larisation. Le résultat de ces déformations est qu’il apparaît à la surface du diélectrique
des charges positives d’un côté, négatives de l’autre.

La figure a) montre le diélectrique en l’absence de champ électrique extérieur. Les cercles


représentent les atomes. Dans la figure b), on applique un champ électrique par les arma-
tures d’un condensateur chargé. le champ étire un peu les atomes et sépare les centres des
charges positives et négatives. La séparation provoque des charges de surface (charges de
polarisation) sur les faces du diélectrique. Ces charges établissent un champ E ~ ′ qui s’op-
pose au champ appliqué E ~ 0 . Le champ résultant E
~ à l’intérieur du diélectrique (E
~ =E ~ 0 +E
~ ′)
a la même direction que E ~ 0 mais sa grandeur est moindre (figure c)).

Lorsqu’on calcule le champ à l’aide de la loi de Gauss, la charge enfermée Q ′ est la somme
algébrique de la charge Q de la plaque et de la charge de polarisation. Ces deux charges
étant de signe opposé, il est évident que Q ′ est plus petit que Q . Le champ est moins
intense que s’il n’y avait pas de diélectrique, car E = Q ′ /ǫ0 S . La tension aussi est plus
petite, car U = Ed . Par contre, la capacité est plus grande car elle vaut toujours C =
Q /U (et non pas Q ′ /U ). Ainsi, en introduisant un diélectrique, on peut montrer que la
capacité est multipliée par un facteur ǫ plus grand que 1, qui ne dépend que de la nature
du diélectrique. Ce facteur est appelé constante diélectrique. Par exemple, ǫ = 2 pour
la paraffine, 81 pour l’eau, entre 5 et 7 pour le verre, 1, 0006 pour l’air et 1 pour le vide.
Le facteur ǫ0 est d’ailleurs aussi appelé la constante diélectrique du vide. Ce dernier se
comporte un peu comme une sorte de diélectrique...

La capacité d’un condensateur contenant un diélectrique est donc

S
C = ǫ ǫ0
d
Si l’on introduit un diélectrique entre les plaques d’un condensateur portant une charge
fixe, la tension diminue. Mais, si le condensateur est relié à une source, sa tension et le
champ électrique doivent rester les mêmes. Dans ce cas, c’est la charge qui augmente.

29
8.3 Groupement de condensateurs

Nous appelons condensateur équivalent à un groupement donné, le condensateur unique


qui, chargé sous la même tension, emmagasine la même charge électrique.

condensateurs en parallèle

Lorsqu’on charge le système, la source doit fournir aux plaques positives une charge totale
Q 1 + Q 2 + Q 3 et aux plaque négatives une charge totale −Q 1 − Q 2 − Q 3 . La charge reçue par
le condensateur équivalent est donc

Q = Q1 + Q2 + Q3

La tension aux bornes de chaque condensateur est la même et égale à U . En effet, les
fils conducteurs qui relient les armatures (+) des condensateurs à la borne positive de la
source sont tous reliés entre eux et forment un unique conducteur dont les points sont au
même potentiel, disons V1 . De même, les armatures négatives sont au même potentiel V2
avec V2 < V1 et U = V1 − V2 . On a donc

Q = CU Q 1 = C 1U Q 2 = C 2U Q 3 = C 3U

et donc CU = C 1U + C 2U + C 3U . D’où

C = C1 + C2 + C3 condensateurs en parallèle

Lorsque des condensateurs sont en parallèle, on trouve la capacité équivalente en addi-


tionnant les différentes capacités.

condensateurs en série

30
Deux armatures reliées l’une à l’autre constituent un système isolé. Si une charge apparaît
sur l’une d’elles, la charge opposée doit apparaître sur l’autre. Il s’en suit que tous les
condensateurs doivent posséder la même charge. Désignons-la par Q . Lorsqu’on met le
système sous tension, la source fournit une charge +Q à la borne positive et une charge −Q
à la borne négative. Ainsi, la charge du condensateur équivalent est égale à la charge d’un
seul des condensateurs. D’autre part, la tension totale U est répartie entre les différents
condensateurs. On a

U = U AE = U AB + UBD + UDE

et donc Q /C = Q /C 1 + Q /C 2 + Q /C 3 . D’où

1 1 1 1
= + + condensateurs en série
C C1 C2 C3

Lorsque des condensateurs sont en série, on trouve l’inverse de la capacité équivalente en


additionnant les inverses des différentes capacités.

Comme exemple, déterminons la capacité équivalente de l’ensemble des capacités du schéma


électrique a) ci-dessous.

Les capacités sont C 1 = 12µF, C 2 = 5, 3µF et C 3 = 4, 5µF. La tension U qui s’applique à


l’ensemble est fixée.

On observe que les condensateurs (1) et (2) sont en parallèle. On peut donc les remplacer
par un condensateur unique de capacité équivalente (notée C 12 ) qui vaut

C 12 = C 1 + C 2 = 12 µF + 5, 2 µF = 17, 3 µF

On obtient ainsi le schéma b) dans lequel il reste deux condensateurs en série. Les bran-
chements aux points A et B sont les mêmes dans les figures a) et b). On peut remplacer
ces deux condensateurs par un seul de capacité équivalente (notée C 123 ) qui se calcule par

1 1 1 1 1 1
= + = + = 0, 28 µF−1 ⇒ C 123 = = 3, 57 µF
C 123 C 12 C 3 17, 3 µF 4, 5 µF 0, 28 µF−1

Supposons maintenant que la tension U qu’on applique à l’ensemble soit U = 12, 5 V.


Quelle est la charge du condensateur (1) ?

31
Cette tension de 12, 5 V s’applique au condensateur unique équivalent (123) dont on connaît
la capacité. On peut donc déterminer sa charge (notée q 123) par

q 123 = C 123 U = 3, 57 µF 12, 5 V = 44, 6 µC

Les condensateurs en série (12) et (3) de la figure b) portent la même charge que leur équi-
valent (123). Par conséquent, le condensateur (12) porte la charge q 12 = q 123 = 44, 6 µC.
On peut donc évaluer la tension aux bornes du condensateur (12) qui est U AB

q 12 44, 6 µC
U AB = = = 2, 58 V
C 12 17, 3 µF

Finalement cette tension U AB est également celle qui s’applique aux bornes de C 1 (et aussi
C 2 ) puisque les condensateurs (1) et (2) sont en parallèle. On peut donc trouver la charge
du condensateur (1) (notée q 1 ) par

q 1 = C 1 U AB = 12 µF 2, 58 V = 31 µC

8.4 Energie d’un condensateur chargé

Un condensateur chargé possède une énergie potentielle. Celle-ci s’exprime par le travail
qu’il faut fournir pour charger le condensateur, ou par le travail qu’il fournit en se déchar-
geant.

Imaginons un condensateur de capacité C qui possède une charge q (positive) à l’anode et


une charge − q à la cathode. Notons U ′ la tension entre les armatures. Calculons le travail
que la source doit fournir pour déplacer quelques électrons supplémentaires de charge
− d q (supposée très petite) de l’anode vers la cathode. On peut raisonner en admettant
que le transfert de la charge − d q se fait par un chemin situé entre les armatures. C’est
évidemment pas le cas en réalité, puisque le transfert se fait par la source, mais c’est
équivalent du point de vue du travail et il est plus facile de le déterminer de cette façon.
La force qu’exerce la source (ou l’agent extérieur) pour transférer − d q s’oppose à la force
électrique due au champ électrique entre les armatures. Or le travail de la force électrique
est négatif et vaut dW = − d q U ′ (la force électrique est dirigée dans le sens opposé au
déplacement). Ainsi le travail qu’exerce la source est l’opposé de dW . Il est donc positif.
Notons-le dWext . Ainsi

dWext = d q U ′

Ainsi, pour augmenter de d q la charge d’un condensateur (de capacité C ) qui possède déjà
une charge q et dont la tension est par conséquent q/C , l’agent extérieur (la source de
tension) doit exercer un travail de
q
dWext = d q
C
Partons d’un condensateur de capacité C déchargé. Amenons progressivement des charges
(c’est la source qui le fait) sur ses armatures jusqu’à ce qu’il possède une charge Q et une

32
tension U (la tension de la source). Pour faire passer q de 0 à Q par quantités infinitési-
males d q, il faut fournir un travail Wext qui est la somme (l’intégrale) de tous les travaux
élémentaires dWext . Chacun de ces dWext est représenté par l’aire du petit rectangle ha-
churé du graphique ci-dessous.

tension

Q /C
dq

q /C

q charge
0 Q

Ce graphique représente la tension U ′ entre les armatures, en fonction de la charge q


du condensateur à chaque étape du processus de charge. Il s’agit d’une droite U ′ = q/C .
Le travail total correspond donc à l’aire du triangle sous cette droite. Comme l’aire d’un
triangle est facile à calculer, il vient pour le travail total Wext nécessaire au chargement
du condensateur (en se souvenant que Q = CU )

Q Q /C Q 2 1
Wext = = = CU 2
2 2C 2
Ainsi, l’énergie potentielle E pot d’un condensateur de capacité C chargé sous une tension
U est

1
E pot = CU 2
2

Cette énergie potentielle est en fait contenue dans le champ électrique qui règne entre les
armatures du condensateur.

8.5 Remarque concernant les canons à électrons

Nous avons vu plus haut qu’un canon à électrons est constitué d’une cathode émettrice et
d’une anode. A cet égard, le canon constitue un condensateur. Selon la géométrie que l’on
donne à ce condensateur, l’énergie atteinte par les électrons après la sortie du canon peut
varier fortement pour une même tensions entre la cathode et l’anode. Illustrons ceci par
deux exemples. Le schéma suivant montre un canon de type condensateur plan

33
Le graphique ci-dessous montre l’allure du potentiel de ce condensateur le long de l’axe
Oz.

A la sortie de l’anode, l’électron subit encore un champ électrique qui le freine, l’électron
étant plus proche de l’armature + que de l’armature −. Cela se traduit par la présence
de la courbe décroissante illustrant le potentiel en-dehors du condensateur. Ce potentiel
tend assez vite vers 0 (cela dépend de la taille des plaques et de leur séparation) de sorte
que, un peu plus loin, l’énergie cinétique de l’électron tend à devenir la moitié de celle
qu’il avait juste à la sortie du canon. En effet, l’énergie cinétique est proportionnelle à la
différence de potentiel. Pour un point c quelconque situé sur l’axe Oz, l’énergie cinétique
de l’électron lorsqu’il arrive en c se calcule selon

E cin ( c) = (− e)Uac = − e(Va − Vc ) = e(Vc − Va )

Lorsque c est assez loin à l’extérieur du canon, la différence Vc − Va tend à devenir la moitié
de Vb − Va . Imaginons maintenant un canon sphérique illustré par le schéma suivant

34
Dans cette dernière situation et vu la symétrie sphérique, le potentiel est constant à l’exté-
rieur du canon (cela signifie que le champ électrique est nul en dehors du canon), contrai-
rement à l’exemple précédent. Ainsi, l’électron conserve l’énergie cinétique qu’il avait à
l’anode. Il n’est plus freiné comme avant. Par conséquent, pour une même tension appli-
quée entre la cathode et l’anode, l’énergie cinétique de l’électron, une fois sorti du canon,
peut varier d’un facteur 2 dépendant de la géométrie de ce dernier.

9 Courant, loi d’Ohm

9.1 Courant

Lorsqu’on joint par un fil conducteur (cuivre par exemple) les bornes d’une pile, les élec-
trons libres du fil, qu’on appelle aussi les électrons de conduction, vont se déplacer en
direction de la borne positive. Ils vont entrer dans la pile par la borne +, d’autres élec-
trons vont sortir de la borne − et se déplacer dans le fil. Par une section quelconque du fil,
on observe un débit d’électrons. Ce débit est d’ailleurs le même par une autre section du
fil, un peu comme le suggère l’image d’un courant d’eau dans un tuyau (le débit reste le
identique même si la section change).

Il est important de noter que seuls les électrons libres subissent cette dérive vers la borne
+ de la pile. Les noyaux ainsi que les autres électrons, eux, ne subissent pas de mouvement
≪ migratoire ≫ .

On appelle courant électrique le débit de charge qui passe par unité de temps par une
section du fil. Plus précisément, si, pendant un intervalle de temps ∆ t, une section d’un
conducteur est traversé par une charge ∆ q, on définit l’intensité moyenne du courant dans
ce conducteur par le quotient ∆ q/∆ t. L’intensité du courant est définie, elle, par la limite
du rapport ∆ q/∆ t lorsque ∆ t tend vers zéro. On écrit alors
dq
I=
dt
Dans le cas d’un courant constant I , la charge q qui traverse une section du conducteur
pendant un temps t est proportionnelle à ce temps. On a alors
q
I= courant constant
t
Dans le système international d’unités, la charge se mesure en coulombs et le temps en
secondes, donc le courant s’exprime en C/s, unité qu’on nomme ampère (abréviation A).
coulomb
1 ampère = 1
seconde
D’un point de vue microscopique, le courant est un mouvement d’ensemble de particules
chargées : électrons libres dans un métal, ions dans une solution électrolytique. Les chocs
que subissent les charges mobiles avec les autres particules agissent comme des frotte-
ments. Une force est nécessaire pour compenser ce frottement et maintenir les charges

35
en mouvement. Pour qu’un courant circule dans un conducteur, il faut donc qu’un champ
électrique y règne. Si le conducteur a la forme d’une fil, les lignes de champ électrique
longent le fil d’une extrémité à l’autre (voir figure).

Sens du courant : on prend la convention d’orienter le courant I dans le sens des lignes de
champ électrique dans le conducteur (ou dans la solution électrolytique). Ce choix a pour
conséquence que dans un fil électrique, le mouvement des électrons libres est opposé au
sens du courant électrique I .

Pour trois points A , B et C pris le long du courant, dans le même sens que ce dernier, il
existe maintenant une tension entre ces points (par opposition à ce que l’on a en électro-
statique), vu la présence d’un champ électrique à l’intérieur du conducteur. Dans notre
cas, on a

U AB > 0, U AC > U AB

D’autre part, si le courant I est constant, on peut encore définir un potentiel électrique (le
champ électrique est encore conservatif) et l’on a

VA > VB > VC

Lois de conservation du courant

Il résulte de la loi de la conservation de la charge électrique (voir début du cours), les deux
lois de conservation du courant suivantes

- Le long d’un fil conducteur, le courant est partout identique, même si la section du conduc-
teur varie.

- La somme des courants qui arrivent à un noeud (point de jonction de plusieurs fils) d’un
circuit est égale à la somme des courants qui en partent.

Dans l’exemple de la figure :

I1 + I2 = I3 et I3 + I4 = I5 + I6

36
9.2 Loi d’Ohm

Prenons un conducteur quelconque et appliquons entre ses extrémités une tension U . Il


s’établit dans le conducteur un courant d’intensité I . Si U varie, I varie aussi. L’expérience
montre que, pour beaucoup de conducteurs, I est proportionnel à U .

Loi d’Ohm : La tension le long d’un conducteur et le courant qui y circule sont proportion-
nels.

Cette loi se traduit par la relation :

U=R I

R est un coefficient de proportionnalité positif caractéristique du conducteur considéré. La


grandeur R est appelée la résistance du conducteur. Pour une tension donnée, plus R est
grand, moins bien le courant passe.

Dans le système international d’unités, l’unité de résistance est l’ohm. On le représente


par la lettre Ω. De la loi d’Ohm, il vient

1 volt = 1 ohm · 1 ampère

Etudions la résistance de fils cylindriques qui diffèrent les uns des autres par leur lon-
gueur, leur section et la matière dont ils sont fait.

Lorsqu’on applique une tension U à un tel fil de longueur L, le champ E ~ qui s’y établit a
partout la même intensité. On obtient donc la relation (définition de la tension)

U=E L

En appliquant la même tension au même fil mais de longueur plus grande, l’intensité
du champ électrique est alors plus faible et le courant aussi. Cela signifie que plus L est
grande, plus R l’est aussi. Pour une même longueur de fil, en augmentant sa section S , on
ne modifie pas l’intensité du champ (elle ne dépend que de la longueur), mais on augmente
le flot d’électrons libres, c’est-à-dire le courant. La résistance, elle, diminue donc d’autant.
On observe, en effet, que le flot d’électrons libres est homogène sur toute la section du
fil lorsque le courant est constant. En revanche, pour un courant alternatif de très haute
fréquence (qui nous concerne pas ici), le courant a tendance à se répartir sur une fine
couronne proche de la surface du conducteur et moins vers son centre (effet de peau). Avec
ce qui vient d’être dit, la résistance est grande si sa longueur est grande et sa section

37
petite. Plus précisément, l’expérience montre que, pour les conducteurs qui obéissent à la
loi d’Ohm, on a

L
R =ρ
S

où ρ est un coefficient de proportionnalité qui ne peut dépendre ni de la longueur ni de la


section du fil et qui, par conséquent, caractérise la matière dont est formé le conducteur.
ρ est appelé la résistivité du matériau. La résistivité se mesure en [Ωm]. On verra ce
que cette relation signifie dans une analyse ≪ microscopique ≫ de la conductibilité dans les
métaux.

La loi d’Ohm n’est pas toujours valable et, quand elle l’est, c’est toujours dans un domaine
limité de valeurs pour le courant (et donc pour la tension). Si la loi d’Ohm est vérifiée,
cela signifie en particulier que la résistivité ρ doit être indépendante du courant et donc
de la température (plus il y de courant, plus le fil chauffe). Or, pour la plupart des mé-
taux, ρ dépend plus ou moins fortement de la température. Dans certain alliage comme
le constantan (60% Cu, 40% Ni), cette dépendance est très faible, d’où son utilisation en
électronique par exemple. La figure ci-dessous montre la dépendance de la résistivité en
fonction de la température pour le cuivre.

On constate que la résistivité croît plus ou moins linéairement en fonction de la tempéra-


ture. La ligne pointillée qui croise la courbe montre un point de référence : la température
(ambiante) T0 = 293 K et la résistivité correspondante ρ 0 = 1, 69 · 10−8 Ωm. On peut ainsi
améliorer la loi d’Ohm en formulant une approximation empirique de la dépendance en
température de la résistivité par la relation

ρ = ρ 0 (1 + α (T − T0 ))

où α est un coefficient appelé coefficient thermique de la résistivité. Cette relation dé-


crit bien la résistivité en fonction de la température de la plupart des métaux. Par exemple
pour le cuivre α = 6, 8 · 10−3 K−1 et pour le constantan α = 10−5 K−1 , soit près de 700 fois
plus faible.

Approche microscopique et simplifiée de la loi d’Ohm

A cause de l’agitation thermique, les électrons libres (et les autres constituants de la ma-
tière aussi) décrivent des trajectoires très compliquées et erratiques suite aux multiples

38
collisions de ces particules entre elles. En appliquant une tension aux extrémités d’un fil, il
se superpose, à ces mouvements désordonnés, un mouvement uniforme de l’ensemble des
électrons libres. Dans une approche simplifiée, nous allons nous intéresser uniquement à
ce dernier mouvement, en imaginant que chaque électrons libres subit deux forces. Une
force de propulsion F~ exercée par le champ électrique et une force de frottement T ~ (due à
l’ensemble des collisions subies par l’électron). En appliquant la deuxième loi de Newton
à l’électron (de masse m et de charge − e) :

~ +T
F ~=m~
a=0

Le mouvement de migration de chaque électron étant quasi uniforme, l’accélération peut


être négligée. Faisons l’hypothèse que l’intensité de la force de frottement T ~ est propor-
~
tionnelle à la vitesse de l’électron, mais le sens de T est opposé à ~
v comme c’est le cas pour
une bille qui chute dans un liquide. Posons

~ = − k~
T v

~ = −eE
où k est une constante de proportionnalité positive. De F ~ , il vient

e ~ ~
~
v=− E = −u E
k
où la constante positive u vaut u = e/ k.

Considérons maintenant un fil de section S dans lequel les électrons libres se déplacent à
la vitesse v. Désignons par n le nombre d’électrons libres contenus dans le métal par unité
de volume. Le débit de ce gaz électronique à travers la section S est v S (nombre de m3 de
gaz électronique qui passent par seconde). Donc n v S est le nombre d’électrons libres qui
traversent la section par seconde. Ainsi e n v S est le nombre de coulombs qui traversent
la section par seconde. C’est donc le courant I . D’où

I=e nv S

En remplaçant v dans cette expression par u E trouvé plus haut, il vient

I=e n u S E

En appliquant une tension U aux extrémités du fil de longueur L, on a donc

U=E L

D’où il vient avec la relation précédente :

L
U= I
enuS
qui montre bien la proportionnalité de U et I . Le facteur de proportionnalité étant ce qu’on
a appelé la résistance R :

1 L
R=
enu S

39
La résistivité ρ devient dans ce modèle
1 k
ρ= = 2
enu e n
qui ne contient que des constantes propres au matériaux (excepté e) dont est fait le conduc-
teur.

Ce modèle classique à force de frottement proportionnelle à la vitesse est donc équivalent


à la loi d’Ohm.

La vitesse des électrons v est très faible. Pour un courant de 10 A circulant dans un fil
de cuivre de 1 mm2 de section, cette vitesse est de 6 · 10−4 m/s, soit un demi-millimètre
par seconde. Ce qui a une grande vitesse en revanche (proche de celle de la lumière),
c’est l’établissement du champ électrique dans le conducteur. Le courant s’établit presque
instantanément dans tout le conducteur.

10 Puissance, générateur, récepteur

10.1 Puissance électrique

Une source de tension est reliée par des câbles de connexion (sans résistance) à un dispo-
sitif non précisé. Il peut s’agir d’une résistance, d’une pile rechargeable, d’un moteur ou
de quelque autre appareil électrique. La source maintient une tension U entre ses propres
bornes et, par conséquent, entre les bornes du dispositif non précisé : U = Uab > 0.

Un courant I est produit par la source et circule de la borne a vers la borne b. On peut
raisonner comme si une charge positive élémentaire d q passe de la borne a à la borne
b pendant l’intervalle de temps dt et tel que d q = I dt (en réalité, c’est un petit paquet
d’électrons libres de charge − d q qui passe de b vers a pendant dt). Notons que la charge
− d q qui entre en b et celle qui sort en a après l’intervalle de temps dt ne sont en général
pas constituées des mêmes électrons libres vu leur faible vitesse de migration. Quoi qu’il
en soit, la source doit exercer un travail dW égal à :
dW = U d q = U I dt

40
pour maintenir le courant I dans le circuit pendant l’intervalle de temps dt.

La puissance électrique P de la source est donc égale à

dW
P= ⇒ P =U I
dt
De façon générale, l’unité de puissance est le watt ([W]), qui sont des joules par secondes.
Il vient donc pour une puissance sous forme électrique :

1 watt = 1 volt · 1 ampère

P représente la puissance qui passe de la source au dispositif. S’il s’agit d’un moteur relié à
un objet à tracter, cette puissance P est transférée sous forme de puissance mécanique. Si
le dispositif en question est une batterie d’accumulateurs en chargement, P est transférée
sous forme de puissance chimique emmagasinée dans la batterie. Finalement, si le dis-
positif est une résistance électrique, P est transférée sous forme de puissance thermique
dans la résistance qui a pour effet de faire augmenter sa température.

Cette dernière situation est également appelée effet Joule. Lorsqu’un courant I traverse
une résistance R aux bornes de la quelle existe une tension U , la puissance électrique P
est entièrement dissipée sous forme de puissance thermique Pth (d’énergie thermique par
unité de temps). Puisque l’on a U = R I (loi d’Ohm), il vient

U2
Pth = R I 2 ou Pth = effet Joule
R

Remarque :
Il faut bien distinguer P = U I de Pth . La puissance électrique P est valable pour tous les
types de transfert de puissance, tandis que Pth n’est valable que dans les situations où
toute la puissance électrique P est entièrement transférée sous forme de puissance ther-
mique dans une résistance. En général, les dispositifs électriques contiennent des câblages
électriques qui possèdent une certaine résistance globale R . Pour ces systèmes, il y a tou-
jours une dissipation de puissance thermique. Une partie de la puissance électrique P est
donc transformée en chaleur.

10.2 Générateur et récepteur

générateur

Lorsqu’on une pile est branchée à un appareil, une partie de son énergie chimique est
transformée en chaleur. La pile chauffe un peu, en plus du dégagement de chaleur de l’ap-
pareil. On peut tenir compte de ce phénomène de manière satisfaisante en imaginant la
pile comme constituée d’une source idéale de tension fixe, appelée tension électromo-
trice Uem et d’une résistance interne r qui permet de rendre compte des pertes ther-
miques. Du point de vue électrique, une pile et par extension la plupart des sources de
tension (générateur), sont représentées par le schéma

41
La tension U aux bornes de la source réelle dépend ainsi du courant I qu’elle débite. En
effet

U AB = U AC + UCB ⇒ U = Uem − r I

vu que U AC = Uem et UCB = − rI (le courant entre dans la source par la borne −, donc
UBC = rI et donc UCB = −UBC = − rI ).

La tension Uem est donc la tension aux bornes de la source lorsqu’aucun courant n’est
débité. On appelle caractéristique d’une source le graphique de la tension U à ses
bornes en fonction du courant I qu’elle débite. Ce graphique a donc l’allure d’une portion
de droite de pente négative − r et d’ordonnée à l’origine Uem > 0.

En multipliant par I l’équation encadrée, on a

Uem I = U I + r I 2

Le membre de droite s’interprète facilement : U I est la puissance électrique disponible


dans le circuit et r I 2 est la dissipation thermique à l’intérieur de la source. Par conser-
vation de l’énergie, le terme Uem I est la puissance totale (sous forme chimique dans une
pile sèche par exemple).

Pour un générateur qui est une pile, le bilan énergétique peut se mettre sous la forme
d’une figure du type suivant

42
On définit alors le rendement η (se dit éta) de cette pile comme le quotient de l’éner-
gie électrique produite (énergie utile) par l’énergie chimique absorbée (énergie totale). Ou
aussi, ce qui revient au même, comme le quotient de la puissance électrique par la puis-
sance chimique. On exprime généralement le rendement η en pourcent.

E électrique Pélectrique
η= =
E chimique Pchimique

récepteur

On appelle récepteur tout appareil électrique qui transforme la puissance électrique re-
çue en puissance d’un autre type (mais aussi éventuellement en puissance électrique). Le
courant I entre par la borne positive du récepteur et sort par sa borne négative, contrai-
rement au générateur. Un récepteur peut être un moteur électrique, une résistance, une
cuve à électrolyse, etc.

Du point de vue électrique, un récepteur est constitué d’un récepteur idéal de tension
fixe appelée tension contre-électromotrice Ucem et d’une résistance interne r (qui tient
compte des pertes thermiques). Désignons par U la tension aux bornes du récepteur réel.
Son schéma électrique est le suivant

La tension U dépend du courant I qui entre dans le récepteur. En effet

U AB = U AC + UCB ⇒ U = Ucem + r I

vu que U AC = Ucem et UCB = rI (le courant entre dans le récepteur par la borne + ici).

La caractéristique du récepteur a l’allure suivante

En multipliant par I l’équation encadrée, il vient

U I = Ucem I + r I 2

43
Le terme de gauche est la puissance électrique disponible et, comme d’habitude, le deuxième
terme de droite rI 2 est la dissipation thermique. Par conservation de l’énergie, le terme
Ucem I est la partie de la puissance électrique qui est transformée par le récepteur en puis-
sance d’un autre type que thermique, par exemple mécanique dans un moteur électrique,
etc. En résumé, un récepteur qui fonctionne avec un courant I délivre une puissance utile
(autre que thermique) égale à Ucem I

Ucem I = puissance utile autre que thermique

Dans le cas où un récepteur transforme totalement en puissance thermique la puissance


électrique disponible (un radiateur électrique par exemple), la tension contre-électromotrice
est nulle (Ucem = 0).

Pour un récepteur qui est un moteur électrique, le bilan énergétique peut se mettre sous
la forme d’une figure du type suivant

On définit alors le rendement η de ce moteur comme le quotient du travail mécanique


produit (travail utile) par l’énergie électrique absorbée (énergie totale). Ou aussi, ce qui
revient au même, comme le quotient de la puissance mécanique par la puissance élec-
trique.

Wmécanique Pmécanique
η= =
E électrique Pélectrique

11 Groupement de résistances, lois de Kirchhoff, cir-


cuits

On appelle résistance une portion de conducteur (généralement un tronçon de fil cylin-


drique) qui possède une certaine résistance électrique R . Lorsqu’on réalise un circuit élec-
trique où plusieurs de ces résistances sont reliées entre elles et aux bornes d’une source de
tension (pile par exemple), on distingue entre les câbles de connexion (qui servent à relier
les résistances) et les résistances elles-mêmes. Prenons l’exemple du circuit ci-dessous,
composé de trois résistances (des ampoules par exemples) et de fils de connexion qui les
relient entre elles et à la source. Au contraire des résistances, les fils de connexion sont
supposés de résistance électrique nulle (c’est-à-dire négligeables devant celle des résis-
tances). Cela revient à prendre des fils de connexion suffisamment gros.

44
Cela a pour conséquence que le champ électrique est nul dans les fils de connexion, mais il
ne l’est pas dans les résistances. Ainsi, tout le conducteur qui relie la borne + de la pile aux
deux résistances est partout au même potentiel (disons VA ), le suivant est au potentiel VB
et le dernier, qui relie la troisième résistance à la borne − est au potentiel VC . Si U désigne
la tension aux bornes de la pile, on a

VA > VB > VC et U = VA − VC

La tension aux bornes d’une des deux résistances groupées en parallèles est VA − VB . Pour
l’autre, la tension est aussi VA − VB . La tension aux bornes de la troisième résistance est
VB − VC .

De ceci, on tire déjà une règle importante pour la suite :

- les tensions aux bornes de résistances placées en parallèle sont les mêmes.

11.1 résistances en parallèle

Un courant I se divise en deux courant I 1 et I 2 dans deux résistances R 1 et R 2 . Désignons


par U la tension entre les points de jonctions.

En vertu de ce que l’on vient de constater, U est également la tension aux bornes de R 1 et
aussi aux bornes de R 2 . On a donc :
U U 1 1
I = I1 + I2 = + = U( + )
R1 R2 R1 R2
Ce calcul montre que U et I sont proportionnels. L’ensemble des deux résistances en paral-
lèle satisfait à la loi d’Ohm. Cet ensemble est équivalent à une résistance unique R (qu’on

45
appelle parfois résistance équivalente) telle que :

1 1 1
= + résistances en parallèle
R R1 R2

Ce résultat s’étend à un nombre quelconque de résistances. Lorsque des résistances sont


en parallèle, on trouve l’inverse de la résistance équivalente en additionnant les inverses
des différentes résistances.

11.2 résistances en série

Un courant I traverse successivement deux résistances R 1 et R 2 placées en série.

En appliquant la loi d’addition des tensions et la loi d’Ohm, on obtient

U AC = U AB + UBC = R 1 I + R 2 I = (R 1 + R 2 ) I

La tension appliquée à l’ensemble des deux résistances est proportionnelle au courant qui
les traverse. Cet ensemble satisfait donc à la loi d’Ohm. Il équivalent à une résistance
unique R dont la valeur est

R = R1 + R2 résistances en série

Ce résultat s’étend à un nombre quelconque de résistances. Lorsque des résistances sont


placées en série, on trouve la résistance équivalente en additionnant les différentes résis-
tances.

11.3 Lois de Kirchhoff

Bien des circuits électriques ne sont pas constitués par des groupes de résistances placées
en série, en parallèle ou mixte. On ne peut donc pas appliquer la méthode des groupements
précédente pour remplacer par une résistance équivalente l’ensemble des résistances de
tels circuits.

Etant donné un circuit complexe comportant plusieurs sources de tension, dont les ten-
sions aux bornes sont connues et plusieurs résistances connues également. On souhaite
déterminer les courants (en sens et en intensité) qui traversent chaque résistance. Consi-
dérons l’exemple suivant.

46
Rappelons ici la loi de la conservation des courants, appelée aussi la première loi de Kirchhoff :

La somme des courants qui arrivent à un noeud (sommet d’un circuit) est égale
à la somme des courants qui en partent.

D’autre part, dans des systèmes ou les courants sont constants, le champ électrique est
conservatif, ce qui implique que la tension le long de tout chemin fermé est nulle. C’est
équivalent à dire que la tension entre deux points quelconques est indépendante du che-
min qui joint ces points. On peut alors énoncer la deuxième loi de Kirchhoff :

Le long de toute maille (ou boucle) d’un circuit, la somme des tensions est nulle.

Appliquons ces deux lois au circuit donné afin de déterminer les courants (en sens et en
intensité), en supposant connues les résistances, les tensions des sources avec leur résis-
tance interne. On applique également la loi d’Ohm aux bornes de chaque résistance (les
câbles de connexion sont supposés parfaits, c’est-à-dire de résistances négligeables).

On commence par choisir arbitrairement un sens à chaque courant et un sens de par-


cours de chaque maille (voir figure). Dans l’exemple, il y a trois courants indépendants : I 1 ,
I 2 et I 3 . Il faut trois équations pour les déterminer. La première est fournie par la première
loi de Kirchhoff appliquée à un des deux noeuds. Les deux autres équations proviennent
de la deuxième loi de Kirchhoff appliquée aux deux mailles mentionnées (on aurait pu
choisir le circuit complet pour maille au lieu d’une des deux mailles proposées). Il vient
alors le système d’équations :

I1 − I2 − I3 = 0
R 3 I 3 + R 4 I 1 − U1 + r 1 I 1 = 0
r 2 I 2 − U2 + R 5 I 2 − R 3 I 3 = 0

Il suffit alors de résoudre le système linéaire par rapport aux inconnues I 1 , I 2 et I 3 . Si,
dans la solution trouvée, un courant est négatif, cela signifie que le sens choisi pour le
courant en question n’était pas le bon (le courant va donc dans l’autre sens).

47
11.4 Exemples de circuit : décharge et charge d’un condensateur

Décharge d’un condensateur

Considérons un condensateur de capacité C portant une charge Q . On relie ces armatures


au moyen d’un conducteur de résistance R . La charge du condensateur va rapidement
s’annuler. On veut déterminer la charge q( t) que porte le condensateur (la plaque (+)) en
fonction du temps.

On va appliquer la deuxième loi de Kirchhoff (valable encore dans les situations où les
courants ne varient pas trop vite, ce qui est le cas ici). Lorsque le condensateur possède
une charge q( t), la tension à ses bornes est déterminée par la relation q( t) = C U ( t). Cette
même tension se retrouve aux bornes de la résistance R et y produit un courant I ( t), avec,
par la loi d’Ohm, U ( t) = R I ( t). Notons qu’ici
dq
I ( t) = − car d q est la charge perdue par le condensateur !
dt
Comme
dq
q ( t) = C U ( t) et U ( t) = R I ( t) et I ( t) = −
dt
Il vient
dq
q ( t) = − R C
dt
Cette dernière équation différentielle pour q( t) a pour solution, sachant qu’à t = 0, q(0) =
Q,

q ( t ) = Q e − t/T avec T = RC constante de temps

Il est facile de vérifier que cette expression est bien une solution, en dérivant q( t) et en
remplaçant l’expression trouvée dans l’équation différentielle.

La charge du condensateur décroît de façon exponentielle vers 0, partant de la charge Q


(à t = 0).

Après un laps de temps d’une constante de temps, ( t = T ), la charge qui reste sur le conden-
sateur est égale à e−1 ≃ 0, 368 ≃ 37% de la charge contenue au départ. Après un temps égal
à cinq ou dix fois la constante de temps, le condensateur peut être considéré comme dé-
chargé.

48
q

Q q ( t ) = Q e − t/T
T = RC

0 t

Charge d’un condensateur

Un condensateur de capacité C est placé en série avec une résistance R . L’ensemble est
relié aux bornes d’un générateur de tension constante U .

On veut connaître la charge q( t) que porte la plaque (+) du condensateur en fonction du


temps, partant de q(0) = 0 au départ. On applique à nouveau la deuxième loi de Kirchhoff
q ( t)
U AB + UBD + UD A = 0 ⇒ R I ( t) + −U = 0
C
et l’on obtient, sachant qu’ici I ( t) = d q/ dt et en divisant toute l’équation différentielle par
R
dq 1 U
+ q ( t) =
dt RC R
La solution générale de cette équation différentielle pour q( t) est de la forme (à vérifier)

q( t) = A e−t/T + CU

où T est la même constante de temps que pour la décharge du condensateur. La constante


A est déterminée par la condition initiale : q(0) = 0. Donc 0 = A + CU , d’où A = −CU . Ainsi
³ ´
q( t) = CU 1 − e−t/T avec T = RC constante de temps

Partant de 0, la charge croît pour atteindre de manière asymptotique la valeur CU qui


représente la charge maximale du condensateur. Après un temps égal à quelques fois la
constante de temps, on peut considérer l’équilibre atteint.

49
q

CU

q( t) = CU 1 − e−t/T
¡ ¢

T = RC

0 t

12 Instruments de mesure

En électrostatique, pour mesurer une tension entre deux corps chargés, on utilise un ap-
pareil appelé voltmètre électrostatique ou électromètre. Avant l’ère de l’électronique,
un tel voltmètre électrostatique était par exemple l’électromètre à feuilles d’or ou à tiges
mobiles (voir expérience de cours), ou aussi l’électromètre à condensateur variable (fait de
deux plateaux conducteurs mais isolés en forme de secteurs circulaires et centrés sur le
même axe. Ces deux plateaux se recouvrent plus ou moins selon la tension qui leur est
appliquée).

Dans un circuit électrique où des courants existent, la mesure de ceux-ci se fait à l’aide
d’un ampèremètre. A l’intérieur de l’instrument, il y a un conducteur qui relie les bornes.
A cause de la résistance de ce conducteur, une tension apparaît entre les bornes toutes
les fois qu’un courant traverse l’instrument. L’aiguille (ou l’affichage digital) peut donc
aussi bien afficher la tension entre les bornes, que le courant qui traverse l’appareil. Il
suffit d’avoir une graduation convenable. Si l’instrument est étalonné pour mesurer des
courants, c’est un ampèremètre. S’il est étalonné pour mesurer des tensions, c’est un volt-
mètre. Ne pas confondre ce dernier avec le voltmètre électrostatique qui, lui, n’est pas
traversé par un courant. Le principe de fonctionnement d’un ampèremètre (ou d’un volt-
mètre) est discuté plus loin (voir la section sur la force de Laplace).

Pour mesurer un courant dans une portion de circuit, il faut placer l’ampèremètre dans le
circuit (en série). Sa résistance interne doit donc être la plus petite possible pour perturber
le moins possible le courant à mesurer.

Pour mesurer la tension entre deux points d’un circuit, il faut placer le voltmètre ≪ par
dessus le circuit ≫ (en parallèle). Sa résistance interne doit donc être la plus grande pos-
sible pour ne pas trop modifier les caractéristiques du circuit.

50
Avec un voltmètre et un ampèremètre, il est impossible de mesurer simultanément et
d’une manière exacte la tension aux bornes d’une résistance et le courant qui la traverse.
Avec un voltmètre électrostatique (figure de droite) et un ampèremètre, la chose est pos-
sible.

A gauche : U exact, I trop grand. Au milieu : U trop grand, I exact. A droite : U exact, I
exact.

Remarque : Il n’est pas possible de mesurer la tension entre deux objets chargés statique-
ment à l’aide d’un voltmètre ordinaire (c’est-à-dire un voltmètre qui n’est pas un voltmètre
électrostatique).

On peut incorporer aux ampèremètres et aux voltmètres des résistances qui diminuent
leur sensibilité et, par conséquent, élargissent leur domaine de mesure. Ces résistances
sont appelées shunts.

Shunt pour un ampèremètre : Pour mesurer un courant I important (par rapport aux pos-
sibilités de l’ampèremètre) dans une portion de circuit, on place le shunt R en parallèle
avec l’instrument. On choisit la valeur de R de telle manière qu’une fraction seulement du
courant I passe dans l’instrument ( i est le courant qui passe dans l’ampèremètre).

Calculons le courant dans l’ampèremètre (de résistance interne r ) en fonction du courant


total.
Rr R
U=r i= I ⇒ i= I
R+r R+r
Nous constatons que le courant mesuré dans l’ampèremètre est une fraction bien déter-
minée du courant total. Si cette fraction vaut 0, 1 ou 0, 001 par exemple, on trouvera le
courant total en multipliant par 10 ou par 100 les indications de l’appareil.

Shunt pour un voltmètre : On place le shunt R en série avec l’instrument (de résistance
interne r ).

51
Calculons la tension u appliquée au voltmètre en fonction de la tension totale U

u U r
I= = ⇒ u= U
r R+r R+r
Ce calcul montre que la tension mesurée par le voltmètre est une fraction bien déterminée
de la tension totale. Si cette faction vaut 0, 001 par exemple, il suffira de multiplier par
1000 les indication du voltmètre pour trouver la tension appliquée au système.

13 Champ magnétique et force de Lorentz

Les propriétés magnétiques des substances trouvent leur origine dans leurs atomes, plus
particulièrement leurs électrons. Chaque électron (proton et neutron aussi d’ailleurs) pos-
sède une caractéristique intrinsèque (comme la masse et la charge) qui le fait ressembler
à une espèce de mini boussole. Dans certaines substances, appelées ferromagnétiques, une
partie des électrons sont alignés (leurs mini boussoles pointent dans la même direction),
ce qui produit des régions qui ressemblent à des mini aimants. Ces régions s’appellent des
domaines (domaines de Weiss). Si, par conditions normales, la plupart de ces domaines
pointent dans la même direction, la substance est alors un aimant permanent. Comme
substances ferromagnétiques on trouve par exemple le fer, le nickel, le cobalt ainsi que des
alliages de ces éléments.

La figure montre de la limaille de fer que l’on a saupoudrée autour d’un barreau aimanté.
Cette limaille de fer fait apparaître ce que l’on appelle des lignes de champ magnétique.

52
Pour donner un sens à ces lignes de champ, on doit faire une convention. L’usage s’est
établi de donner aux lignes le sens indiqué par la pointe nord de l’aiguille de boussole
(pointe qui indique le nord sous l’effet du champ magnétique terrestre).

Les lignes de champ d’un barreau aimanté traversent l’aimant et forment des boucles fer-
mées (même celles qui ne semblent pas fermées sur la figure de droite). Les effets magné-
tiques extérieurs d’un barreau aimanté se font davantage sentir près de ses extrémités,
où les lignes de champ sont plus rapprochées.

Les lignes de champ entrent à une extrémité de l’aimant et en ressortent à l’autre. On


appelle pôle nord (N) l’extrémité de l’aimant où les lignes sortent et pôle sud (S) celle où
les lignes entrent.

Remarque : Contrairement au champ électrostatique où les lignes de champ partent d’une


charge + et aboutissent à une charge −, les lignes de champ magnétique sont toujours
refermées sur elles-mêmes. On ne peut pas isoler un pôle N ou un pôle S. En cassant un
barreau aimanté en deux, on obtient deux aimants chacun avec son pôle nord et son pôle
sud. Il n’y a pas de charge magnétique (monopôle). On en a en tous cas jamais observé.

On constate expérimentalement qu’un champ magnétique (région où il y a des lignes de


champ magnétiques) exerce une force sur toute particule chargée en mouvement dans son
sein. Cette force est appelée force de Lorentz. Elle est toujours perpendiculaire à la
vitesse de la particule et aux lignes du champ.

53
Considérons une particule de charge q positive se déplaçant perpendiculairement aux
lignes d’un champ magnétique. L’expérience montre qu’en un point bien déterminé du
champ, l’intensité de la force de Lorentz est proportionnelle à la charge q et à la vitesse v
de la particule. On peut donc écrire :

F = Bqv

où B est un coefficient de proportionnalité qui dépend du point considéré. En mesurant


la force F et en connaissant q et v, on peut déterminer ce coefficient B en chaque point
de l’espace par le rapport : B = F / qv. Ce coefficient B s’appelle l’intensité du champ
magnétique.

Finalement, en chaque point de l’espace, on peut construire un vecteur ayant la direction


et le sens de la ligne de champ passant par ce point et ayant la grandeur B. On obtient ainsi
le vecteur champ magnétique B ~ (ou induction magnétique). L’ensemble des vecteurs B ~
définis en tous les points de l’espace constitue un champ vectoriel, le champ magnétique.

Une fois le champ magnétique B ~ ainsi défini, on peut se demander comment s’exprime
la force de Lorentz qui s’exerce sur une particule de charge q quelconque (positive ou
négative) et ayant une vitesse ~v quelconque (pas nécessairement perpendiculaire à B ~ ).
L’expérience montre en particulier que cette force est nulle si ~
v a la même direction que
~ (même sens ou sens opposé). De façon générale cette force s’écrit comme un produit
B
vectoriel :

~ =q~
F ~
v∧B Force de Lorentz

Résumons quelques propriétés de la force de Lorentz :


- elle est nulle si la vitesse de la particule est nulle ou parallèle au champ magnétique
~.
B
- elle a le même sens que ~ v∧B ~ si q est positif et le sens opposé à ~ ~ si q est négatif.
v∧B

~ est
L’unité de B N
. Cette unité est appelée tesla (abréviation [T]).
C m/s

On utilise parfois un sous-multiple de cette unité : le gauss, qui vaut 10−4 tesla.

Ordre de grandeur : à proximité d’un petit barreau aimanté : B ∼ 10−2 T ; à la surface


de la Terre : B ∼ 10−4 T. Il est difficile de créer en laboratoire des champs magnétiques
supérieurs à quelques teslas.

14 Charges en mouvement dans un champ magnétique

Discutons différentes applications de la force de Lorentz.

54
14.1 Effet Hall

On fait passer un très fort courant I dans une feuille métallique très mince. On place cette
feuille dans un champ magnétique aussi intense que possible. On observe alors une petite
tension entre les deux bords de la feuille. Cette tension s’appelle la tension Hall.

Cela s’explique comme suit. Les électrons libres libres constituant le courant se déplacent
dans un champ magnétique. Ils subissent une force de Lorentz (F ~ ) qui tend à les déplacer
vers un bord de la feuille. On a ainsi une accumulation de charges négatives à un bord,
de charges positives à l’autre bord. Un état stationnaire est atteint lorsque le champ élec-
trique créé par les charges déplacées exerce une force qui compense exactement la force de
Lorentz. La tension Hall qui apparaît entre les bords de la feuille est toujours très faible.

On utilise l’effet Hall pour mesurer des champ magnétique par exemple. Un tel instrument
s’appelle une sonde Hall. Connaissant le courant I dans la sonde, la grandeur de la tension
Hall permet alors de déterminer la composante du champ magnétique perpendiculaire à
la feuille métallique.

14.2 Spectromètre de masse

Au début du 20ème siècle, les savants ont mis au point différentes méthodes pour mesurer
la masse des atomes. L’une des plus précises est le spectromètre de masse (voir la figure).

55
Des ions positifs, produits dans la source S par réchauffement ou par une décharge élec-
trique, émergent de la fente S 1 et pénètrent dans une zone appelée sélecteur de vitesse.

Le sélecteur de vitesse est constitué d’un condensateur aux bornes duquel il y a une ten-
sion U . Un champ électrique E ~ (non présenté) homogène existe donc entre les plaques du
condensateur. Au moyen de bobines (non présentées), on crée un champ magnétique homo-
gène B~ 1 perpendiculaire à E~ . Les lignes de B
~ 1 sortent de la page. Un ion de charge q > 0 et
de vitesse ~v qui traverse le sélecteur subit donc deux forces opposées : une force électrique
d’intensité qE et de Lorentz d’intensité qvB1 . On s’arrange d’ajuster B1 de sorte que ces
deux forces se compensent et qu’ainsi la trajectoire du ion soit rectiligne. Il vient donc
E
qE = qvB1 ⇒ v=
B1
Ainsi, à la sortie du sélecteur de vitesse, les ions qui traversent la fente S 2 ont exactement
la vitesse v = E /B1 .

Dans la deuxième région, où règne seulement un champ magnétique B ~ 2 homogène (dont


les lignes de champ sortent aussi de la page), les ions décrivent une trajectoire circulaire.
En effet, les ions subissent une force de Lorentz égale à q~ v∧B ~ 2 , qui est donc perpen-
diculaire à ~v et toujours contenue dans le même plan (car la force de Lorentz est aussi
perpendiculaire à B ~ 2 ). Le mouvement des ions caractérise un MCU, la force de Lorentz
étant la force centripète. Si m désigne la masse d’un ion et r le rayon de sa trajectoire, on
a donc (voir cours de mécanique)

v2 qB2 r qB1 B2 r
m = qvB2 ⇒ m= =
r v E
Après leur trajectoire semi-circulaire, les ions viennent frapper une pellicule photogra-
phique. L’impact produit permet à l’expérimentateur de déterminer le rayon r du demi-
cercle en question. L’état de ionisation des ions étant connu (ils sont généralement une
fois ionisé, d’où q = e), on peut calculer la masse de ces ions grâce à la relation ci-dessus
(B1 , B2 et E sont connus eux aussi). Remarquons qu’il y a un lien simple entre le champ
électrique E~ et la tension U mesurée aux bornes du condensateur. Si l’écartement des
plaques est d , on a, sachant que E~ est homogène : U = Ed .

14.3 Cyclotron

Conçu et mis au point par E. O. Lawrence (vers 1930), le cyclotron est le premier accéléra-
teur de particules. Les deux objets creux en forme de D (appelés dés), ouverts sur leur côté
rectiligne, sont constitués de feuilles de cuivre. Ces dés sont reliés à un oscillateur (géné-
rateur de tension alternative) qui crée une tension (et donc un champ électrique) entre eux
et qui change alternativement de sens à une certaine fréquence f . D’autre part, l’ensemble
du système est immergé dans un champ magnétique B ~ homogène et perpendiculaire au
plan des dés.

Supposons tout d’abord que la tension entre les dés soit nulle. On a des particules (généra-
lement des protons de masse m et de charge q) qui se meuvent dans un champ magnétique

56
uniforme. Certaines d’entre elles se mettent à décrire des trajectoires circulaires, pour les
mêmes raisons que dans la description du spectromètre de masse. Selon le calcul que l’on
a fait, le rayon de leur trajectoire vaut
mv
r=
qB

Comme ces particules ont des vitesses différentes, les rayons des cercles qu’elles décrivent
sont différents. Remarquons que le champ électrique est nul à l’intérieur de chaque dé
(effet cage de Faraday), mais pas entre les dés. Par ailleurs, le champ magnétique peut
pénétrer à l’intérieur des dés (le cuivre est non magnétique).

Le temps que met une particule pour effectuer un demi-cercle est T /2 (une demi-période)

T πr πm 2π m
= = ⇒ T= (période)
2 v qB qB

La fréquence f est donc

1 qB
f= =
T 2π m
Cette fréquence a ceci de remarquable : elle ne dépend ni de la vitesse ni du rayon de la
trajectoire. Elle ne dépend que de la sorte de particule ( q, m) et de l’intensité du champ
B. On met cela à profit en appliquant aux dés une tension alternative dont la fréquence
est égale à celle de la rotation des particules. Il y a nécessairement un certain nombre de
particules qui passent dans le champ électrique (entre les dés) au moment où son intensité
est maximum et où son sens est tel qu’elles subissent une augmentation de vitesse. Après
avoir accompli un demi-tour, elles se retrouvent dans le champ électrique au moment où
celui-ci atteint son maximum, mais dans le sens opposé. Elles subissent denouveau un
accroissement de vitesse. Au fur et à mesure que leur vitesse croît, le rayon de leur trajec-
toire augmente. Elles décrivent une sorte de spirale et finissent par atteindre la périphérie
des dés où un dispositif approprié les dévie pour les envoyer vers un éventuel détecteur.

Désignons par R le rayon maximum de la trajectoire des particules. Leur vitesse et leur
énergie cinétique finales sont

qBR 1 2 q2 B2 R 2
vfin = E cin = mvfin =
m 2 2m

57
L’énergie finale est proportionnelle au carré du rayon R et au carré du champ B. Pour
obtenir des particules de grande énergie, on doit donc construire des électro-aimants très
puissants et utiliser des dés de grand rayon.

Lorsqu’on atteint des niveaux d’énergie supérieurs à 50 MeV, ce type d’accélérateur fait
défaut. En effet, la fréquence f de révolution des particules dépend de la vitesse de ces
dernières lorsqu’elle devient comparable à la vitesse de la lumière (effets relativistes).

14.4 Aurores boréales

Lorsque la vitesse d’une particule chargée comporte une composante parallèle au champ
magnétique (uniforme), la particule se déplace alors selon une trajectoire hélicoïdale, sui-
vant la direction du champ. En effet, la composante parallèle de la vitesse (v∥ ) n’est pas
modifiée par la force de Lorentz. Le mouvement est donc un MRU dans la direction du
champ. En revanche, la composante perpendiculaire de la vitesse (v⊥ ) subit un change-
ment de direction seulement (pas en grandeur, vu que la force de Lorentz est perpendi-
culaire à ~
v⊥ ). Ainsi, dans le plan perpendiculaire au champ, le mouvement est un MCU
(mouvement circulaire uniforme). La composition de ces deux mouvements est bien une
hélice (voir figure).

La figure ci-dessous illustre le mouvement en forme de spirale d’une particule chargée


dans un champ magnétique non-uniforme. les lignes de champ plus rapprochées à gauche
et à droite indiquent que le champ est plus intense à ces endroits. Lorsque le champ est
assez intense à une extrémité, la particule est ≪ réfléchie ≫ à partir de cette extrémité.
Lorsqu’une particule est réfléchie à partir des deux extrémités, on dit qu’elle est captive
dans une ≪ bouteille magnétique ≫ . Les forces magnétiques aux extrémités ont une com-
posante qui pointe en direction du centre de la figure.

58
C’est de cette façon que les électrons et les protons sont retenus par le champ magnétique
terrestre. Les particules captives forment les ceintures de Van Allen, qui créent une
boucle au-dessus de l’atmosphère terrestre entre les pôles N et S (magnétiques) de la
Terre. Ces particules rebondissent vers l’avant et l’arrière, d’une extrémité à l’autre de
la bouteille magnétique, en quelques secondes.

Lorsqu’un vent solaire puissant projette un surplus d’électrons et de protons vers les cein-
tures de Van Allen, un champ électrique se produit dans la région où les électrons sont nor-
malement réfléchis. Ce champ élimine la réflexion et conduit plutôt les électrons vers l’at-
mosphère terrestre, où ils entrent en collision avec les atomes et les molécules de l’air, ce
qui provoque des émissions lumineuses. Cette lumière forme les aurores qui ressemblent
à un rideau de lumière suspendu à environ 100 km d’altitude. Les aurores peuvent se
produire dans une région qu’on nomme l’ovale auroral.

15 Force de Laplace

Lorsqu’un courant se trouve dans un champ magnétique, chaque particule en mouvement


subit la force de Lorentz. Il s’ensuit que le conducteur subit une force. A cette dernière, on
donne le nom de force de Laplace.

59
Considérons un tronçon de fil conducteur de longueur L et de section S parcouru par un
courant I . Ce tronçon est placé dans un champ magnétique uniforme B ~ . Les électrons
libres se déplacent à une vitesse moyenne ~ v (dans le sens opposé au courant). Notons par
n le nombre d’électrons libres par unité de volume du matériau considéré. Le tronçon de fil
considéré a un volume SL, il contient nSL électrons libres. Chacun d’eux subit une force
de Laplace − e~v∧B~ . La force de Laplace vaut donc
~ = nSL(− e~
F ~)
v∧B
D’autre part, le courant I s’exprime par
I = enSv
En effet, Sv est le débit volumique, d’où nSv est le nombre d’électrons libres qui traversent
une section par unité de temps, donc enSv est bien le courant (charge qui traverse une
section par unité de temps). Il vient donc

F~ = − enSL~v∧B~ = I ( −~
v ~
L) ∧ B
v
Or, le vecteur −~ v/v est un vecteur unité (de norme 1) qui est dans le sens du courant. On
définit alors le vecteur ~L:
~ ~
v
L=− L
v
qui représente un vecteur dont le sens est le même que celui du courant I et dont la norme
est égale à la longueur L du tronçon considéré. Finalement, la force de Laplace s’écrit
~=I~
F ~ force de Laplace
L∧B

15.1 Force entre deux courants parallèles

Deux longs fils parallèles, séparés d’une distance d , et parcourus par des courants I a et I b
respectivement, exercent des forces l’un sur l’autre.

60
Considérons le cas où I a et I b sont dans le même sens (figure). Le courant I a crée un
champ magnétique dans l’espace. Nous verrons plus loin que ses lignes de champ sont des
cercles traversés perpendiculairement en leur centre par le fil rectiligne qui transporte le
courant. A l’endroit où se trouve le fil b, ce champ est représenté par le vecteur B ~ a sur la
figure. Comme le fil b est parcouru par le courant I b , chaque tronçon L de ce fil subit la
force de Laplace notée F~ ba sur la figure. Cette force est dirigée vers l’autre fil (règle du
tire-bouchon).

On peut déterminer l’intensité de la force F ~ a que l’on


~ ba , à l’aide de l’intensité du champ B
calculera plus loin à l’aide la loi d’Ampère. Le résultat est :
µ0 I a
Ba =
2π d
et donc
µ0 LI a I b
F ba = I b LB a =
2π d
Réciproquement, la force que subit un tronçon de longueur L du fil a est dirigée en direc-
~ ba ). Cette force a la même norme que F
tion du fil b (sens opposé donc à F ~ ba .

Ainsi, deux fils parallèles parcourus par des courants de même sens s’attirent. Par un
raisonnement similaire, on a : deux fils parallèles parcourus par des courants de sens
opposés se repoussent.

Définition de l’ampère : Si deux fils parallèles, distants de 1 m, sont parcourus par des
courants de 1 ampère, chaque mètre de conducteur subit une force valant
µ0 LI a I b 4π · 10−7 · 1 · 1 · 1
F= = = 2 · 10−7 N
2π d 2π · 1
Cette expérience et ce calcul servent de base à la définition de l’ampère. L’ampère est
l’intensité d’un courant qui, parcourant deux conducteurs parallèles distants de 1 m, pro-
duit sur chaque mètre de conducteur une force de 2 · 10−7 N.

15.2 Galvanomètre à cadre mobile

Un cadre rectangulaire est enroulé autour d’un noyau de fer cylindrique qui peut tourner
autour d’un axe fixe. Dans l’entrefer (l’espace entre l’aimant et le noyau), existe un champ

61
magnétique radial. Placé dans ce champ, le cadre est soumis à un couple de forces de
Laplace qui provoquent la rotation du cadre.

Un ressort spiral fixé d’une part au cadre et d’autre part à la carcasse de l’ampèremètre
(ou galvanomètre) se tord de sorte que le cadre se retrouve finalement en équilibre sous
l’action de deux couples : le couple des forces de Laplace (proportionnel à l’intensité du
courant) et le couple du ressort spiral (proportionnel à l’angle de torsion, angle égal à la
déviation de l’aiguille). Grâce à la géométrie radiale des lignes de champ dans l’entrefer,
l’angle de déviation est proportionnel au courant, ce qui permet une graduation régulière
du cadran.

15.3 Principe du moteur électrique à courant continu

Un cadre rectangulaire est enroulé autour d’un noyau de fer cylindrique qui peut tourner
autour d’un axe fixe. Le cadre est alimenté en courant par l’intermédiaire du commutateur.
Le courant entre et sort par deux balais en graphite fixes qui frottent contre deux demi-
cylindres métalliques solidaires du cadre lorsque le moteur tourne. Ces demi-cylindres
sont connectés aux extrémités du fil du cadre.

I
I
~L
F ~
B
~v⊥
~
v∥

Dans l’entrefer, c’est-à-dire dans l’espace entre les électro-aimants fixes (stator) et la par-
tie mobile (rotor), existe un champ magnétique radial. Placé dans ce champ, le cadre est
soumis à un couple de forces de Laplace qui provoquent sa rotation. A chaque demi-tour,
le sens du courant dans le cadre est inversé grâce au commutateur. Ainsi, le couple agit

62
toujours dans le même sens et la continuité du mouvement est assurée.

A la place des électro-aimants on peut, bien entendu, mettre un aimant permanent en


forme de fer-à-cheval comme pour l’ampèremètre à cadre mobile.

Un générateur de courant alimente ce moteur à courant continu (groupe stator-rotor).


Le courant I est supposé constant dans le bobinage. Il est engendré par le mouvement
à vitesse v∥ constante des électrons libres. Ce mouvement collectif engendre la force de
Laplace F~ L (F
~2 sur la figure de gauche) qui s’exerce sur les portions de fil verticaux (côté
droit du rotor sur la figure de droite) :

~ L = − N e e~
F ~
v∥ ∧ B

où N e désigne le nombre d’électrons libres dans ces portions de conducteur. On a une


~1 ).
situation similaire du côté gauche du rotor (F

Ce couple de forces de Laplace engendre la rotation du rotor. Désignons par ~ v⊥ la vitesse


du bobinage du côté droit (et −~
v⊥ du côté gauche) . Les électrons libres correspondants ont
pour vitesse ~
v =~
v∥ +~
v⊥ . La force totale qu’ils subissent est :

~ =F
F ~L + F
~′ avec ~ ′ = − N e e~
F ~
v⊥ ∧ B

La force de Laplace F~ L se transmet au bobinage et donc au rotor, tandis que la force F~′
s’oppose au courant. Pour maintenir le courant I , le générateur doit exercer un champ
électrique qui va dans le sens du courant et dont la valeur vaut : E ~ cem ≡ −~ ~ . La
v⊥ ∧ B
tension correspondante mesurée le long des portions de fil considérés et dans le sens du
courant représente la moitié de la tension contre-électromotrice. L’autre moitié provient
de ce qui se passe du côté gauche du rotor. Il vient

Ucem = 2 N (−~ ~ ) ·~l = 2 Nv⊥ Bl


v⊥ ∧ B

où N est le nombre de spires et l la hauteur du rotor sur lequel le fil est bobiné (le vecteur
~l est dans le sens du courant). On voit que la tension Ucem est proportionnelle à la vitesse
de rotation du rotor. Elle est donc nulle si le rotor est bloqué.

~ L ·~
La puissance mécanique Pmec = 2F v⊥ = 2FL v⊥ développée par le moteur vaut

Pmec = 2 N e ev∥ Bv⊥

D’autre part calculons le produit Ucem I , où le courant I s’écrit aussi comme I = ensv∥ avec
n la densité d’électrons libres du conducteur et s la section du fil. On a (avec N e = N nsl )

Ucem I = 2 Nv⊥ Bl ensv∥ = 2 N e ev⊥ Bv∥ = Pmec

En résumé on retrouve, dans un cas particulier d’un processus transformant une puissance
électrique en puissance mécanique (plus des pertes par effet Joules), ce que l’on savait d’un
récepteur (U est la tension au générateur et r la résistance du bobinage) :

U = Ucem + rI et Pmec = Ucem I

63
On utilise par exemple ce moteur pour soulever une charge au moyen d’un treuil en bran-
chant le moteur à un générateur de tension fixe U . Au départ, le couple moteur ( Mmot )
est supérieur au couple résistant ( Mres ) afin de permettre le démarrage de la charge. Le
couple moteur est engendré par les forces de Laplace et le couple résistant par la traction
du câble qui est relié à la charge.

Lorsque Mmot > Mres , le rotor accélère et donc v⊥ augmente. Par conséquent Ucem aug-
mente aussi (Ucem = 2 Nv⊥ Bl ). Mais comme U est fixe le courant I diminue puisque U =
Ucem + rI . Ainsi, v∥ diminue ( I = ensv∥ ) et les forces de Laplace également. Donc le couple
moteur Mmot diminue d’intensité. Ainsi, le couple moteur tend à égaler le couple résistant
et la charge s’élève à vitesse constante après cette période transitoire.

La force de traction dans le câble devient donc identique au poids de la charge qu’on sou-
lève. Le courant I dépend donc de la charge mais pas de la tension U au générateur (tant
qu’elle est suffisante pour que le processus démarre).

Si, pour une même charge à soulever, on augmente un peu la tension U au générateur,
la vitesse de montée va également augmenter puisqu’elle est reliée à v⊥ qui est propor-
tionnelle à Ucem = U − rI ( I étant fixée à la même valeur que précédemment en régime
stationnaire, c’est-à-dire lorsque Mmot = Mres ).

15.4 Courants de Foucault

Lorsqu’une plaque métallique (cuivre) se déplace (à vitesse ~


v) entre les pôles d’un aimant,
on constate que tous ses mouvements sont gênés par une force de freinage qui rappelle
l’action d’un milieu visqueux.

~
v
~
v e∥ ~
~
B B ~∥
F
b b

~
ve ~⊥ ~
~
v e⊥ F B

Ce phénomène est dû à l’apparition de courants (appelés courants de Foucault) à l’intérieur


de la masse métallique. Le mouvement de la plaque dans le champ magnétique induit
une force de Lorentz qui agit sur les électrons libres. Après être entrés dans la région
où règne le champ magnétique, les électrons libres ont maintenant une vitesse ~ v e qui
possède une composante ~ v e⊥ perpendiculaire au déplacement de la plaque en plus d’une
composante parallèle ~ v e∥ à ce déplacement. La force de Lorentz se décompose alors en
~
deux composantes : F∥ (opposée au déplacement de la plaque) et F ~⊥ (perpendiculaire au
~
déplacement de la plaque). Seule la composante F∥ travaille lors du déplacement de la
~∥ agit
plaque (travail négatif). En se transmettant à la plaque (via les atomes de celle-ci), F
comme une force de frottement.

64
Les courants de Foucault, d’allure tourbillonnaire (voir figure), prennent ainsi naissance et
ne peuvent subsister que si le champ magnétique n’existe que dans une partie de la plaque.
Si le champ était uniforme, la force de Lorentz exercerait ses effets partout de la même
manière et le courant ne pourrait avoir lieu. Si l’on pratique des fentes dans le métal, on
empêche partiellement à ces courants tourbillonnaires de s’établir et l’on diminue, ainsi,
l’effet de frottement. Les courants de Foucault dissipent de l’énergie thermique dans le
métal, par effet Joule.

15.5 Moment magnétique

Un fil en forme de boucle rectangulaire, parcouru par un courant et libre de pivoter par
rapport à un axe central, est placé dans un champ magnétique. Les deux forces représen-
tées sur la figure sont les forces de Laplace qui s’exercent sur deux des quatre côtés du
cadre et produisent un couple de forces qui engendre la rotation du cadre autour de l’axe.
Les deux autres forces sont nulles dans la situation de la figure.

Dans une position différente du cadre (après rotation d’un certain angle), les deux côtés
perpendiculaires à l’axe sont soumis, cette fois, à des forces (opposées) de Laplace qui sont
dirigées le long de l’axe du cadre, ils ne contribuent donc pas à sa rotation.

La figure a) ci-dessous illustre une boucle rectangulaire de côté a et b, parcourue par un


courant I et placée dans un champ magnétique uniforme B ~ . La boucle est placée dans le
champ de façon que ses grands côtés 1 et 3 soient perpendiculaires à la direction du champ
(entrant dans la page), mais que ses côtés 2 et 4 ne le soient pas. La figure n’illustre pas
les fils qui font entrer le courant dans la boucle.

65
Afin de déterminer l’orientation de la boucle dans le champ magnétique, on utilise un
vecteur normal (de norme 1) ~ n perpendiculaire au plan de la boucle. La figure b) illustre la
règle dite de ≪ la main droite ≫ qui permet de trouver le sens de ~n. Replier les doigts de la
main droite dans le sens du courant en n’importe quel point de la boucle. Le pouce pointe
alors dans le sens du vecteur normal ~ n. Il s’agit de la même convention d’orientation déjà
rencontrée dans la définition de la circulation en rapport avec la loi d’Ampère.

Sur chaque côté du cadre s’exercent des forces de Laplace. Sur les côtés 2 et 4, ces forces
s’opposent et leur droite d’action passe par le centre du cadre ne produisant donc aucun
~1 et F
moment de force. La situation est différente pour les côtés 1 et 3. Les forces F ~3 ont
une grandeur commune I aB, mais des directions opposées. Ces deux forces n’ont toutefois
pas la même droite d’action. Elles produisent donc un moment de force résultant. Le mo-
ment de force fait pivoter la boucle de manière à aligner son vecteur normal ~
n en direction
~
du champ magnétique B. Ces forces ont un bras de levier ( b sin θ )/2 par rapport à l’axe
central de la boucle. La norme du moment de force M ~ produit par les forces F
~1 et F
~3 est
alors
b b
M = I aB sin θ + I aB sin θ = I abB sin θ = I SB sin θ
2 2
~ simplement, on
où S = ab est l’aire de la surface délimitée par le cadre. Pour exprimer M
définit le moment dipolaire magnétique ou aussi moment magnétique, le vecteur ~ µ:

~
µ = I S~
n

~
µ est normal au plan du cadre et dans le sens de ~
n. Sous forme vectorielle, le moment de
~
force M s’exprime par

~ =~
M ~ ⇒ M = µB sin θ
µ∧B

Evaluons maintenant le travail des forces de Laplace (1 et 3) qui font tourner le cadre
(le vecteur ~
n) de l’angle α vers l’angle β (voir la figure ci-dessous). Désignons par Wαβ ce
travail. On peut sauter le calcul qui suit et ne retenir que le résultat encadré (voir plus
bas).

cadre : position 2 Le cadre tourne de (1) vers (2).


~∥ travaillent.
Seules les F
α−θ
θ α
θ ~
n1
~
n2 β
cadre : position 1
~
B
~∥
F
~
F

L’angle θ est tel que 0 ≤ θ ≤ α − β, sachant que dr = b d θ /2 et F = I aB, il vient


Zα−β Zβ
b
Z
Wαβ = 2 F∥ dr = 2 F sin(α − θ ) d θ = µB − sin( x) dx = µB cos β − µB cos α
0 2 α

66
En définissant l’énergie potentielle magnétique associée à un angle α quelconque par

E mag(α) = −µB cos α = −~ ~


µ·B

le travail Wαβ peut s’écrire

Wαβ = E mag(α) − E mag (β), E mag = −~ ~


µ·B

Les résultats encadrés restent encore valables lorsque le cadre est orienté d’une manière
quelconque dans le champ B ~ . Lorsqu’il s’agit d’une bobine comprenant N spires, au lieu
d’une seule boucle de courant, il suffit de remplacer le courant I par N I dans les formules
précédentes.

On appelle généralement dipôle magnétique un système porteur d’un moment magné-


tique ~
µ. Dans un champ magnétique, un dipôle magnétique a une énergie potentielle ma-
gnétique qui dépend de l’orientation du dipôle dans le champ.

Un dipôle magnétique a le moins d’énergie (= −µB cos0 = −µB) lorsque son moment mag-
nétique ~
µ est aligné avec le champ magnétique (voir la figure). Il a le plus d’énergie (=
−µB cos180◦ = +µB) lorsque ~ µ a une direction opposée au champ.

Force sur un moment magnétique

L’énergie potentielle d’un moment magnétique calculée ci-dessus est encore valable dans
un champ magnétique inhomogène quelconque. Considérons l’exemple d’un moment ma-
gnétique ~µ situé dans un champ magnétique inhomogène (voir figure). Pour simplifier la
discussion, plaçons le moment magnétique le long d’une ligne de champ (en (1) sur la
figure).

1 ~
µ 2 ~
µ
b b
~
B
d~x

67
Envisageons un déplacement d~x du dipôle dans ce champ de la position (1) à la position (2)
supposée très proche de (1). Comme le travail des forces magnétiques qui s’exercent sur le
dipôle est égale à la variation de l’énergie potentielle, on peut écrire

~ ) ⇒ −~
E mag (1) − E mag(2) = W12 (F ~ 1 − (−~
µ·B ~2) = F
µ·B ~ · d~x

On a donc

~ ~2 − B
µ · (B ~1) = F
~ · d~x ⇒ ~ ~=F
µ · dB ~ · d~x

Dans le cas présent, tous ces vecteurs ont la même orientation. Comme B2 est plus grand
que B1 (car les lignes de champ se resserrent), la force F ~ qui s’exerce sur le moment ~
µ est
dirigée sur la droite (en direction de (2)). Cette force se calcule ici par
dB
F =µ
dx
La force est non nulle parce que le champ est inhomogène. Dans un champ uniforme
( dB/ dx = 0) le dipôle ne subit aucun déplacement linéaire ; il peut cependant s’orienter
à cause du moment de force (calculé avant) qui agit sur lui.

Le barreau aimanté de droite dans la figure produit un champ magnétique dont les lignes
de champ rappellent celles de la figure précédente. Dans la mesure où le barreau aimanté
de gauche peut être assimilé, du point de vue de ses propriétés magnétiques, à un unique
moment magnétique ~ µ (voir le ferromagnétisme plus bas), il va donc subir une force en
direction de l’autre barreau. On dit que le pôle nord d’un aimant attire le pôle sud d’un
autre (et inversement). Par un même argument, on voit que deux pôles nord ou deux pôles
sud se repoussent.

~
F
S N S N

~
µ

Remarque sur le ferromagnétisme

Presque toutes les particules subatomiques (y compris les électrons, les protons et les neu-
trons) ont des moments dipolaires magnétiques. Ces derniers ne sont toutefois pas dus à
des mini boucles de courant, mais correspondent à des propriétés intrinsèques des parti-
cules en question. D’un point de vue phénoménologique et dans le cadre d’une première
approche, l’état magnétique de la matière peut être décrit par une distribution de tels
dipôles magnétiques (chaque atome porte un moment magnétique) qu’on appelle l’aiman-
tation.

Il existe plusieurs types de magnétisme. Les aimants correspondent aux substances dites
ferromagnétiques. Dans ce type de magnétisme, les atomes, qui sont porteurs de mo-
ments magnétiques (dont l’origine est le moment magnétique intrinsèque des électrons),

68
interagissent entre eux à grandes échelles et produisent des effets macroscopiques im-
portants (pensez aux aimants permanents). Au-dessous de la température de Curie (1040
K pour le fer, 630 K pour le nickel), les substances ferromagnétiques s’aimantent spon-
tanément : les moments magnétiques s’alignent entre eux par domaines microscopiques.
L’application d’un champ magnétique extérieur tend à modifier ces domaines et à aligner
les directions de leurs moments magnétiques jusqu’à atteindre la saturation de l’aimanta-
tion (tous les moments sont parallèles). L’effacement du champ extérieur laisse la plupart
des moments encore alignés, donnant lieu ainsi à une aimantation permanente.

16 Champ magnétique, loi d’Ampère et loi de Biot-Savart

Il existe une autre manière de produire un champ magnétique que celui produit par des ai-
mants. Des particules chargées en mouvement (par exemple le courant dans un fil) créent
aussi des champs magnétiques.

Après la découverte de la pile par Volta, il y eut plusieurs tentatives pour trouver un lien
entre l’électricité et le magnétisme. En 1820, Hans Oersted faisait un cours de physique
à des étudiants. Par hasard, une boussole était posée sur la table parmi d’autres objects
et un fil branché à une pile passait au-dessus d’elle presque parallèlement à l’aiguille.
Quand il ferma le circuit, l’aiguille tourna pour se mettre presque perpendiculairement
au fil transportant le courant. Oersted venait d’observer qu’un courant crée un champ
magnétique capable de modifier la direction de l’aiguille d’une boussole. Suite à cette dé-
couverte, un jeune professeur, André-Marie Ampère, mena toute une série d’expériences
originales. En particulier, il réalisa que les lignes de champ magnétique produites par un
fil rectiligne parcouru par un courant sont des cercles concentriques. Ces lignes ainsi que
leur orientation sont présentées dans la figure avec la règle dite de la ≪ main droite ≫ (le
pouce de la main droite pointe en direction du courant, les autres doigts s’enroulent alors
naturellement dans le sens des lignes de champ magnétique créé par le courant).

69
En observant ces lignes de champ dans un plan perpendiculaire au courant, on obtient la
figure

En intensité, le champ diminue avec la distance qui sépare la ligne de champ (un cercle)
du fil.

16.1 Loi d’Ampère

On vient de voir que les lignes de champ magnétiques qui entoure un fil rectiligne par-
couru par un courant sont des cercles concentriques. La loi d’Ampère va nous permettre
de déterminer l’intensité de ce champ à une certaine distance du fil. Plus généralement,
cette loi fournit une relation entre le champ magnétique créé par une distribution de cou-
rants et la valeurs de ces courants. La procédure est la suivante :

On commence par considérer une courbe fermée Γ sur laquelle on choisit un sens de par-
cours (on parle aussi d’orientation).

On décompose cette courbe en petits tronçons de sorte que chacun d’eux soit pratiquement
rectiligne et que le champ magnétique (créé par les courants) puisse être considéré comme
~ i le vecteur
uniforme. Désignons par ∆~r i le vecteur représentant le i ème tronçon et par B
champ magnétique sur ce tronçon.

On appelle circulation de B ~ le long de la courbe fermée Γ, l’expression

~ i · ∆~r i ~ le long de Γ
X
B circulation de B
i

Plus exactement, la circulation est la limite de cette expression, lorsque tous les ∆~r i
tendent vers zéro. La somme ci-dessus devient alors une intégrale :
Z
~ · d~r
B

C’était la même démarche pour la définition de la tension et aussi du travail en mécanique.


Mais ici, pour la circulation, le chemin est fermé.

70
I1 I2
I3
Γ chemin fermé
I4
∆~r i sens de parcours
~i
B

Pour formuler la loi d’Ampère, il reste à tenir compte uniquement des courants qui sont
enlacés par le contour Γ. Ici, il y en a trois : I 1 , I 2 et I 3 . Le courant I 4 , bien qu’il participe
à l’existence du champ magnétique, ne va pas jouer de rôle dans l’expression de la loi
d’Ampère. La somme des courants enlacés est (en tenant compte du signe + ou − des
courants selon la règle de la main droite appliquée au sens du chemin Γ, voir la figure)

I enlacé = I 1 − I 2 + I 3 cas de la figure

On peut finalement énoncer la loi d’Ampère qui est valable lorsque les courants ne va-
rient pas au cours du temps :

Loi d’Ampère : La circulation du champ magnétique le long d’une courbe fermée quel-
conque est proportionnelle au courant constant enlacé par cette courbe.

Sous forme mathématique :

~ i · ∆~r i = µ0 I enlacé
X
B
i

La constante de proportionnalité µ0 est appelée la constante d’induction. Sa valeur


numérique est

µ0 = 4π 10−7 dans les unités internationales

Les deux constantes fondamentales en électromagnétisme sont la constante d’influence ǫ0


(voir la loi de Gauss relative au champ électrique) et µ0 .

Voyons quelques applications de la loi d’Ampère :

16.2 Champ magnétique d’un long fil rectiligne parcouru par un


courant

La figure de gauche montre un long fil rectiligne (vu de face) parcouru par un courant I qui
~ sont des cercles concentriques. Pour des raisons
sort de la feuille. Les lignes de champ B
de symétrie, l’intensité B du champ est la même en chaque point d’une ligne de champ.
Choisissons comme parcours d’intégration une de ces lignes de champ située à l’extérieur

71
du fil. Orientons ce parcours dans le même sens que la ligne de champ. Ainsi le vecteur
déplacement d~r et le vecteur B ~ sont colinéaires et de même sens, l’angle θ entre les deux
~ · d~r = Bdr .
est donc nul. Ainsi B

~ est donc simple à effectuer. Il vient


Le calcul de la circulation de B
Z Z Z
~ · d~r = Bdr = B dr = B 2πr
B

En appliquant la loi d’Ampère, on a

µ0 I
B 2 π r = µ0 I ⇒ B= champ à l’extérieur du fil
2π r

On constate donc que l’intensité du champ à l’extérieur du fil décroît en 1/ r .

La figure de droite est une coupe transversale de ce fil de rayon R parcouru par un courant
I uniformément distribué sur sa section. On souhaite maintenant calculer l’intensité B du
champ magnétique, à l’intérieur du fil, à une distance r du centre du fil ( r < R ).

La circulation le long d’un cercle de rayon r est, comme avant :


Z Z Z
~
B · d~r = Bdr = B dr = B 2πr

Ce qui change par rapport à la situation précédente, c’est l’intensité du courant enlacé.
Seule une fraction du courant I total passe à travers le contour. Cette fraction est donnée
par le rapport entre πr 2 et la surface de la section du fil : πR 2 . Il vient pour I enlacé :

πr2 r2
I enlacé = I= I
πR 2 R2
En appliquant de nouveau la loi d’Ampère à cette situation, il vient

r2 µ0 I
B 2 π r = µ0 I ⇒ B= r champ à l’intérieur du fil
R2 2π R 2

On constate donc que l’intensité du champ à l’intérieur du fil croît linéairement avec r .
Lorsque r = R , on trouve le champ à la périphérie du fil, où il est maximal. Sa valeur est
Bmax = µ0 I /2πR .

72
16.3 Champ magnétique à l’intérieur d’un solénoïde

On appelle solénoïde une longue bobine de fil enroulé de façon hélicoïdale (figure de gauche).
Le diamètre de la bobine est petite par rapport à sa longueur.

La figure de droite montre une coupe transversale de l’axe central d’un solénoïde étiré. Les
parties arrière de cinq spires sont illustrées, comme le sont quelques lignes du champ ma-
gnétique produit par un courant dans le solénoïde. Chaque spire produit autour d’elle des
lignes de champ circulaires. Près de l’axe du solénoïde, les lignes de champ se combinent
en un champ magnétique résultant dirigé le long de l’axe. A l’extérieur du solénoïde, les
lignes de champ sont très espacées ; le champ est très faible à cet endroit.

Aux points situés au-dessus du solénoïde, tel P , le champ produit par les parties supé-
rieures des spires est dirigé vers la gauche et tend à annuler le champ produit par les
parties inférieures des spires, qui est dirigé vers la droite (non indiqué sur le schéma).
Dans le cas spécial d’un solénoïde idéal, le champ magnétique est nul à l’extérieur du
solénoïde.

Pour fixer les idées, la figure ci-dessous représente quelques lignes de champ dans le plan
contenant l’axe d’une seule spire (ou boucle de courant). On peut imaginer une bobine
contenant N spires comme la superposition de N telles boucles.

Les lignes de champ dans une vrai solénoïde de longueur finie sont montrées dans la figure

73
de gauche ci-dessous. Le champ est fort et uniforme aux points intérieurs comme P1 , mais
il est relativement faible aux points extérieurs comme P2 .

Nous allons considérer un solénoïde idéal (figure de droite) où le champ est parfaitement
homogène à l’intérieur et nul à l’extérieur, afin de déterminer le champ intérieur en uti-
lisant la loi d’Ampère. Cette idéalisation est suffisante dans la plupart des cas que l’on
rencontre en pratique lorsque l’on utilise des solénoïde.

Considérons un contour d’intégration rectangulaire abcd de longueur L contenant N


spires de courants. La circulation de B ~ le long de ce contour est facile à calculer. On a
Z Zb Zc Zd Za
~ · d~r =
B ~ · d~r +
B ~ · d~r +
B ~ · d~r +
B ~ · d~r = B L + 0 + 0 + 0 = B L
B
a b c d

La première intégrale (somme) vaut simplement BL puisque le champ est constant en


~ ⊥ d~r sur bc (et B est nul à
norme sur la portion ab. La deuxième intégrale est nulle car B
l’extérieur du solénoïde). De même, les deux dernières intégrales sont nulles également.

Le courant enlacé par le contour d’intégration vaut N I . En appliquant la loi d’Ampère, il


vient
N I
BL = µ0 N I ⇒ B = µ0 à l’intérieur d’un solénoïde
L
Même si ce résultat est exact pour un solénoïde idéal (infiniment long) dont on s’intéresse
qu’à une portion de longueur L, il est valable pour les solénoïdes réels, si on l’applique aux
points intérieurs, à bonnes distances des extrémités du solénoïde.

16.4 Loi de Biot-Savart

~ dans des situations physiques où


La loi d’Ampère est très utile pour évaluer le champ B
ce dernier possède des propriétés de symétrie.

Dans des situations où le champ n’a apparemment pas des propriétés de symétrie évi-
dentes, on préfère utiliser la loi de Biot-Savart qui est équivalente à la loi d’Ampère.

74
Elle s’exprime sous forme différentielle par la relation
µ0 I d~l ∧ ~
u µ0 I dl sin θ
~=
dB et, en norme dB =
4π r2 4π r2
Le vecteur d~l est une portion infinitésimale du fil transportant le courant ; sa direction est
celle de I . Le vecteur ~
u est un vecteur de norme 1 ( u = 1) qui vaut ~r / r (voir figure). Pour
~
obtenir le champ B il faut sommer (intégrer) sur toutes les portions de courant.

En quelque sorte, d B~ correspond au champ magnétique à l’endroit ~r , produit par le tronçon


de fil infinitésimal d~l parcouru par le courant I . Pour obtenir le champ B ~ , il faut ajouter
les contributions dues aux autres portions de courant (c’est-à-dire, il faut intégrer sur tout
le fil).
Z ~
~ (~r ) = µ0 I dl ∧~u
B 2
4π fil r
Voyons quelques applications très utiles dans la pratique.

B au centre d’une boucle de courant

Un courant I circule dans un fil circulaire de rayon R . On veut déterminer l’intensité B


du champ magnétique au centre du cercle. L’intégrale ci-dessus est facile à effectuer. Dans
l’intégrant, r = R est constant et ~u est perpendiculaire à ~l . Les vecteurs d B
~ sont tous dans
la même direction. On a donc, sachant que dl = Rd θ
Z2π
µ0 I dl µ0 I µ0 I 2 π µ0 I
Z Z Z
B= dB = 2
= 2
Rd θ = dθ =
fil 4π fil r 4π R 0 4π R 0 2R
En résumé
µ0 I
B= au centre d’une boucle de courant
2R

B en un point quelconque de l’axe de symétrie d’une boucle de courant

La figure montre une boucle de courant de rayon R . Le plan de la boucle est perpendicu-
laire à la page et on ne voit que la moitié arrière de la boucle. On veut déterminer le champ
~ en un point P de l’axe de symétrie (Oz) de la boucle.
B

75
On applique la loi de Biot-Savart à un élément infinitésimal dl situé du côté gauche de
la boucle. Le vecteur d~l de cet élément pointe perpendiculairement vers l’extérieur de la
page. L’angle θ entre d~l et ~ ~ produit au point P se
u est de 90◦ . L’élément de champ d B
trouve donc dans le plan de la page (il est perpendiculaire à ~
u).

On décompose d B ~ en une composante parallèle à l’axe de la boucle dB∥ et une composante


perpendiculaire à cet axe dB⊥ . Par symétrie, la somme vectorielle de toutes les compo-
santes perpendiculaires dB⊥ produites par tous les éléments dl de la boucle est égale à
zéro. On se retrouve seulement avec les composantes axiales dB∥ . Ainsi, on a
Z
B = dB∥ avec dB∥ = dB cos α

En ce qui concerne l’élément d~l de la figure, la loi de Biot-Savart indique que le champ
infinitésimal dB situé à une distance r est
µ0 I dl sin(90◦) µ0 I dl
dB = =
4π r2 4π r 2
On obtient donc
µ0 I cos α
dB∥ = dl
4π r 2
Exprimons r et cos α en fonction de la distance z qui repère le point P sur l’axe.
p R R
r= R 2 + z2 et cos α = =p
r R 2 + z2
Il vient alors pour dB∥
µ0 I R
dB∥ = dl
4π(R 2 + z2 )3/2
Pour le champ B au point P (situé à la distance z du centre de la boucle), on calcule
µ0 I R
Z Z
B = dB∥ = dl
4π(R 2 + z2 )3/2

76
R
Or, comme dl représente simplement la circonférence 2πR de la boucle,

µ0 I R 2
B( z) = sur l’axe d’une boucle de courant
2(R 2 + z2 )3/2

Lorsque z = 0, on retrouve bien le champ au centre de la boucle que l’on a calculé plus
haut.

B sur l’axe d’une bobine

On considère une bobine de longueur l , de rayon R , contenant N spires traversée par un


~ n’est pas le même au centre de la bobine que vers ses extrémités.
courant I . Le champ B

~ est défini par les angles θ1 et θ2 .


Sur la figure, le point P où l’on veut calculer le champ B

Le courant total N I créé par les N spires de la bobine de longueur l , peut être assimilé à
une ≪ nappe ≫ de courant de largeur l . On découpe cette nappe en anneaux élémentaires
infinitésimaux de largeur dx et de rayon R (le rayon de la bobine). Ainsi, dans chaque
anneau circule un courant d I donné par
dx
dI = NI
l
On utilise ensuite la relation obtenue pour le champ créé par un seul anneau et l’on intègre
sur les anneaux.

dx

R θ2 θ1
b b b
x
P

Par symétrie, B~ en P est selon l’axe Ox. Chaque anneau élémentaire produit un champ
qui vaut (en utilisant la formule pour le champ sur l’axe trouvée plus haut)
µ0 R 2 dx µ0 R 2 dx
dB = N I = NI
2(R 2 + x2 )3/2 l 2r3 l

77
Pour intégrer sur tous les anneaux, il est préférable de faire un changement de variable.
On considère l’angle θ entre l’axe du cylindre et la droite passant par P et le bord de
l’anneau considéré. L’angle θ varie entre :

θ1 ≤ θ ≤ π − θ2

Le dessin ci-dessous

dx rd θ

R r
θ
π−θ
b

montre que

r dθ r dθ R
= sin(π − θ ) ⇒ dx = et sin θ =
dx sin θ r
En introduisant ces expressions dans la relation pour dB, on a

µ0 R 2 r d θ N I µ0 N I
dB = = sin θ d θ
2 r3 sin θ l 2 l
On intègre finalement sur θ entre θ1 et π − θ2
Zπ−θ2
µ0 N I µ0 N I
B= sin θ d θ = (− cos(π − θ2 ) + cos θ1 )
2 l θ1 2l

On retiendra

µ0 N I
B(P ) = (cos θ1 + cos θ2 ) sur l’axe d’une bobine
2l

Lorsque la bobine est un solénoïde, cela signifie que θ1 comme θ2 sont très petits et donc
leurs cosinus proches de 1. On retrouve ainsi le champ obtenu pour un solénoïde (B =
µ0 N I / l ).

16.5 Bobine et barreau aimanté

L’allure des lignes de champ magnétique d’une bobine parcourue par un courant est tout à
fait semblable aux lignes de champ d’un aimant en forme de barreau. On associe donc un
pôle nord (N) et un pôle sud (S) à une telle bobine avec la même convention faite pour un
aimant (figure de gauche). En changeant le sens du courant, on échange le rôle des pôles.

78
Deux telles bobines placées l’une à la suite de l’autre s’attirent si le pôle N de l’une est
proche du pôle S de l’autre ou se repoussent si deux mêmes pôles sont proches (N-N ou
S-S), comme pour deux aimants.

En plaçant un noyau de fer doux à l’intérieur de la bobine (figure de droite), on renforce


considérablement (d’un facteur 100 à 1000) le champ magnétique initialement présent en
l’absence du noyau. En effet, le champ magnétique produit par le courant oriente tous les
domaines de Weiss dans la même direction ce qui amplifie le champ initial. On utilise un
tel dispositif dans les électro-aimants.

17 Loi de Gauss relative au champ magnétique

~ à travers une surface


Nous avions vu dans le cas du champ électrique que le flux de E
fermée quelconque S (surface de Gauss) est, au facteur ǫ0 près, égal à la charge totale
enfermée par la surface S .

Dans le cas du champ magnétique B ~ dont les lignes de champ sont fermées sur elles-
~ à travers S (surface
mêmes (pas de charge magnétique), la loi de Gauss dit que le flux de B
fermée quelconque) est toujours nul. Cette loi est vérifiée pour tous les champs magné-
tiques.

Avec les mêmes notations introduites pour la loi de Gauss relative au champ électrique, il
vient maintenant
n
~i
~ i · ∆S
X
Φ= B
i=1
L’expérience permet d’énoncer la loi suivante :

Loi de Gauss : le flux du champ magnétique à travers une surface de Gauss (donc fermée)
est toujours nul. Nous noterons :

Φ=0 ~
Loi de Gauss relative à B
Une conséquence de cette loi est la suivante. Dans une région où les lignes de champ se
resserrent, le champ devient plus intense et vice-versa. C’est le cas par exemple proche
des pôles d’un barreau aimanté.

79
En effet, considérons un ≪ tube de champ ≫ (la surface de Gauss a sa partie latérale qui
coïncide avec des lignes de champ) assez mince pour qu’on puisse en définir des sections
~ ait une valeur bien déterminée.
droites (S 1 et S 2 ) et que sur chacune d’elles l’intensité de B
En appliquant la loi de Gauss à la surface fermée constituée par une portion de tube et
deux sections droites, on obtient :
~1 + B
~1 · S
Φ=0⇒B ~2 = 0 ⇒ B1 S 1 − B2 S 2 = 0 ⇒ B1 S 1 = B2 S 2
~2 · S

~ est donc inversement proportionnel


Le long d’un tube de champ, le champ magnétique B
à la section du tube.

Dans le champ électrique, cette règle n’est valable que dans une portion d’espace où il n’y
a pas de charge. Dans le champ magnétique, elle est toujours valable.

18 Loi d’induction de Faraday

Jusqu’ici, nous avons étudié deux aspects de la relation qui existe entre l’électricité et
le magnétisme. Premièrement un courant électrique produit un champ magnétique et
deuxièmement un champ magnétique exerce une force sur un courant électrique ou sur
une charge électrique en mouvement. Ces découvertes datent des années 1820-1821. Par
la suite les scientifiques se sont demandés si, à l’inverse, un champ magnétique pouvait
faire apparaître un courant électrique, ou de façon plus fondamental un champ électrique.
Quelque dix ans plus tard, l’américain Joseph Henry et l’anglais Michael Faraday ont
confirmé cette hypothèse chacun de son côté. Ce phénomène et certaines de ses applica-
tions ont entraîné des transformations majeures dans le monde technologique moderne.

Pour produire un courant électrique au moyen d’un champ magnétique, Faraday a uti-
lisé un dispositif semblable à celui de la figure ci-dessous. En fermant l’interrupteur, un
courant parcourt le circuit relié au générateur et créé un champ magnétique. Ce champ
magnétique traverse de ce fait l’intérieur de l’autre circuit relié à un galvanomètre (ampè-
remètre très sensible). Notons que ce dernier circuit n’est relié à aucune source de courant.
Faraday espérait observer un courant dans ce circuit. Malheureusement il n’observa rien
du tout. En réalité, il s’aperçut qu’au moment où il fermait l’interrupteur, l’aiguille du
galvanomètre se déplaçait rapidement dans un sens et revenait ensuite à la position 0,
et lorsqu’il l’ouvrait, elle se déplaçait dans l’autre sens pour revenir ensuite à 0. En re-
vanche, un courant continu parcourant le premier circuit ne produisait aucun courant
dans l’autre.

80
Faraday en a conclu qu’un champ magnétique doit être variable pour faire apparaître un
courant électrique, appelé courant induit. Lorsque le champ magnétique varie, un cou-
rant traverse le galvanomètre comme si le circuit contenait une source de tension appelée
tension électromotrice induite ( e.m.).

Faraday a également observé que lorsqu’on introduit rapidement un aimant à l’intérieur


d’une bobine de fil (voir figure), un courant électrique induit se met à parcourir le fil.
Lorsqu’on le retire aussi rapidement, on obtient le même résultat en sens inverse.

Faraday a étudié de façon quantitative les facteurs influant sur la tension électromotrice
induite provoquant le courant induit dans le fil conducteur. Cette tension induite est pro-
portionnelle non pas au rythme de variation du champ B ~ , mais plutôt à celui du flux
magnétique Φ. Mathématiquement la relation s’exprime

Loi d’induction de Faraday :


Uind. = −
dt

Voyons ce que signifie cette relation. Le membre de gauche étant une tension (s’exprimant
en volts) signifie qu’en réalité il existe dans l’espace un champ électrique appelé champ
électrique induit et noté E ind. , dont l’existence provient de la variation au cours du
temps du champ B ~ . Notons Γ un chemin fermé quelconque dans l’espace (qui peut être
la courbe décrite par un fil conducteur, mais pas nécessairement). Effectuons les choix
suivants :

1. Décidons d’un sens de parcours de Γ ;


2. imaginons une surface ouverte quelconque (notée Σ) dont le bord est justement Γ ;
3. Orientons la surface Σ en accord avec la règle du tire-bouchon.

81
Gardons en mémoire (voir figure ci-dessous) les deux orientations possibles de la surface
Σ et des sens de parcours (s.p.) du chemin Γ en accord avec la règle du " tire-bouchon "

~
S s.p.
Γ Γ
Σ ou
Σ
s.p.
~
S

Ces choix étant faits, la loi d’induction s’exprime par :

~ ind. · d~r = − d
Z Z
E ~ · d~s
B
Γ dt Σ

Notons que la surface Σ s’appuyant sur la courbe Γ est quelconque. Un autre choix, disons
~ sur une
Σ′ , conduirait à un flux identique, cela découle du fait que le flux du champ B
surface fermée est toujours nul. Ainsi
Z Z
~
B · d~
s= ~ · d~s
B
Σ Σ′

Notons encore que le signe − dans la loi d’induction est fixé par les conventions d’orienta-
tions choisies (la tension dépend du sens de parcours de Γ, le flux dépend de l’orientation
de la surface Σ). Avec ces conventions, l’expérience monte qu’alors Uind. = −dΦ/dt.

Si la courbe Γ coïncide avec la position qu’occupe un fil conducteur formant un circuit


fermé, le champ électrique induit régnant à l’intérieur de ce conducteur met en mouvement
les électrons libres de ce dernier et provoque ainsi un courant électrique qui est le courant
induit.

Remarque importante :
Le champ électrique induit E ~ ind. a une propriété bien spécifique : son intégrale le long d’un
contour fermé Γ n’est pas nulle en général (vu la loi d’induction) contrairement au cas d’un
champ électrostatique où cette intégrale est toujours nulle. Pour cette raison on ne peut
pas définir de potentiel électrique dans le cas d’un champ électrique induit, car l’intégrale

82
du champ sur une courbe joignant un point A à un point B dépend du chemin emprunté
contrairement au cas électrostatique.

Lorsqu’un champ magnétique B ~ ( t) varie au cours du temps en un certain lieu, il engendre


en ce lieu un champ électrique induit E ~ ind. ( t).

~ ( t) ⇔ existence de E
variation de B ~ ind. ( t)

Ce champ électrique induit n’a pas la même structure que celui engendré par des charges
électriques, que l’on va noter E~ cha. (qui est le champ électrostatique habituel). On a en
général
Z Z
~ cha. · d~r = 0
E alors que ~ ind. · d~r 6= 0 en général
E (2)
chemin fermé chemin fermé

En général, dans un système électrique, les deux types de champs électriques cités sont
~.
présents et s’additionnent pour former le champ électrique total E
~ =E
E ~ cha. + E
~ ind.

~ ind.
Par la première propriété de (2), la loi d’induction peut s’écrire aussi en remplaçant E
~ . Il vient
par E

~ · d~r = − d Φ
Z
E
Γ dt

Rappelons la définition de la tension électrique U le long d’un chemin joignant le point P


~ qui intervient dans la définition) :
au point Q (c’est le champ électrique total E
Z
U= ~ · d~r
E U dépend en général du chemin emprunté
de P à Q

Comme exemple, considérons un champ magnétique uniforme limité par une surface cy-
lindrique à l’extérieur de laquelle le champ est nul. Supposons que l’intensité de ce champ
croisse ou décroisse linéairement avec le temps. On veut déterminer dans tout l’espace, la
direction, le sens et l’intensité du champ électrique induit.

Le système possède une symétrie de révolution, donc les lignes du champ électrique induit
ont également cette symétrie. Les seules lignes fermées qui possèdent cette symétrie sont
des cercles centrés sur l’axe (de symétrie) et perpendiculaires à celui-ci.

83
Pour appliquer la loi d’induction, il faut une courbe fermée. Choisissons une ligne du
champ électrique induit. Fixons un vecteur S~ et un sens de parcours conformément aux
conventions. On a

~ = B ( t) S
~ ( t) · S dΦ dB
Φ( t ) = B ⇒ Uind ( t) = − = −S
dt dt
Lorsque le champ magnétique croît, Uind est négatif. Cela implique que le champ électrique
induit est dans le sens opposé au sens de parcours. Lorsque le champ magnétique décroît,
Uind est positif. Dans ce cas, le sens du champ électrique coïncide avec le sens de parcours.

Déterminons l’intensité du champ électrique à l’extérieur du cylindre où règne le champ


magnétique. Notons R le rayon de ce cylindre. Le flux de B ~ vaut BπR 2 . La circulation du
champ électrique est −E ind 2πr lorsque B croît et E ind 2πr lorsque B décroît. La distance r
désigne le rayon du cercle de la ligne de champ considérée. Il vient donc (dans le cas où B
croît) :
1 dB R 2
−E ind 2πr = −πR 2 dB/ dt ⇒ E ind =
2 dt r
Le champ électrique est donc inversement proportionnel à la distance à l’axe. Il en va
autrement pour le champ électrique situé à l’intérieur du cylindre. En effet, le flux vaut
maintenant Bπr 2 où r < R . Il vient comme avant (pour B croissant)
1 dB
E ind = r
2 dt
On voit donc que le champ électrique est proportionnel à la distance à l’axe tant que r < R .

Exemple d’une spire perpendiculaire à un champ magnétique variable

Un conducteur en forme de boucle est placé dans le champ électrique induit (par un champ
~ variable) de sorte que le fil suive les lignes du champ électrique. Distinguons le cas d’une
B
boucle fermée et celui d’une boucle ouverte.

Dans le premier cas, en chaque point du fil, le champ électrique induit est tangent au
conducteur. C’est la situation la plus favorable pour la production d’un courant induit.

84
Celui-ci a le même sens que le champ induit. Sa valeur est fournie par la loi d’Ohm I =
Uind /R où R est la résistance du fil.

Pour trouver le sens du courant induit, on peut utiliser la règle suivante, appelée règle de
Lenz (qui est une conséquence de la loi de Faraday).

Règle de Lenz : le courant induit est toujours dans un sens tel qu’il tend à s’opposer à la
variation du flux qui l’a provoqué.

Pour appliquer cette règle, il faut considérer le champ magnétique créé par le courant
induit. Si l’effet d’induction est provoqué par une augmentation de flux, ce champ magné-
tique est orienté de façon à faire diminuer le flux. Et vice versa.

Dans le second cas, les charges déplacées par le champ électrique induit s’accumulent
aux extrémités du conducteurs. L’une des extrémités devient positive et l’autre négative.
Le champ électrique est alors nul à l’intérieur du fil. Appliquons la loi d’induction à une
courbe allant le long de la spire et se refermant en traversant la fente PQ de celle-ci. La
tension induite s’écrit

Uind = UQP (fil) + UPQ (air) = UPQ (air)

La dernière égalité suit de ce que le champ électrique est nul dans le fil et donc UQP (fil) = 0.
Dans cet exemple, la tension induite est due au champ électrique intense dans la fente et
produit par les charges accumulées aux extrémités du conducteur.

19 Générateur, moteur et alternateur

Nous allons considérer trois exemples. Dans les deux premiers, il règne un champ ma-
gnétique statique (indépendant du temps) dans une partie du système considéré. Il n’y a
donc pas de champ électrique induit dans le sens défini ci-dessus. En revanche, il appa-
raît une variation de flux du champ magnétique causé par la déformation d’une partie du
circuit électrique. Dans le troisième exemple, un champ magnétique variable au cours du
temps va engendrer un champ électrique induit et l’on pourra appliquer la loi d’induction
de Faraday.

19.1 Exemple 1 : le générateur de courant continu

Une tige conductrice CD se déplace à vitesse ~ v constante dans un champ magnétique


~
homogène B. Elle est en contact permanent avec deux rails conducteurs reliés à une ré-
sistance r . Cette dernière représente la résistance de l’ensemble du circuit, de sorte que la
tige CD ainsi que les rails sont supposés parfaits (sans résistance électrique). La tige, le
champ B~ et la vitesse ~v sont mutuellement perpendiculaires (voir la figure).

85
Pour décrire l’aspect électrique de ce système, supposons d’abord l’interrupteur ouvert.
Chaque électron libre de la tige, de charge − e, subit la force de Lorentz (en direction de la
borne D ) F~Lor = (− e) ~
v∧B ~ . La migration de ces électrons libres en direction de D provoque
une polarisation de la tige et donc un champ électrique E ~ dont les lignes de champ partent
de la bornes C et aboutissent à la borne D . Rapidement, le mouvement de ces électrons
cesse dès que la force électrique F ~ e = (− e ) E
~ que subit l’électron libre s’oppose à F
~Lor . Une
fois l’équilibre atteint, le champ E ~ homogène dans la tige vaut :
³ ´
(− e ) E~ +~
v∧B~ =0 ⇒ ~ = −~
E v∧B~

−−→ −−→
On peut donc calculer la tension UCD . Notons par ~l le vecteur CD (~l = CD )

~ ·~l = E l = v B l > 0
UCD = E

On ferme maintenant l’interrupteur. Dès les premiers instants un courant (dirigé dans
le sens des aiguilles d’une montre) va naître et traverser la résistance r . Cela a pour
conséquence une diminution de l’intensité de E ~ dans la tige CD . La force de Lorentz
~Lor reprend momentanément (temps très bref) le dessus sur la force électrique F
F ~ e ce
qui engendre un mouvement d’électrons en direction de D afin de rétablir l’équilibre entre
ces deux forces. L’équilibre ayant lieu quasiment instantanément, on peut donc dire qu’à
chaque instant la résultante des forces sur chaque électron libre est nulle.

Remarque : dans un conducteur de résistance nulle parcouru par un courant I , le champ


électrique est toujours nul, excepté si ce conducteur est en mouvement dans un champ
magnétique. En réalité, c’est la résultante des forces qui s’exercent sur chaque électron
qui doit être nulle.

Dans le circuit fermé CFEDC la deuxième loi de Kirchhoff s’applique car E ~ ind. est nul
partout vu que le champ B ~ est indépendant du temps. Choisissons le sens de parcours
du circuit dans le même sens que le courant par exemple. De ce fait, l’orientation de la
~ ( t) sous-tendue par le circuit est orientée dans le sens opposé à B
surface S ~ (vu la règle du
~
tire-bouchon). Le flux de B à travers cette surface est

~ ( t) = −BS ( t)
~·S dΦ dS dx
Φ( t ) = B ⇒ = −B = −Bl = −Blv = −UCD
dt dt dt
Par la deuxième loi de Kirchhoff, sachant que UFE = rI ,


UFE + UDC = 0 ⇒ rI = UCD = −
dt

86
Au vu de ce dernier résultat, la tension UCD a la même allure qu’une tension induite. On
la note parfois aussi Uind. . Dans cet exemple, elle joue le rôle d’une tension électromotrice
Uem .

On constate que le sens du courant I est bien en accord avec la règle de Lenz.

Procédure à suivre : dans un cas simple comme celui-ci, pour ne pas se compliquer l’exis-
tence avec des choix de sens de parcours et autres orientations, on procède ainsi : 1) on
repère le sens du courant induit par la règle de Lenz ; 2) on calcule d Φ/ dt en valeur abso-
lue qu’on appelle Uind. ; 3) on applique Uind. = rI .

19.2 Exemple 2 : le moteur à courant continu

On considère un circuit électrique particulier comprenant un générateur (idéal) de tension


constante U g = U AB > 0 relié à un moteur à courant continu à l’intérieur duquel règne un
champ magnétique constant B ~ . On désigne par r la résistance interne du moteur. Toutes
les autres connexions sont supposées idéales (sans résistance). La présence des charges
aux bornes du générateur assure l’existence d’un champ électrique de type E ~ cha. dans son
voisinage.

La partie du circuit qui se déforme se situe à l’intérieur du moteur. Il s’agit du barreau


conducteur C ′ D ′ qui peut glisser le long des deux conducteurs jouant le rôle de rails (voir
figure). Les bornes du moteur idéal sont C et D ; celles du moteur réel sont C et E . Notons
que UDE = r I ( I est le courant qui sort du générateur).

Considérons le chemin fermé BACDEB. La deuxième loi de Kirchhoff s’applique puisque


~ ind. = 0. Il vient donc :
E

UBA + UCD + UDE = 0 ⇒ −U g + r I = −UCD (3)


−−−→
Le barreau C ′ D ′ de longueur l (~l = C ′ D ′ ) se situe dans la région où le champ B ~ existe. Etant
parcouru par le courant I , le barreau subit la force de Laplace F ~Lap = I ~l ∧ B
~ et se met ainsi
en mouvement. Désignons par ~ v la vitesse du barreau à un instant donné. Chaque électron
libre du barreau, de charge − e, subit donc la force de Lorentz F ~Lor = (− e) ~
v∧B ~ . Le barreau
′ ′
C D étant de résistance nulle, cela a pour conséquence que la somme totale des forces qui
s’exercent sur chaque électron libre de ce dernier doit être nulle (voir remarque exemple
1). Il existe ainsi un champ électrique E ~ dans le barreau engendré par une polarisation de
ce dernier. Ce champ E ~ est donc de type E ~ cha. .

87
~ se déduit de l’opposition de la force électrique (− e)E
La valeur de E ~ et de la force de
Lorentz.
³ ´
(− e ) E~ +~ ~ =0
v∧B ⇒ ~ = −~
E v∧B~

On peut calculer la tension UCD aux bornes du moteur (qui est la même que UC ′ D ′ ) comme
dans l’exemple 1. On obtient

UCD = Blv = >0 l’orientation est dans le même sens que I
dt
A l’aide de (3) il vient finalement :

Ug = r I + (4)
dt
Une fois le régime stationnaire atteint, la force de Laplace égale le poids de la charge pour
une tension U g fixe. Ceci pour les mêmes raisons évoquées lors de la discussion du moteur
à courant continu (stator-rotor).

En revanche, si U g croît, le courant augmente momentanément (pendant la variation de


U g ), la vitesse du barreau également et donc aussi la tension UCD . Dès que U g cesse de
varier, le courant redevient constant et sa valeur est fixée par l’égalité FLap = mg (le poids
de la charge). Pour une tension U g constante, le courant est donc constant et la vitesse du
barreau également (elle peut être nulle si U g n’est pas assez grande).

Si l’on empêche au barreau d’avancer (on bloque le moteur), la tension UCD devient nulle,
la tension U g du générateur est donc la même que celle aux bornes de la résistance r du
moteur. Le courant qui la traverse peut donc devenir important et endommager le circuit.
Ainsi la présence d’une tension UCD évite au courant de devenir trop important, d’où son
nom de tension contre-électromotrice.

La tension UCD = d Φ/ dt a la même allure qu’une tension induite (en valeur absolue). Dans
cet exemple, elle joue le rôle d’une tension contre-électromotrice Ucem . En effet, le travail
de la force de Laplace sur le déplacement dx est
dW = I lBdx = IBdS = I d Φ
La puissance P (mécanique) associée est donc
dW d Φ
P= = I = Ucem I
dt dt

88
19.3 Exemple 3 : l’alternateur

1ère version (figure de gauche)

L’aimant mis en rotation engendre un champ magnétique variable au cours du temps. Un


champ électrique induit (E ~ ind. ) existe à l’intérieur de l’alternateur. On va faire l’hypothèse
qu’en dehors de l’enceinte où se trouvent l’aimant en rotation et la bobine, le champ élec-
trique induit est nul (il suffit que l’enceinte soit assez grande).

Le champ E ~ ind. à l’endroit où se trouve la bobine varie au cours du temps. Par conséquent,
en reliant un oscilloscope aux bornes A et B de l’alternateur, on mesure une tension va-
riable U AB ( t). Remarquons que U AB est la tension entre A et B par un chemin qui passe
à l’extérieur de l’enceinte, là où E ~ ind. est nul. Ainsi cette tension est indépendante du
chemin qui joint A et B. Il n’en est évidemment pas ainsi si le chemin passe à l’intérieur
de l’enceinte.

Le champ électrique total à l’intérieur de la bobine (supposée idéale) est nul à chaque ins-
tant. Le champ électrique induit E ~ ind. s’oppose au champ créé par la polarisation de la
~ ~ ~
bobine E cha. (E ind. ( t) + E cha.( t) = 0). En alternance, les bornes A et B se chargent positive-
ment ou négativement.

On peut appliquer dans cette situation la loi d’induction de Faraday. Considérons le che-
min fermé qui part de A vers B en suivant la bobine, puis de B vers A par l’extérieur de
l’enceinte. Il vient

U AB (bob.) + UBA (ext.) = −
dt
où U AB (bob.) est la tension le long du conducteur, qui est donc nulle puisque le champ
total est nul dans ce dernier, et où UBA (ext.) est la tension à l’extérieur de l’enceinte. Cette
tension ne dépend donc pas du chemin qui joint B et A (puisqu’elle ne dépend que de E ~ cha. ).
On a donc

UBA (ext.) = −
dt
La variation du flux Φ provient exclusivement de la variation du champ magnétique à
l’intérieur de la bobine, le reste du circuit n’intervient pas. Notons que le sens de parcours

89
de la bobine est défini de A vers B et que l’orientation des spires est fixée en accord avec
la règle du tire-bouchon.

Finalement, si l’on ferme le circuit sur une résistance R et qu’on applique la deuxième loi
de Kirchhoff (sur un contour passant à l’extérieur de l’enceinte), on obtient :


R I = UBA (ext.) = − I est compté positivement dans le sens de parcours
dt

2ème version (figure de droite)

On peut imaginer un alternateur où cette fois la bobine tourne et l’aimant est statique.
La variation du flux à l’intérieur de la bobine est identique à celle du cas précédent, si
la bobine tourne dans le sens opposé de celui de l’aimant (voir les figures). Cette fois,
l’origine de cette variation de flux est due à la force de Lorentz qui s’exerce sur les électrons
libres et qui induit instantanément un champ électrique dont l’origine est la polarisation
de la bobine (voir exemples 1 et 2). Aucun champ électrique de type E ~ ind. n’est présent
ici. La deuxième loi de Kirchhoff s’applique aussi bien à l’intérieur qu’à l’extérieur de
l’enceinte. Vu de l’extérieur, on ne peut pas distinguer cet alternateur de celui de la version
précédente. On a également R I = U AB si on le relie à une résistance R . Contrairement à
la situation précédente on a

U AB (bob.) + UBA (ext.) = 0

mais la tension entre A et B par la bobine se calcule comme dans les exemples 1 et 2, à
savoir :

~ · d~r = d Φ
Z ³ ´
U AB (bob.) = − ~v∧B (5)
de A à B dt

L’intégrale n’est pas aussi simple à calculer que dans les exemples 1 et 2 (la vitesse ~
v ainsi
que le champ B ~ ne sont pas constants le long de la bobine). Quoi qu’il en soit, on peut
encore montrer que cette intégrale vaut d Φ/ dt et qu’ainsi U AB = d Φ/ dt comme dans la
situation précédente.

Preuve : (on peut la sauter, il n’y a pas besoin de la connaître !)

³ ´
~ · d~l ∧ d~r
B ~ dϕ
~ · dS
³ ´ ³
~ · d~r = B~ ∧~
´
~ · (~ B
− ~v∧B v · d~r = B v ∧ d~r ) = = =
dt dt dt

v = d~l / dt et d ϕ est le flux de B


où ~ ~ à travers l’aire du parallélogramme infinitésimal défini
par d~l et d~r. L’aire de cette surface orientée est d S ~ = d~l ∧ d~r . Cette surface correspond
~
au déplacement ( d l ) d’une portion ( d~r ) de la bobine pendant l’intervalle de temps dt. La
somme (l’intégrale) de tous ces flux élémentaires d ϕ correspondant au déplacement de
chaque portion de la bobine donne le flux d Φ qui correspond à la variation du flux dans
toute la bobine durant le laps de temps dt. Ainsi la relation (5) est démontrée.

90
Remarque concernant la ≪ règle du flux ≫

Il est important de faire la distinction entre les deux aspects qui sont responsables de
l’apparition d’une tension induite (voir les deux versions de l’alternateur). Le fait qu’un
champ B ~ dépende du temps implique l’existence d’un champ électrique induit dans l’es-
pace. L’intégrale de ce champ électrique le long d’un parcours fermé (qui peut être un fil
conducteur mais pas nécessairement) est relié au taux de variation du flux de B ~ à travers
ce parcours. C’est cela le contenu de la loi d’induction de Faraday.

Par ailleurs, lorsqu’un circuit (fil conducteur) se déplace ou se déforme dans un champ ma-
gnétique statique (indépendant du temps), on observe également un courant (si le circuit
est fermé) ou une tension aux bornes du fil (si le circuit est ouvert). La raison de cette ten-
sion est ici différente : c’est la force de Lorentz qui déplace les électrons libres du fil. Il n’y
a pas de champ électrique induit dans l’espace. On a montré juste avant que cette tension
peut encore s’exprimer par le taux de variation du flux du champ B ~ à travers le circuit. Or,
ceci est vrai si la matière dont le circuit est constitué reste la même. Sinon cette règle du
flux n’est pas valable. Mais dans tous les cas, on peut utiliser : soit la force de Lorentz, soit
la loi d’induction de Faraday.

Comme illustration, nous allons discuter deux situations pour lesquelles la règle du flux
n’est pas valable.

91
La figure de dessus illustre un circuit électrique un peu particulier. Lorsque le disque
tourne, il apparaît bien une tension aux bornes du voltmètre alors qu’il n’y a pas ici de
variation de flux. C’est la force de Lorentz qui est responsable de cette tension.

La situation de dessous montre deux plateaux de cuivre légèrement incurvés placés dans
un champ magnétique perpendiculaire à leurs surfaces. Chaque plateau est relié au volt-
mètre. Ils sont en contact au point P , le circuit est donc fermé. En faisant glisser les
plateaux l’un contre l’autre, on peut amener le point de contact en P ′ . Si l’on imagine le
circuit dans les plateaux comme étant spécifié par les traits pointillés, le flux du champ
varie beaucoup lorsque le point de contact passe de P à P ′ . Pourtant, le mouvement de
glissement peut être effectué aussi lentement que l’on veut. La tension aux bornes du volt-
mètre peut donc devenir pratiquement nulle (la force de Lorentz est très faible). Donc une
variation de flux n’engendre pas nécessairement une tension induite.

19.4 Courant sinusoïdal produit par une dynamo

Il s’agit ici d’un alternateur (type 2ème version) où


un cadre tourne à vitesse angulaire ω constante
(appelée aussi pulsation) dans un champ ma-
gnétique B ~ uniforme. Les bornes de l’alternateur
sont fermées sur une résistance R dans laquelle
circule le courant induit dont nous allons calculer
sa dépendance temporelle. Notons C et D les
bornes de l’alternateur (définies par les deux
bagues).

En vertu de (5), on peut calculer la variation du


flux de B~ à travers la surface mobile (d’aire S ) qui
est "immergée" dans le champ B ~ , pour obtenir la
tension UCD et également le courant I dans le cir-
cuit lorsqu’il est fermé. Dans la position actuelle
(voir figure), l’angle θ est aigu (il croît), le flux est
~ ) et décroît. Comme
positif (voir l’orientation de d S
la vitesse angulaire ω est constante, l’angle croît
linéairement avec le temps : θ ( t) = ω t (on pose
qu’à l’instant t = 0 l’angle θ est nul).

On a donc :
~ ( t) = B S cos(ω t)
~·S
Φ( t ) = B

La tension UCD (le long du cadre mobile selon le sens de parcours idoine) vaut par (5) :

UCD ( t) = = −ω B S sin(ω t) = −Û sin(ω t)
dt
avec Û = ω B S > 0 la tension de crête. A l’instant t considéré dans la figure, UCD est
négative (ω t < π/2). En fermant l’alternateur sur une résistance R , le courant va donc la

92
traverser dans le sens indiqué (pour cet instant-là). Et l’on a :
UCD ( t) Û
I ( t) = = − sin(ω t) = − Î sin(ω t)
R R
avec Iˆ = Û /R > 0 le courant de crête. En d’autres termes, la tension et le courant sont
des fonctions périodiques sinusoïdales du temps dans ce circuit. Si le cadre effectue 50
tours par seconde (la fréquence f = 50 Hz, rappelons que ω = 2π f ), le courant ainsi que la
tension changent de sens également 50 fois par seconde. C’est le cas dans notre réseau de
distribution.

Quel joli calcul bien compliqué qu’on a fait là : se souvenir de la relation (5), tous ces sens
de parcours, orientation et autre règles de signes...N’y aurait-il pas un principe simple
pour venir nous sauver de tout cela ? Mais si ! La règle de Lenz. On voit tout de suite que
le courant I doit aller dans le sens indiqué puisqu’il s’oppose à la variation du flux qui l’a
créé. Pour le reste, on calcule (comme pour la loi d’induction de Faraday) : Uind. = − d Φ/ dt
et lorsque le circuit est fermé sur une résistance R : Uind. = R I . On retiendra cependant
que le courant et la tension aux bornes de la résistance sont en phase. Ce ne sera pas
toujours le cas, si l’on place autre chose qu’une résistance ! Affaire à suivre...

Puissance électrique moyenne sur une période

Dans l’exemple ci-dessus, la puissance électrique produite par la dynamo à l’instant t est
P ( t) = U ( t) I ( t) = Û Î sin2 (ω t)

A titre illustratif, le graphique ci-dessous représente les trois fonctions : Û sin(ω t), Î sin(ω t)
et P ( t) pour f = 2 Hz (ω = 2π f ), Û = 3 V et Î = 2 A. La puissance de crête est donc de 6 W.
La puissance "oscille" à un rythme double que la tension (ou le courant). Elle est naturel-
lement toujours positive (ou nulle).

La puissance moyenne sur une période (notée P̄ ) est définie comme le quotient de l’énergie
produite en une période T par la valeur de la période.
1 T 1 T
ZT
1
Z Z
2
P̄ = P ( t) dt = Û Î sin (ω t) dt = Û Î (1 − cos(2ω t)) dt
T 0 T 0 0 2T

93
On a utilisé l’identité trigonométrique (voir formulaire) :
(1 − cos(2α))
sin2 α =
2
On voit tout de suite que la valeur de cette intégrale est 1/2. Ainsi :
1 Û Î
P̄ = Û Î = p p = Ueff I eff pour un circuit résistif pur
2 2 2
p p
où l’on appelle tension efficace Ueff = Û / 2 et courant efficace I eff = Î / 2. La formule
trouvée est légèrement différente lorsque des éléments autres que des résistances appa-
raissent dans les circuits. Un multimètre indique toujours les valeurs efficaces quand il
s’agit de grandeurs périodiques sinusoïdales (tension ou courant).

20 Auto-induction

Considérons un courant dans un circuit quelconque. Ce courant crée un champ magné-


tique. Si le courant varie, le champ magnétique varie aussi. Or, les variations de champ
magnétique créent un champ électrique induit. Ce champ électrique induit agit sur le cou-
rant donné. Il s’agit donc d’un effet d’induction provoqué par un circuit sur lui-même, on
parle alors d’auto-induction ou de self-induction.

Analysons la situation dans le cas d’une spire de courant. Prenons la convention qui
consiste à définir le sens de parcours ( s.p.) dans le même sens que le courant I . De ce
fait, l’orientation de la surface de la spire est dans le même sens que le champ B ~ créé par
~ est positif. Si le courant croît, B
le courant et ainsi, Φ, le flux de B ~ et son flux croissent, la
dérivée du flux est donc positive. Par conséquent, la tension induite est négative. Le champ
électrique induit est opposé au sens de parcours et donc au courant. Ce champ électrique
a donc tendance à faire diminuer le courant. De manière tout à fait analogue, si le courant
décroît, le champ électrique induit a tendance, cette fois, à maintenir le courant (donc à
s’opposer à sa décroissance).

Le phénomène de self-induction est donc un effet d’inertie dans un circuit. Il s’oppose à


toute variation du courant. Nous verrons que cet effet ne parvient jamais à empêcher les
variations de courant, mais il les freine toujours.

Pour analyser quantitativement le phénomène de self-induction, remarquons d’abord que


le flux créé par un courant est proportionnel à ce courant. En effet, si le courant est am-
~ en chaque point de l’espace voit également
plifié d’un facteur k (quelconque), le champ B
son intensité multipliée par k (conséquence de la loi d’Ampère, ou de Biot-Savart). Il en
est donc encore de même pour le flux qui est multiplié par k. Pour un circuit quelconque
on a donc
Φ=L I s.p. = même sens que I
où la constante L est appelée coefficient de self-induction du circuit considéré. Cette
constante est nécessairement positive au vu de la convention du sens de parcours effec-
tuée.

94
Remarque : on vient d’évoquer la loi d’Ampère dans notre raisonnement, alors que cette
dernière n’est valable, en toute rigueur, seulement si les courants ne dépendent pas du
temps. Cependant, pour des courants variables dans le temps à des fréquences radioélec-
triques ( f < 1012 Hz) et même au-delà, la loi d’Ampère constitue une excellente approxi-
mation.

La tension induite Uind. s’écrit donc dans cette situation (par la loi de Faraday) :

dΦ dI
Uind. = − = −L
dt dt
Cette dernière relation remplace la loi d’induction dans tous les cas où la tension induite
dans un circuit ne provient que des variations de courant dans ce circuit. L’unité de L
est [Vs/A] que l’on appelle henry et que l’on note [H].

Lorsqu’un circuit contient une bobine, le flux d’induction à travers le circuit entier est im-
putable presque exclusivement à la bobine. Il en résulte que le coefficient de self-induction
L du circuit complet peut-être confondu avec celui de la bobine. Lorsqu’un circuit ne
contient pas de bobine, son coefficient de self-induction est très faible et on peut souvent
le négliger.

Etablissons le coefficient de self-induction L pour un solénoïde de longueur l , contenant N


spires de section S . Le champ B~ étant uniforme et perpendiculaire au plans des spires. Le
flux s’écrit :

N I N2 S
Φ = N B S = N S µ0 = µ0 I
l l
Le coefficient L vaut donc

N2 S
L = µ0
l

Si l’on introduit un noyau de fer dans la bobine, le champ B ~ est énormément renforcé. Le
flux d’induction et le coefficient de self-induction augmentent dans les mêmes proportions.

20.1 Enclenchement du courant dans une bobine

Relions une bobine à une batterie. Le courant ne peut s’établir instantanément à cause du
phénomène de self-induction. Cherchons comment il croît en fonction du temps.

Désignons par L le coefficient de self-induction et par U la tension électromotrice de la


batterie. Appelons R la résistance totale du circuit. Il vient (UC A = −U , UBC = 0)

95
I

U /R

I ( t) = U − t/T
¡ ¢
R 1− e
L
T= R

0 t

dI dI
U AB + UBC + UC A = −L ⇒ L +R I =U
dt dt
Le courant satisfait à une équation différentielle linéaire avec second membre (U ). On
cherche d’abord la solution générale de l’équation différentielle sans second membre (U =
0). On détermine ensuite une solution particulière (ici I = U /R ). La solution générale de
l’équation de départ est alors la somme de ces deux solutions. On obtient :

t/L U
I ( t ) = A e −R +
R
Vérifier que cette fonction constitue bien une solution de l’équation différentielle de départ.
La constante A est quelconque a priori. Elle est fixée par les conditions initiales du pro-
blème physique considéré. Dans notre cas, lorsque t = 0, le courant I (0) est nul ( I (0) = 0).
La constante A peut donc être déterminée : 0 = A + U /R donc A = −U /R . Finalement, la
solution de notre problème est
U −R t/L U U³ ´
I ( t) = − e + = 1 − e − t/T où T = L/R constante de temps
R R R
Le graphique ci-dessus montre que le courant croît, d’abord rapidement, puis de moins en
moins vite. Il atteint de manière asymptotique sa valeur maximum U /R représentée par
la droite horizontale.

20.2 Energie d’une bobine traversée par un courant

Considérons une bobine (self) de coefficient de self-induction L traversée par un courant


I . Notons R la résistance de la bobine. Cette dernière est reliée aux bornes d’un généra-
teur de tension fixe U . Nous avons déjà remarqué que le courant croît progressivement et
s’approche de sa valeur limite. L’équation du circuit est

96
R I ( t) − U = −L İ ( t) ⇒ U I ( t) = R I ( t)2 + L I ( t) İ ( t)

Le terme positif U I ( t) est la puissance totale fournie à la bobine réelle par le générateur.
Cette puissance se décompose en deux termes. Le premier, R I ( t)2, est la puissance ther-
mique dissipée dans la résistance de la bobine. Le second s’interprète comme la puissance
nécessaire à l’établissement du courant I ( t) compte tenu de l’effet d’auto-induction. Plus
précisément, l’énergie dW reçue par la bobine pendant l’intervalle de temps dt est

dW = R I 2 dt + L I d I

Le deuxième terme LI d I montre que tout accroissement du courant dans une bobine cor-
respond pour elle a une absorption d’énergie. Supposons l’effet Joule négligeable (R ≃ 0)
devant l’autre terme. Lorsque le courant s’établit et passe de la valeur 0 à la valeur I 1 , la
bobine reçoit l’énergie
ZW ZI1
1
W= dW = L I d I = L I 12
0 0 2

Une bobine dont la self-induction vaut L et dans laquelle circule un courant I possède une
énergie

1
W= L I2
2

Cette énergie est en fait accumulée dans le champ magnétique créé dans la bobine par le
courant.

20.3 Surtension dans une ampoule

Une bobine d’inductance L et de résistance r est branchée en parallèle avec une ampoule
de résistance R (qu’on suppose indépendante de la température) sur un générateur de
tension continue U g > 0. Lorsque l’interrupteur est fermé (figure de gauche), deux courants
I a et I b traversent l’ampoule et la bobine respectivement. La tension Ua ≡ U AB aux bornes
de l’ampoule est alors positive et est égale à la tension U g (Ua = U g ). De même pour la
tension Ub aux bornes de la bobine.

A l’instant t = 0, on ouvre l’interrupteur (figure de droite) et l’on observe alors un flash


lumineux dont l’origine est le phénomène d’auto-induction. Une variation de flux dans la
bobine engendre alors une tension induite dans celle-ci qui produit un courant de décharge
I d dont l’intensité est liée à ce changement brusque de régime. Le sens de ce courant
dans la bobine est le même que lorsque l’interrupteur était fermé (règle de Lenz). Dans
l’ampoule en revanche, I d est dans le sens opposé au courant I a précédent. Par conséquent,
la tension Ua change de signe et aussi d’amplitude.

97
Notre but est de quantifier l’évolution au cours du temps de la tension Ua aux bornes
de l’ampoule après l’ouverture de l’interrupteur. On pourra également caractériser les
conditions d’apparition de ce flash lumineux. En appliquant la Loi d’induction de Fara-
day lorsque t > 0 (figure de droite), il vient l’équation

dId dId R+r


R I d + r I d = −L ⇒ =− Id
dt dt L
dont la solution est

I d ( t) = I d (0+ ) e−t/T avec T = L/(R + r ) constante de temps

où I d (0+ ) est le courant juste après l’ouverture (à la limite lorsque t → 0 par valeur posi-
tive).

Il est important de remarquer qu’il n’y a pas de discontinuité pour le courant qui traverse
la bobine lors du passage : interrupteur fermé à interrupteur ouvert. En quelque sorte, la
tension induite prend le "relais" sur la tension U g du générateur à l’instant t = 0. On a
donc :
Ug
I d (0+ ) =
r
où le courant U g / r traversait la bobine lorsque l’interrupteur était fermé ( t < 0).

Pour la tension UBA ( t), lorsque t > 0, on a donc

R
UBA ( t) = R I d ( t) = U g e − t/T
r
Sachant que Ua ( t) = −UBA ( t), on a pour l’évolution temporelle de la tension aux bornes de
l’ampoule :
(
Ug si t < 0
Ua ( t) = R − t/T
− r Ug e si t > 0

On constate bien un saut brusque pour Ua , ainsi qu’un pic de tension (si R > r ) qui tend
vers 0 au cours du temps, comme l’illustre le graphique de gauche correspondant à : U g =
5 V, r = 10 Ω, R = 25 Ω et L = 1 H (T ≃ 0, 03 s).

98
La puissance dissipée dans l’ampoule P a ( t) après l’ouverture de l’interrupteur vaut (au
cours du temps)
R 2 −2 t/T
P a ( t ) = U a ( t )2 / R = U e
r2 g
Juste après l’ouverture de l’interrupteur, la puissance P a (0+ ) est donc (R / r )2 fois la puis-
sance que dissipait l’ampoule lorsque l’interrupteur était fermé (U g2 /R ). Evidemment cette
situation ne dure pas (voir graphique de droite).

Admettons que l’on règle la tension U g à la valeur nominale de l’ampoule et comparons


P a ( t) à sa puissance normale de fonctionnement (U g2 /R ). On obtient le graphique de droite.
Dans cet exemple, la puissance nominale est de 1 W. On observe que pendant environ
0, 03 s, P a est supérieure à 1 W. Cette surpuissance est responsable de l’observation du
flash lumineux. Pour être visible, il est nécessaire que la constante de temps T soit assez
grande (L grand, donc pic large) et que la résistance r de la bobine soit petite (pic haut). Un
pic de puissance même haut, mais étroit, peut très bien ne pas rendre visible ce phénomène
lumineux. Ceci en raison de l’inertie thermique du filament de l’ampoule.

21 Induction mutuelle : le transformateur

On parle d’induction mutuelle lorsque les variations de


courant dans un circuit provoquent un effet d’induction dans
un circuit voisin. Considérons l’exemple du transformateur.
Sur un noyau de fer sont enroulés deux bobines. L’une, qu’on
nomme enroulement primaire, est reliée à une source
de courant alternatif. Un champ magnétique alternatif
prend naissance dans le noyau. Ses lignes épousent la forme
du fer, de sorte qu’il est "transporté" dans l’autre bobine,
qu’on appelle enroulement secondaire. Ce champ magné-
tique variable engendre un champ électrique induit dont les
lignes sont approximativement des cercles disposés comme
les spires des enroulements.

Dans la bobine primaire, le champ électrique induit est continuellement opposé aux va-
riations de courant (par la self-induction), de sorte que le courant est beaucoup plus faible
que si on avait une source de courant continu. Dans l’enroulement secondaire, le champ

99
électrique induit détermine une tension induite d’autant plus grande que le nombre de
spires est grand. Dans le noyau, le champ électrique induit est disposé à engendrer des
courants de Foucault. Comme ceux-ci entraînent une dissipation d’énergie sous forme de
chaleur, on les empêche de naître en prenant un noyau feuilleté, c’est-à-dire formé de
tôles de fer isolées les unes des autres par du papier.

Désignons par φ le flux du champ magnétique à travers une section du noyau. Dans le
cas idéal, on suppose que le champ est complètement canalisé dans le noyau de fer. No-
tons N1 et N2 le nombre des spires du primaire et du secondaire respectivement. Les flux
d’inductions qui les traversent et les tensions induites qui y apparaissent valent

1 dφ
Φ1 = N1 φ Uind = − N1
dt

2 dφ
Φ2 = N2 φ Uind = − N2
dt
On en déduit
1
Uind N1 U 1 N1
2
= ⇒ =
Uind N2 U 2 N2

D’autre part, Uind = U (fil) + U (extérieur) où le premier terme est la tension le long du
fil. Si la résistance du fil est négligeable et si le courant est faible, on peut négliger ce
terme. Le deuxième terme est la tension aux bornes de l’enroulement (celle qu’on mesure à
l’oscilloscope par exemple). On la note simplement U . La tension induite est donc égale - en
valeur absolue - à la tension aux bornes. Ainsi, les tensions aux bornes des enroulements
d’un transformateur sont proportionnelles aux nombres de spires.

Le rapport des nombres de spires est appelé le rapport de transformation. Lorsqu’un trans-
formateur a pour fonction d’élever une tension, le secondaire contient plus de spires que
le primaire. Et inversement.

22 Oscillations propres

De nombreux systèmes physiques, lorsqu’on les écarte de leur position d’équilibre, montrent
un mouvement d’oscillations sinusoïdales de fréquence bien déterminée.

22.1 Masse suspendue à un ressort

Considérons comme premier exemple d’une telle situation, une masse m suspendue à l’ex-
trémité d’un ressort (supposé de masse négligeable) de constante k (voir figure).

100
z
I II

z̄ z ( t)
0 m

La masse est à l’équilibre dans la situation ( I ). Sa position est repérée à la cote z = 0.


La force de rappel du ressort ( k z̄) compense la pesanteur de la masse ( mg). On note par
z̄ > 0 la déformation du ressort sous l’action de la charge. Dans la situation ( I I ), on écarte
la masse de sa position d’équilibre et l’on repère sa position au cours du temps z( t) (qui
peut être positive comme négative). A chaque instant, la force que la masse subit est la
résultante de la force de rappel du ressort et du poids de m. Ce qui donne, en composante
selon Oz

F z ( t) = k ( z̄ − z( t)) − m g

En appliquant la deuxième loi de Newton et compte tenu qu’à l’équilibre ( I ) on a la relation


k z̄ = m g, on a

F z ( t) = m z̈( t) ⇒ − k z( t) = m z̈( t)

où l’on note z̈( t) la seconde dérivée de z( t) par rapport à t. De même, ż( t) désignera la
première dérivée de z par rapport à t. Ainsi, la solution z( t) que l’on cherche satisfait à
l’équation différentielle linéaire

k
z̈( t) + z ( t) = 0 (6)
m
Il existe une méthode générale pour résoudre ce genre d’équations. Nous nous contente-
rons de donner la fonction z( t), puis de vérifier qu’il s’agit bien d’une solution. Vérifions en
effet que la fonction du type suivant, pour une certaine valeur de ω, est bien une solution
de (6)

z( t) = A cos(ω t + φ) (7)

Les constantes A et φ sont quelconques a priori. En dérivant une, puis deux fois, il vient
successivement.

ż( t) = − A ω sin(ω t + φ)
z̈( t) = − A ω2 cos(ω t + φ) = −ω2 z( t)

On constate sur cette dernière relation que z( t) proposé est bien une solution de (6) pour
autant que
s
k
ω= (8)
m

101
Ainsi, la masse a un mouvement périodique sinusoïdal de pulsation définie par (8). Il s’agit
d’un mouvement harmonique.

D’autre part, fixons les conditions initiales pour ce système. On pose qu’à l’instant t = 0,
z(0) = z0 (une certaine cote fixée), et que la vitesse de la masse est nulle ż(0) = 0. Ces deux
conditions entraînent :

A cos φ = z0
− A ω sin φ = 0

De la deuxième relation on tire φ = 0, puis de la première : A = z0 . Par conséquent, la


solution est
s
k
z( t) = z0 cos(ω t) avec ω =
m

Il s’agit d’une fonction sinusoïdale du temps t de pulsation ω, dont le graphique est la


suivant

Résumé et rappel

z̈( t) + ω2 z( t) = 0 ⇔ z( t) = A cos(ω t + φ)

où les constantes A et φ dépendent des conditions initiales. Ces constantes sont détermi-
nées dès lors que la position et la vitesse de la masse sont connues au départ, c’est-à-dire à
t = 0. Un tel mouvement s’appelle un mouvement harmonique de pulsation ω. Il s’agit
d’un mouvement périodique (vu la présence de la fonction sinus). Lorsque l’argument du
sinus augmente de 2π radian, c’est-à-dire pour un certain laps de temps T plus tard, ap-
pelé la période, la fonction z reprend la même valeur :

z ( t + T ) = z ( t) pour chaque instant t

Donc le lien entre la période T et la pulsation ω est le suivant


ω T =2 π ⇒ ω=
T

La masse effectue donc un mouvement de va-et-vient, qu’on appelle aussi une oscillation,
dans un laps de temps d’une période. Autrement dit, le nombre d’oscillations que la masse

102
effectue en une seconde est l’inverse de la période. On appelle fréquence f le nombre
d’oscillations par seconde. Ainsi
1
f= et donc ω = 2π f
T
l’unité de la fréquence est ainsi s−1 qu’on note par Hz et qu’on appelle le hertz.

Remarque :
Dans chaque système physique où une grandeur qui dépend du temps satisfait l’équation
différentielle ci-dessus, pour une certaine pulsation ω, la solution est alors toujours une
sinusoïde telle que décrite. Dans la suite, la fonction z( t) pourra être un courant électrique,
une tension électrique, etc.

22.2 Circuit LC

Un condensateur de capacité C est chargé au préalable (charge Q sur la plaque + et −Q


sur l’autre). On relie ce condensateur à une bobine dont la self est L. A t = 0 on ferme
l’interrupteur. On veut déterminer le courant I ( t) en fonction du temps. On choisit le sens
de parcours ( s.p.) et on applique la loi d’induction de Faraday. Notons A la borne qui est
positive au début et B l’autre.
UBA ( t) = −L İ ( t)
La tension UBA ( t) est proportionnelle à la charge q( t) portée par la plaque + à l’instant t
q ( t)
UBA ( t) = −
C
On a donc pour la fonction q( t), sachant que le courant est I = − q̇ (attention : q est la
charge sur le condensateur !)
q ( t) 1
− = L q̈( t) ⇒ q̈( t) + q ( t) = 0
C LC
Cette dernière équation pour q( t) est la même que (6) pour z( t). On peut donc tout de suite
écrire la solution
1
q( t) = A cos(ω t + φ) avec ω = p
LC
Avec les conditions initiales : à t = 0, q(0) = Q et q̇(0) = 0, on déduit comme précédemment
que φ = 0 et A = Q . Donc,
1
q( t) = Q cos(ω t) avec ω = p (9)
LC
La charge du condensateur varie de façon périodique (elle oscille) et donc le courant change
de sens avec la pulsation ω donnée en (9). L’énergie est transférée du condensateur vers
la bobine et de la bobine vers le condensateur et ceci sans aucun amortissement si la ré-
sistance du circuit est négligeable. Un tel circuit "LC " s’appelle un oscillateur. Le dessin
ci-dessous montre des phases successives du circuit, à un intervalle d’un huitième de pé-
riode, en comparaison avec l’oscillation d’un pendule.

103
Comparons la tension U ( t) ≡ U AB ( t) et le courant I ( t)

U ( t) = q( t)/C = Q /C cos(ω t) et I ( t) = − q̇( t) = Q ω sin(ω t)

Le courant et la tension n’ont pas leurs maxima au même moment. Ils sont déphasés de
π/2.

Dans cette situation, le courant atteint sa valeur maximale un peu plus tard que la ten-
sion, après un intervalle de temps ∆ t tel que ω∆ t = π/2.

23 Oscillations forcées - circuits alternatifs

23.1 Circuit RL (série) en régime alternatif

On place une self L et une résistance R en série aux bornes d’un générateur de tension
alternative U ( t) :

U ( t) = Û sin(ω t + φ1 )

On parle d’oscillations forcées car la tension U ( t) est donnée et ne peut être modifiée. La
pulsation ω est imposée par le générateur. La phase φ1 indique simplement qu’à l’instant
choisi comme instant de départ ( t = 0), la tension U ( t) n’est pas nécessairement nulle. On
cherche à déterminer le courant I ( t) dont la forme sinusoïdale sera celle qui persistera
après une certaine période transitoire que nous ne discuterons pas. L’équation du circuit
est (la somme des tensions le long du circuit=− d Φ/ dt = −Ld I / dt, où I est positif dans le
sens du parcours choisi)

R I ( t) − Û sin(ω t + φ1 ) = −L İ ( t) ou L İ ( t) + R I ( t) = Û sin(ω t + φ1 ) (10)

104
On cherche donc une solution de la forme

I ( t) = Î sin(ω t + φ2 )

Pour qu’il en soit ainsi, le courant de crête ( Î ) et le déphasage entre la tension et le courant
(φ1 − φ2 ) doivent avoir une valeur respective particulière qui dépend de ω, L et R .

On remplace I ( t) dans (10) et l’on obtient :

R Î sin(ω t + φ2 ) + Lω Î cos(ω t + φ2 ) = Û sin(ω t + φ1 )

On utilise la relation trigonométrique (voir formulaire)


p
a sin α + b cos α = a2 + b2 sin(α + δ)
b (11)
avec tan δ = et signe(sin δ) = signe(a)
a
On a donc que la relation suivante doit être vraie à chaque instant t
p
R 2 + (ω L)2 Iˆ sin(ω t + φ2 + δ) = Û sin(ω t + φ1 )
p
Il faut nécessairement que : R 2 + (ω L)2 Î = Û et que δ = φ1 − φ2 . En résumé, le courant
a la forme suivante

circuit RL série

I ( t) = Î sin(ω t + φ2 ) avec Î = (12)
Z
p ωL
où l’impédance Z = R 2 + (ω L)2 et le déphasage φ1 − φ2 : tan(φ1 − φ2 ) =
R

Comme φ1 − φ2 est positif, le courant est en retard sur la tension. Si la résistance R


tend vers 0 alors le déphasage φ1 − φ2 tend vers π/2 (vu que tan(φ1 − φ2 ) → ∞).
φ1 −φ2
Le graphique donne un exemple ∆t = ω
= 2, 4 ms
a.c.
pour R = 0, 2 Ω, L = 5 [mH] et
f = 100 [Hz]. Le déphasage vaut U
I R
environ φ1 − φ2 = ω ∆ t ≃ 1, 5 rad
t
≃ 86◦ . L

23.2 Circuit RC (série) en régime alternatif

On place cette fois un condensateur de capacité C et une résistance R en série avec un


générateur de tension alternative

U ( t) = Û sin(ω t + φ1 )

105
L’équation du circuit est ici
q ( t)
R I ( t) + = Û sin(ω t + φ1 )
C
On dérive : I ( t) = q̇( t) (si I > 0, q croît sur le condensateur).
1
R İ ( t) + I ( t) = Û ω cos(ω t + φ1 )
C
Comme avant, on cherche le courant de crête Iˆ et le déphasage φ1 − φ2 entre la tension et
le courant pour que le courant de la forme I ( t) = Î sin(ω t + φ2 ) soit solution de l’équation
différentielle ci-dessus. D’où l’on obtient
1
R Iˆ cos(ω t + φ2 ) + Î sin(ω t + φ2 ) = Û cos(ω t + φ1 )
ωC
On utilise une relation trigonométrique du même type qu’avant
p
a cos α + b sin α = a2 + b2 cos(α − δ)
b (13)
avec tan δ = et signe(sin δ) = signe( b)
a
Le courant doit donc avoir la forme suivante :

circuit RC série
s
µ ¶2
Û 1
I ( t) = Î sin(ω t + φ2 ) avec Î = où l’impédance Z = R2 + (14)
Z ωC
1
et le déphasage φ1 − φ2 : tan(φ1 − φ2 ) = −
ωRC

Comme φ1 − φ2 est négatif, le courant est en avance sur la tension. Si R → 0 alors


φ1 − φ2 → −π/2.

Le graphique donne un exemple a.c.


pour R = 0, 2 Ω, C = 1, 4 [mF] U
et f = 100 [Hz]. Le déphasage I R
vaut φ1 − φ2 = −ω ∆ t ≃ −1, 4 rad t
≃ −80◦ . C

φ2 −φ1
∆t = ω
= 2, 2 ms

23.3 Circuit RLC (série) en régime alternatif

On place cette fois une résistance R , une self L et une capacité C en série aux bornes d’un
générateur de tension alternative U ( t)

U ( t) = Û sin(ω t + φ1 )

106
L’équation du circuit est ici
q ( t)
R I ( t) +
− Û sin(ω t + φ1 ) = −L İ ( t)
C
En dérivant et en regroupant les termes, on obtient l’équation différentielle suivante pour
le courant I ( t)
1
L Ï ( t) + R İ ( t) + I ( t) = Û ω cos(ω t + φ1 )
C
On cherche une solution du type : I ( t) = Î sin(ω t + φ2 ). On dérive, on remplace dans
l’équation et l’on trouve :
µ ¶
1 Û
− ω L sin(ω t + φ2 ) + R cos(ω t + φ2 ) = cos(ω t + φ1 )
ωC Î
En utilisant à nouveau la relation trigonométrique (13), on en déduit finalement pour le
courant I ( t)

circuit RLC série


s
µ ¶2
Û 1
I ( t) = Î sin(ω t + φ2 ) avec Î = l’impédance Z = R2 + ω L− (15)
Z ωC
ω L − 1/(ω C )
le déphasage φ1 − φ2 : tan(φ1 − φ2 ) =
R

Pic de résonance

Le courant de crête Î atteint une valeur maximale si la pulsation ω du générateur est telle
que l’impédance Z est la plus petite possible. On constate immédiatement que c’est p le cas
pour une pulsation ω0 qui est telle que ω0 L − 1/(ω0 C ) = 0, autrement dit, si ω0 = 1/ L C
qui est exactement la pulsation propre d’un circuit LC (exemple 2). On observe le pic de
résonance du courant de crête en fonction de la pulsation du générateur sur le graphique
de gauche.


a.c.

R C φ1 − φ2 [rad]
L
π/2

0 ω [rad/s]
ω0

ω0 ω [rad/s] −π/2
0

Les graphiques donnent un exemple pour R = 0, 2 Ω, L = 5 mH et C = 1, 4 mF. La pulsation


de résonance est ω0 ≃ 378 s−1 . La fréquence de résonance vaut donc f 0 = ω0 /2π ≃ 60 Hz.

Concernant le déphasage φ1 − φ2 entre la tension et le courant, il peut être positif comme


négatif,pcela dépend de la pulsation ω. Le déphasage est nul à la pulsation de résonance
ω0 = 1/ L C (puisque la tangente est nulle pour cette pulsation).

107
23.4 Puissance consommée par un appareil en régime alternatif

La puissance consommée par un appareil est égale au produit de sa tension aux bornes
par le courant qui le traverse.

Supposons que la tension aux bornes de l’appareil soit

U ( t) = Û sin(ω t + φ1 )

Le courant n’est pas nécessairement en phase avec la tension, il n’atteint pas forcément
ses extrema aux mêmes moments. Comme on l’a vu, il y a généralement un déphasage
φ1 − φ2 entre la tension U ( t) et le courant I ( t) :

I ( t) = Î sin(ω t + φ2 )

La puissance instantanée consommée par un appareil est donc

Û Î ¡ ¢
P ( t) = U ( t) I ( t) = Û Î sin(ω t + φ1 ) sin(ω t + φ2 ) = cos(φ1 − φ2 ) − cos(2 ω t + φ1 + φ2 )
2
où l’on a utilisé la relation trigonométrique suivante

1¡ ¢
sin α sin β = cos(α − β) − cos(α + β)
2

La puissance moyenne P̄ consommée sur une période est définie par


ZT
1 2π
P̄ = P ( t) dt où la période T =
T 0 ω

L’intégrale sur une période de cos(2 ω t + φ1 + φ2 ) étant nulle, on a finalement

P̄ = Ueff I eff cos(φ1 − φ2 ) (16)

Prenons l’exemple d’une bobine branchée à un généra-


teur de tension alternative (R est négligeable). Donc le
déphasage vaut φ1 − φ2 = π/2 et la puissance moyenne
P̄ est nulle. La puissance P ( t) est positive ou négative
selon que la tension et le courant sont de même signe
ou de signe contraire. Durant le deuxième quart et le
quatrième quart de période (voir le graphique), P est
positive, ce qui signifie que la bobine reçoit de l’énergie
qui est stockée sous la forme d’un champ magnétique
(en valeur absolue le courant croît). Pendant le premier
et le troisième quart de période, la puissance consom-
mée est négative. La bobine restitue l’énergie accumu-
lée aux autres moments (en valeur absolue le courant
décroît). En moyenne la puissance consommée est ainsi
nulle sur une période.

108
24 Circuits alternatifs : généralisation

24.1 Tensions aux bornes : UL , UR et UC

Lorsqu’on mesure la tension (à l’aide d’un oscilloscope par exemple) aux bornes d’une bo-
bine de constante L placée à l’intérieur d’un circuit qui peut contenir d’autres éléments
(bobines, condensateurs, résistances, etc.), on obtient une fonction qui dépend du temps,
que l’on désigne par UL ( t) et qui vaut :

UL ( t) = L İ ( t)

où İ ( t) est la dérivée par rapport au temps du courant I ( t) qui traverse la bobine à l’instant
t. En effet, considérons une bobine idéale (dont le fil est de résistance négligeable) pour
laquelle le champ magnétique créé par le courant est non nul seulement à l’intérieur de la
bobine (on néglige le champ magnétique et ses variations à l’extérieur de celle-ci).

Pour un chemin fermé qui va de A à B par le fil et revient à A par l’extérieur de la bobine,
on obtient par la loi d’induction

U AB (fil) + UBA (ext) = −L İ

Vu que U AB (fil) ≃ 0 (car la résistance du fil est négligeable), la tension U AB (ext) (qui ne
dépend pas du chemin qui joint A à B par l’extérieur de la bobine puisque le champ ma-
gnétique est supposé nul à l’extérieur de celle-ci) est justement la tension que mesure
l’oscilloscope : UL ≡ U AB (ext) = VA − VB . En l’occurrence ici cette tension s’exprime comme
la différence VA − VB des potentiels en A et en B respectivement.

De manière similaire, la tension aux bornes d’une résistance R que mesure l’oscilloscope
vaut

UR ( t) = R I ( t)

et la tension aux bornes d’un condensateur de capacité C s’exprime par

q ( t)
UC ( t) =
C
où q( t) est la charge (positive ou négative) que porte (à l’instant t) la plaque qui ≪ reçoit
le courant ≫ I ( t) lorsque ce dernier circule dans le sens de parcours ( I ( t) est alors compté
positif). Voir la figure.

109
Par exemple, dans le cas du circuit RLC en série traité plus haut, on peut appliquer la loi
des mailles de Kirchhoff pour un chemin qui passe à l’extérieur des éléments du circuit
(voir la ligne en traitillé sur la figure).

Sur ce chemin en effet aucun champ électrique induit n’est présent vu que l’on néglige les
éventuelles variations du champ magnétique à l’extérieur des éléments (situation idéali-
sée). Il vient alors

UR ( t) + UL ( t) + UC ( t) − U ( t) = 0

qui est bien la même équation du circuit que celle discutée (voir plus haut).

24.2 Représentation de Fresnel

Les résultats obtenus dans l’exemple d’un circuit RLC en série suggèrent une interpréta-
tion géométrique. Pour simplifier la notation dans la suite, on pose égale à zéro la phase
φ1 de la tension du générateur. Cela revient à choisir l’instant du départ ( t = 0) pour lequel
U (0) = 0. Le déphasage entre la tension et le courant sera désigné par φ (avant il valait
φ1 − φ2 ). Reprenons l’équation du circuit (R L C série)

UR ( t) + UL ( t) + UC ( t) = U ( t)

avec

U ( t) = Û sin(ω t)
UR ( t) = R I ( t) = ÛR sin(ω t − φ)
UL ( t) = L İ ( t) = ω L Î cos(ω t − φ) = ÛL sin(ω t − φ + π/2)
1 ˆ
UC ( t) = q( t)/C = − I cos(ω t − φ) = ÛC sin(ω t − φ − π/2)
ωC

110
où l’on a défini

ÛR = Z R Î avec Z R = R résistance pure


ÛL = Z L Î avec Z L = ω L impédance de la bobine
1
ÛC = Z C Î avec Z C = impédance du condensateur
ωC
Û = Z Î avec Z = l’ impédance du circuit (15)

Ici, toutes les tensions sont sinusoïdales, on peut se les représenter comme des vecteurs
tournant (dans le sens contraire des aiguilles d’une montre) tous à la vitesse angulaire ω
et déphasés entre eux par l’angle qui apparaît dans l’argument de sin(ω t + . . .).

Dans le dessin a), le courant I ( t) est la projection sur l’axe vertical du "vecteur-courant"
défini par sa norme Î et l’angle ω t − φ. De même pour les autres "vecteur-tension" défi-
nis par leurs normes ÛR , ÛL , ÛC et leurs angles respectifs (dessin b)). En particulier, on
constate que le "vecteur-ÛL " et le "vecteur-ÛC " sont sur la même droite (ils sont déphasés
de π entre eux). Ils sont cependant perpendiculaires au "vecteur-ÛR " (déphasés de ±π/2
selon).

De plus, comme la somme UR + UL + UC doit donner U , il en découle que la somme des


trois vecteur-tension correspondants doit être égale au "vecteur-U " (dessin d )). Cette re-
présentation géométrique, qu’on appelle la représentation de Fresnel est bien pratique
car elle est encore valable pour des circuits alternatifs quelconques comprenant des résis-
tances, selfs et capacités.

Pour obtenir l’impédance Z du circuit (15), il suffit d’appliquer le théorème de Pythagore


(voir dessin d )) à l’aide de la résistance R , de l’impédance associée à la bobine Z L et de
l’impédance associée au condensateur Z C :
q q
Z Î = (R Î )2 + (( Z L − Z C ) Î )2 ⇒ Z = R 2 + ( Z L − Z C )2

111
Pour le déphasage φ, c’est immédiat également (voir dessin d )) :

ÛL − ÛC ( Z L − Z C ) Î (ZL − ZC )


tan Φ = = =
ÛR R Î R

Remarque : en régime alternatif, les impédances Z L et Z C jouent un rôle similaire à une


résistance R du point de vue du passage du courant dans le circuit. Pour une même tension
Û et pour une grande valeur de ω, le courant passe mal dans une bobine (le courant de
crête est petit), alors qu’il passe bien dans un condensateur (le courant de crête est grand).
La situation est inversée lorsque ω est petit.

Pour simplifier la représentation de Fresnel, il suffit de se rappeler de la situation à l’ins-


tant t tel que ω t − φ = 0 (vu que les angles relatifs entre les vecteurs ne changent pas au
cours du temps). On obtient le dessin suivant (où l’on a représenté des vecteurs proportion-
nels aux impédances R , Z L et Z C puisque seulement eux interviennent dans l’expression
de l’impédance du circuit et du déphasage).

ZC
ZL Z
ZL − ZC
φ
0
ZC R

Une résistance "pure" R est représentée par un vecteur horizontal (pointant à droite), une
impédance Z L par un vecteur vertical (pointant en haut) et une impédance Z C par un
vecteur vertical (pointant en bas). Selon que Z L est plus grand ou plus petit que Z C , le
déphasage φ est positif ou négatif. Globalement, le circuit réagit plutôt comme un circuit
inductif dans le premier cas (le courant est en retard sur la tension) et comme un circuit
capacitif dans le second cas (le courant est en avance sur la tension).

24.3 Notation par les nombres complexes

Dans notre situation, le calcul à l’aide de vecteurs dans le plan est remplacé avantageu-
sement par le calcul utilisant les nombres complexes. L’axe horizontal est l’axe réel et
l’axe vertical, l’axe imaginaire. Les impédances Z L et Z C sont alors représentées par des
nombres purement imaginaires, alors qu’une résistance R est toujours représentée par un
nombre réel. En lieu et place des définitions introduites plus haut, on pose alors :

i 1
ZR = R ZL = i ω L ZC = − =
ωC iωC

où le nombre imaginaire pur i est tel que i 2 = −1.

112
L’impédance complexe Z (qui est également un nombre complexe) contient l’information à
la fois de l’impédance du circuit et du déphasage. Elle se calcule donc
s
1 1 2
Z = Z R + Z L + Z C = R + i (ω L − ) ⇒ | Z | = R 2 + (ω L − )
ωC ωC

Le module | Z | du nombre complexe Z est l’impédance introduite en (15) et l’argument de


Z (l’angle φ) est donné par

Im( Z ) ω L − 1/(ω C )
tan φ = =
Re( Z ) R

24.4 Généralisation aux autres types de circuits

Dans un circuit complexe contenant des bobines, condensateurs et résistances, on procède


de manière similaire à la méthode utilisée pour des résistances pures. On associe aux
bornes de chaque élément d’une maille, la tension correspondante : UL , UC ou UR et l’on
applique la deuxième loi de Kirchhoff à cette maille. Le chemin d’intégration passe à l’ex-
térieur des éléments (le champ magnétique et ses variations peuvent alors être considérés
comme négligeables, la somme des tensions le long de ce chemin fermé est donc nulle). De
plus la première loi de Kirchhoff s’applique : à chaque noeud du circuit, la somme des cou-
rants entrant égale à la somme des courants sortant (ou aussi, la somme algébrique des
courants est nulle à chaque noeud). Dans les circuits réductibles à des circuits plus simples
équivalents, on peut montrer que les règles qu’on applique aux circuits résistifs sont en-
core valables pour les impédances complexes. Z désigne l’impédance complexe équivalente
du circuit.
1 1 1
Z = Z1 + Z2 impédances en série = + impédances en parallèle
Z Z1 Z2

Le calcul de l’impédance d’un circuit et du déphasage entre la tension et le courant s’en


trouve considérablement simplifié.

Prenons l’exemple d’un circuit RLC en parallèle. L’impédance complexe Z vaut


µ ¶
1 1 1 1 1 1 1 1
= + + = + +i ω C= +i − +ω C
Z ZR ZL ZC R i ω L R ωL

Ainsi, l’inverse de l’impédance du circuit est


s
µ ¶2 µ ¶2
1 1 1
| |= + − +ω C
Z R ωL

et le déphasage φ se calcule en utilisant le fait que la tangente de l’argument de 1/ Z est


l’opposé de la tangente de l’argument de Z (à vérifier !). Par le résultat de 1/ Z calculé
ci-dessus, on trouve :
µ ¶
Im( Z ) Im(1/ Z ) 1/(ω L) − ω C 1
tan φ = =− = =R −ω C
Re( Z ) Re(1/ Z ) 1/R ωL

113
Procédure à suivre :
Pour des circuits mixtes (série-parallèle), on procède de la même manière que pour
un circuit résistif pur. On associe d’abord à chaque résistance, bobine et condensateur
les impédances complexes suivantes : Z R = R , Z L = i ω L et Z C = 1/( i ω C ). On calcule
ensuite l’impédance complexe équivalente du circuit ( Z ), par les mêmes règles vues
pour des circuits résistifs. Le module | Z | de Z donne le rapport Û / Î entre la tension
de crête aux bornes du générateur et le courant de crête à la sortie du générateur. La
tangente du déphasage φ entre la tension et le courant se calcule par le quotient de la
partie imaginaire de Z (Im( Z )) par la partie réelle de Z (Re( Z )).

Preuve (On peut la sauter !)


Formules utiles concernant les nombres complexes :

ei φ
= cos φ + i sin φ
1 (φ1 +φ2 )
e− i φ
= ei = ei φ1
ei φ2
ei φ
Z = | Z | e iφ où φ est l’argument de Z

Pour un circuit en parallèle :


Considérons deux impédances | Z1 | et | Z2 | placées en parallèle aux bornes d’un générateur
de tension alternative U ( t). L’impédance | Z1 | produit un déphasage φ1 entre la tension
U1 ( t) (aux bornes de | Z1 |) et le courant I 1 ( t) qui traverse cette impédance. Et similairement
pour l’impédance | Z2 |. Les déphasages φ1 et φ2 sont connus, de même que les impédances
| Z1 | et | Z2 |.

On a U ( t) = U1 ( t) = U2 ( t) et I ( t) = I 1 ( t) + I 2 ( t). On cherche à déterminer | Z | ≡ Û / Î ainsi que


le déphasage φ entre U et I . Il vient donc pour les courants

Î sin(ω t − φ) = Iˆ1 sin(ω t − φ1 ) + Iˆ2 sin(ω t − φ2 )

Comme pour la représentation de Fresnel associée aux tensions, on peut également définir
des "vecteurs-courants" dont la projection sur l’axe vertical de la somme vectorielle donne
l’égalité ci-dessus. Cette représentation est équivalente à l’écriture sous forme de nombres
complexes. On obtient
(ω t−φ) (ω t−φ1 ) (ω t−φ2 )
Î e i = Iˆ1 e i + Iˆ2 e i

D’où
Û i (ω t−φ) Uˆ1 i (ω t−φ1 ) Uˆ2 i (ω t−φ2 )
e = e + e
|Z | | Z1 | | Z2 |

Comme Û = Uˆ1 = Uˆ2 et avec les formules sur les nombres complexes ci-dessus, il vient

ei ω t
ei ω t
ei ω t
1 1 1
= + ⇒ = +
|Z | ei φ | Z1 | e i φ1 | Z2 | e i φ2 Z Z1 Z2

Pour un circuit en série :


Considérons deux impédances | Z1 | et | Z2 | placées en série aux bornes d’un générateur de

114
tension alternative U ( t). Cette fois l’on a I ( t) = I 1 ( t) = I 2 ( t) et U ( t) = U1 ( t) + U2( t). Il n’y
a ici pas de déphasage entre les courants (leur dépendance temporelle est sin(ω t)). En
revanche, les tensions n’ont pas les mêmes phases. Pour conserver la même convention de
phases que précédemment, on obtient pour les tensions

Û sin(ω t + φ) = Uˆ1 sin(ω t + φ1 ) + Uˆ2 sin(ω t + φ2 )

Sous la forme en nombres complexes :


(ω t+φ) (ω t+φ1 ) (ω t+φ2 )
Û e i = Uˆ1 e i + Uˆ2 e i

D’où
(ω t+φ) (ω t+φ1 ) (ω t+φ2 )
| Z | Iˆ e i = | Z1 | Iˆ1 e i + | Z2 | Iˆ2 e i

Comme Î = Iˆ1 = Iˆ2 il vient cette fois

|Z | e i φ
ei ω t
= | Z1 | e i φ1
ei ω t
+ | Z2 | e i φ2
ei ω t
⇒ Z = Z1 + Z2

Pour un circuit mixte :


Il suffit d’appliquer les deux règles précédentes.

25 Applications

25.1 Expérience de lévitation d’un anneau au-dessus d’une bobine


parcourue par un courant alternatif

Un anneau conducteur est placé au-dessus d’une bobine dans laquelle est introduit un
long noyau de fer doux. La bobine est parcourue par un courant alternatif. Un courant
induit prend donc naissance dans l’anneau. Ce dernier subit alors une force de Laplace
due à la composante horizontale du champ magnétique à l’endroit où se trouve l’anneau.
On observe que l’anneau reste en lévitation autour du noyau, voire est expulsé.

La figure montre le dispositif à l’exception du noyau de fer. On souhaite, en effet, com-


prendre la condition nécessaire à cette lévitation sans tenir compte des effets, certaine-
ment importants, qu’apportent les particularités des lignes de champ dues à la présence
du noyau.

Le courant dans la bobine est de la forme

I bob ( t) = Î bob sin(ω t)

Le champ magnétique produit par la bobine varie au cours du temps comme le courant
I bob ( t). La tension induite dans l’anneau Uind ( t), se calcule par la loi d’induction. On trouve

Uind ( t) = −Ûind sin(ω t + π/2)

115
ligne de champ

anneau

a.c.

Cette tension induite va créer un courant induit dans l’anneau I ann ( t). Or, l’anneau a une
faible constante d’auto-induction L de même qu’une faible résistance R . Il constitue un
circuit RL, dont le courant est en retard sur la tension. Le déphasage φ entre Uind ( t) et
I ann ( t) est donné par

ωL
tan φ = et I ann ( t) = − Iˆann sin(ω t + π/2 − φ)
R
Si la résistance R était négligeable devant ωL, le déphasage serait φ = π/2. Ainsi le courant
dans l’anneau serait en opposition de phase par rapport au courant dans la bobine. Sur
le graphique, les instants b et c coïncideraient, de même que d et e. Donc, la force de
Laplace (qui est proportionnelle à I ann ( t) et à la composante horizontale de B~ ( t)) serait
dirigée vers le haut à chaque instant (force de répulsion sur l’anneau), ce qui permettrait
de comprendre une éventuelle lévitation de l’anneau, voire une éjection de ce dernier.

Dans une situation plus réaliste, ces deux courants ne sont pas en opposition de phase et
il faut admettre que l’intervalle de temps bc n’est pas nul, de même pour l’intervalle d e.
Pendant ces deux intervalles de temps, la force de Laplace est dirigée vers le bas (force
d’attraction sur l’anneau en direction de la bobine), alors qu’elle est dirigée vers le haut
durant les intervalles ab et cd . Dans la mesure où R < ωL, le déphasage peut très bien
être légèrement inférieur à π/2, de sorte que, sur une période, la durée où la force de
Laplace est répulsive (ab + cd ) est plus longue que la durée où elle est attractive ( bc + d e).

116
Dans ce cas, en moyenne sur une période, la répulsion l’emporte sur l’attraction en ce qui
concerne la force de Laplace. Cette dernière peut alors éventuellement créer les conditions
nécessaires à une lévitation de l’anneau.

Dans l’expérience réalisée en classe, l’anneau est fait d’aluminium de r 1 = 4, 5 cm (rayon


moyen) et de r 2 = 3, 4 mm (rayon de sa section), le coefficient L est très petit, il vaut
L = 1, 65 · 10−7 H (pour un anneau, L se calcule par la formule : L = µ0 r 1 (log(8 r 1 / r 2 ) − 7/4)).
D’autre part, la résistance R de l’anneau est également très petite. Elle vaut R = 2, 12 ·
10−4 Ω. Le déphasage φ vaut ainsi : φ = tan−1 (ωL/R ) = 15◦. Le déphasage entre I ann et I bob
est de 90◦ + 15◦ = 105◦ et non de 180◦ comme dans la situation idéale d’une opposition de
phase. Cela signifie que pendant environ 60% du temps, la force de Laplace est répulsive.
Elle est attractive pendant les 40% du temps restant.

La situation est bien différente lorsqu’un noyau de fer doux est inséré dans la bobine et que
l’anneau est placé autour du noyau. Le champ magnétique est renforcé, dû à l’orientation
des domaines magnétiques, et peut facilement être µr = 1000 fois supérieur (c’est un ordre
de grandeur) à celui créé par la bobine seule. Le nombre µr est appelé perméabilité relative
du fer doux. Le coefficient L de l’anneau devient alors beaucoup plus grand aussi. Pour un
solénoïde, le coefficient de self-induction serait µr fois supérieur à celui que l’on aurait en
l’absence du noyau. Ce n’est pas aussi simple que cela pour notre anneau. Même si L est,
disons, 10 fois plus grand en présence du noyau, la tangente du déphasage est 10 fois plus
grande aussi. Le déphasage devient φ ≃ 70◦ , de sorte que le courant dans l’anneau et le
courant dans la bobine sont, cette fois, déphasés de 160◦, c’est-à-dire presque en opposition
de phase. En présence d’un noyau de fer, il devient raisonnable de considérer que ces deux
courants sont en opposition de phase et que, de ce fait, la force de Laplace est tout le temps
répulsive.

25.2 Transport de l’électricité et moteur (a)synchrone

Transport, lignes à hautes tensions, courants triphasés et réseau domestique :

La distribution de l’énergie électrique se fait par un réseau qu’on appelle triphasé. Un


générateur à courant triphasé ou alternateur triphasé est construit sur un principe
analogue à celui vu plus haut dans le cours. On fait tourner un électroaimant (ou aussi un
barreau aimanté) dans une cavité où sont disposées trois bobines dont les axes se coupent
suivant des angles de 120◦.

Alternateur
Ab
câble de garde (paratonnerre)
A
B
N
b
B b
C
S

C b

117
Dans l’alternateur, une borne de chaque bobine est reliée à un point commun appelé terre
(c’est un conducteur ancré physiquement en terre). Les tensions induites aux bornes des
bobines sont sinusoïdales. En effet, la variation de flux d’induction dans les bobines y
produit une tension induite. Par des pièces de fer dans le rotor (la partie de l’alternateur
qui tourne) et dans le stator (la partie fixe), on donne au champ d’induction la forme qu’il
faut pour que les tensions induites soient sinusoïdales.

Vu la disposition ≪ en étoile ≫ de ces bobines, il existe un déphasage de 120◦ (2π/3 rad)


entre l’une et une autre de ces tensions. Désignons ces tensions (appelées aussi phases)
par U A ( t), UB ( t) et UC ( t). Les bobines de l’alternateur étant identiques, ces tensions ont
même amplitude maximale Û . Au lieu de A , B et C , on désigne aussi les 3 bornes de sortie
de l’alternateur par R , S et T dans la littérature. On a

U A ( t) = Û sin(ω t), UB ( t) = Û sin(ω t + 2π/3), UC ( t) = Û sin(ω t − 2π/3)

La fréquence f (ω = 2π f ) de rotation du rotor (l’électroaimant qui tourne) est fixée, en


Europe, à 50 Hz (60 Hz aux USA). Au démarrage, l’électroaimant est alimenté par une
source auxiliaire de courant continu. Ensuite, il utilise une petite partie du courant (qui
est alors redressé) que l’alternateur produit.

U UA UC UB

0 t

A la sortie de l’usine de production (centrale hydroélectrique par exemple), la puissance


électrique est transportée sous haute tension (plusieurs centaines de milliers de volts) et
donc faible courant, afin de réduire les pertes par effet joules dans les câbles. Un trans-
formateur élève la tension en début de ligne (la centrale) et un autre l’abaisse en fin de
ligne (le consommateur), voir plus bas. La tension efficace de chaque phase à l’entrée des
habitations est fixée à 230 V en Europe (110 V aux USA).

Généralement, les lignes à haute tension sont constituées par des groupes de 3 câbles (un
câble par phase) appelés des ternes. Plusieurs ternes peuvent être attachés par pylône (2
ternes sur le schéma de la ligne ci-dessus). Au sommet du pylône, un câble jouant le rôle
de paratonnerre (câble de garde) est relié à la terre via le pylône lui-même.

On remarque assez fréquemment sur les lignes à haute tension la présence d’une paire
de câbles par phase (au lieu d’un seul câble). Il s’agit d’un dispositif destiné à limiter les
pertes d’énergie électrique dues à l’effet couronne (effet corona). Un champ électrique très
intense au voisinage d’un conducteur produit une ionisation de l’air dans son voisinage
immédiat ce qui entraîne une perte d’énergie. A la surface d’un câble placé à haute tension
le champ électrique est d’autant plus intense que son diamètre est petit (on a vu cela
dans le cadre de l’électrostatique, mais c’est aussi vrai ici). Il suffirait donc d’augmenter
le diamètre des câbles. Ces derniers deviendraient alors trop lourds et l’installation serait

118
trop coûteuse. On remédie à cela en reliant, par des traverses conductrices, deux câbles
(de petits diamètres) proches l’un de l’autre (environ 30 cm). Les deux câbles sont alors au
même potentiel. Le champ électrique dans le voisinage des deux câbles ressemble à celui
d’un câble unique mais de plus gros diamètre. On diminue de la sorte les pertes par effet
couronne (c’est la cause du crépitement que l’on entend parfois à proximité de certaines
lignes).

On peut calculer
p que la tension entre deux phases est sinusoïdale et que son amplitude
maximale est 3 fois plus élevée que l’amplitude maximale d’une seule phase :
p
Û AB = 3 Û

En effet, à l’aide de la somme de fonctions trigonométriques en produit (voir formulaire) :

sin α − sin β = 2 sin((α − β)/2) cos((α + β)/2)

il vient

U AB ( t) = U A ( t) − UB ( t) = Û sin(ω t + ) − Û sin(ω t)
3
µ ¶ µ ¶
ω t + 2π/3 − ω t ω t + 2π/3 + ω t
= 2 Û sin cos = 2 sin(π/3) Û cos(ω t + π/3)
2 2
p
= 3 Û cos(ω t + π/3)

Laptension U AB ( t) est manifestement sinusoïdale et son amplitude maximale est bien égale
à 3 Û .
p
Le même rapport de 3 se retrouve évidemment entre les tensions efficaces. Par exemple,
si la tension efficace
pentre une phase et la terre est de 230 V, la tension efficace entre deux
phases est de 230 · 3 = 398 V, soit presque 400 V.

Un appareil qui nécessite une grande puissance électrique pour fonctionner est branché
sur les 3 phases du réseau (une cuisinière électrique par exemple). On parle de courant
triphasé pour le courant qui l’alimente. D’autres appareils (les lampes par exemple) sont
branchées sur une seule phase (courant monophasé).

Le schéma ci-dessous donne le principe de distribution électrique dans les habitations.


La partie de gauche montre un transformateur qui permet de diminuer la tension à la
valeur requise (les 230 V par phase en valeur efficace en Europe). Le primaire est fait de
3 bobines montées en triangle (nombre de spires élevé). Le secondaire, lui, est fait de 3
bobines montées en étoile (nombre de spires faible). La borne commune de ces 3 bobines
est reliée à un câble qu’on appelle le neutre. C’est le câble de ≪ retour du courant ≫ . Ainsi,
4 câbles sont présents à l’entrée des habitations.

119
Haute tension Transformateur Basse tension
primaire secondaire A b b b

A b

398 V
b
B b b b

230 V 398 V
B b 398 V
C b b
230 V b

C b
neutre mis à terre N b b b
230 V
b b

branchements
monophasés
(lampes)

branchement terre (maison)


triphasé
(cuisinière)

Dans le transformateur, le câble du neutre est mis à terre. Si les impédances (des appareils
branchés) sont les mêmes sur chaque phase, alors le courant est nul dans le câble du
neutre (car la somme de 3 sinusoïdes identiques déphasées de 2π/3 donne zéro). Dans la
pratique, ce courant est généralement assez faible.

La partie de droite du schéma ci-dessus, montre les branchements possibles à l’intérieur


des habitations. Les lampes sont branchées entre une phase et le neutre, tandis qu’une
cuisinière utilise généralement les 3 phases. Pour des raisons de sécurité, les chassis des
appareils (parties extérieures des appareils) sont reliés entre eux et mis à terre au moyen
d’un câble supplémentaire appelé câble de terre. En effet si, par suite d’un incident, un
câble de phase entre en contact avec le chassis d’un appareil qui n’est pas mis à terre et
qu’une personne touche ce chassis, elle se fera électrocuter. Ce câble de terre (le cinquième)
est ancré dans le sol (mis à terre). La figure ci-dessous montre les prises murales que l’on
a en Suisse.

prise murale (3 fiches) prise murale (5 fiches)

neutre phase ( A , B ou C ) phase B phase A

terre phase C terre


neutre

Pour améliorer la sécurité, on installe souvent un disjoncteur différentiel (sur chaque


phase) à l’entrée du bâtiment. Normalement, le courant qui circule dans le fil de phase
doit être le même que celui qui circule dans le fil neutre (retour du courant). Si ce n’est pas
le cas, c’est qu’un courant de fuite a passé ailleurs (par le fil de terre ou par une personne,
etc.). Une légère différence entre ces courants (de l’ordre de 5 mA) et le circuit s’ouvre.
Le temps de réaction du disjoncteur est de 25 ms. Le principe de fonctionnement est le
suivant. Un anneau fait d’un matériau magnétique (fer doux) et sur lequel est enroulé un
fil conducteur (bobine), entoure le fil de phase et le fil neutre. En temps normal, aucune
variation de flux n’est détectable et donc la tension induite aux bornes de la bobine est
nulle. Si les courants (de phase et du neutre) devaient être différents, une variation de
flux prendrait naissance dans l’anneau et une tension induite existerait aux bornes de la
bobine, trahissant ainsi la présence d’un courant de fuite. Cette tension induite actionne
le disjoncteur.

120
Remarque : Un courant de 30 mA qui traverse le corps humain est mortel.

Moteur synchrone et asynchrone :

Si l’on alimente avec des courants triphasés trois bobines dont les axes sont dans un même
plan et font entre eux des angles de 120◦, les champs d’induction se combinent et il en
résulte un champ tournant au point P . Le vecteur B ~ ( t) qui le représente est situé dans
le plan des axes. Il a une grandeur constante et tourne d’un mouvement régulier. Il fait un
tour pendant une période du courant.

Du point de vue des connexions, la bobine 1 est branchée aux bornes B et C , la bobine 2
aux bornes C et A et la bobine 3 aux bornes A et B. De plus, une bobine est connectée aux
deux autres (circuit en triangle).

Le champ B ~ 1 ( t) produit par la bobine 1 en P est dans l’axe de la bobine 1. De même pour
les champs B ~ 2 ( t) et B
~ 3 ( t) produits par les bobines 2 et 3. La somme vectorielle B
~ ( t) ≡
~ 1 ( t) + B
B ~ 2 ( t) + B~ 3( t) est constante en norme, mais sa direction tourne à la même fréquence
que la fréquence du courant.

b b
A
B b b

C b b
b
b

P
b b b

b
1
~ ( t)
B
3
b b

Dans chaque bobine, le courant est sinusoïdal de fréquence angulaire ω. Il en est donc
de même pour le champ B ~ 1 ( t) et également pour les autres champs. Les courants étant
déphasés de 2π/3 entre eux, leurs champs respectifs le sont aussi.

B ~ˆ 1 cos(ω t)
~ 1 ( t) = B ~ˆ 2 cos(ω t + 2π/3)
~ 2 ( t) = B
B ~ˆ 3 cos(ω t − 2π/3)
~ 3 ( t) = B
B

Pour voir que le champ B ~ est tournant, il est très pratique de travailler avec les nombres
complexes. Désignons par θ = 2π/3 rad l’angle que forme les bobines entre elles et par z1
le nombre complexe qui représente le vecteur (d’intensité maximale) du champ de la bo-
bine 1, c’est-à-dire B ~ˆ 1 . De manière similaire, z2 = e iθ z1 représente B ~ˆ 2 et z3 = e iθ z2 = e i2θ z1
représente B ~ˆ 3 (lorsqu’on multiplie un nombre complexe par e iθ , ce nombre subit une rota-
tion d’un angle θ dans le plan complexe). Ainsi, le nombre complexe w1 ( t) = z1 cos(ω t) re-
présente B ~ 1 ( t) et similairement pour w2 ( t) = z2 cos(ω t + θ ) et w3 ( t) = z3 cos(ω t − θ ). Sachant
que cos α = 21 ( e iα + e− iα ), on peut calculer la somme w1 ( t) + w2 ( t) + w3( t) qui représente B ~ ( t).

121
On obtient
z1 ³ i ω t − iω t i(ω t+2θ) − iω t i(ω t+θ) − i ω t i 3θ
´
w1 ( t) + w2 ( t) + w3 ( t) = e +e +e +e +e +e e
2
z1 ³ i ω t ´ 3z
1 − iω t
= e (1 + e iθ + e i2θ ) + 3 e− iωt = e
2 2

où l’on a utilisé le fait que e i3θ = 1 et 1 + e iθ + e i2θ = 0, puisque θ = 2π/3. Ce résultat montre
que le champ B ~ ( t) tourne à la vitesse angulaire ω et que son intensité est constante. Elle
vaut 1, 5 fois l’intensité maximale du champ en P créé par une des 3 bobines.

Cette propriété des courants triphasés de pouvoir créer des champs tournants est mise à
profit pour la construction de moteurs. Dans le moteur synchrone, le rotor est constitué
par un aimant ou un électroaimant qui est entraîné par le champ d’induction. La fréquence
de rotation est identique à la fréquence du courant. Ce type de moteur peut être utilisé
pour la construction d’horloges électriques, pour autant que le réseau ait une fréquence
bien stable. Si la charge est trop grande, le moteur ≪ décroche ≫ et s’arrête.

Dans le moteur asynchrone triphasé, le rotor est constitué par une cage d’écureuil
en gros fil de cuivre (les barreaux de la cage), les deux anneaux (figure) sont conducteurs
également.

I induit

~
F
~
B ~vcage
~
vchamp

barreau de la cage

La flèche sur la figure indique la direction du champ B~ à un certain instant. Cette cage
est balayée par le champ magnétique tournant. Les conducteurs sont alors traversés par
des courants de Foucault induits. Des courants circulent dans les ≪ boucles ≫ formées par
la cage, les forces de Laplace qui en résultent exercent un couple sur le rotor. D’après la
loi de Lenz les courants induits s’opposent par leurs effets à la cause qui leur a donné
naissance. Le rotor tourne alors dans le même sens que le champ mais avec une vitesse
légèrement inférieure à la vitesse de ce dernier.

Le rotor ne peut pas tourner à la même vitesse que le champ magnétique, sinon la cage ne
serait plus balayée par le champ tournant et il y aurait disparition des courants induits
et donc des forces de Laplace et du couple moteur. Les deux fréquences de rotation ne
peuvent donc pas être synchrones d’où le nom de moteur asynchrone.

Prenons l’exemple d’un moteur dont la fréquence de rotation nominale relevée sur la
plaque signalétique est de 2′ 840 tr/min, ce moteur étant alimenté en courant de fréquence
50 Hz. La fréquence de rotation du champ magnétique est donc de 50 tr/s soit 3′000 tr/min.
Le rotor est donc balayé par un champ magnétique qui tourne à un fréquence de rotation
relative de 3′ 000 − 2′840 = 160 tr/min.

122
Le moteur asynchrone triphasé est utilisé dans des dispositifs nécessitant de forte puis-
sance mécanique. L’avantage d’un tel moteur est qu’il démarre sans problème et que sa
conception est simple.

Dans notre usage quotidien, les moteurs électriques qui font fonctionner les appareils tels
que : lave linge, sèche-cheveux, etc..., sont branchés sur du courant monophasé et sont
moins puissants que les précédents. On parle alors de moteur asynchrone monophasé.

b
A b
N
b

Ne démarre pas tout seul a1


b
b

b
Ab
N b

b 2 b b 1 b

b
b 2 b b 1 b a2

cage d’écureuil

b
Le moteur de gauche ci-dessus ne démarre pas tout seul si la cage est au repos au départ
(par contre, si la cage est déjà en mouvement, une étude plus détaillée montre que le
couple moteur entraîne la cage dans son sens de rotation). On constate en effet que le
moment des forces de Laplace est nul si la cage est au repos. Pour remédier à ce problème,
on branche alors un groupement de 2 bobines auxiliaires a 1 et a 2 perpendiculairement
aux deux autres. Les champs magnétiques créés par ces deux paires de bobines sont alors
perpendiculaires. Le problème est qu’ils ne sont pas déphasés (comme dans le moteur
asynchrone triphasé) et ne produisent pas de champ tournant susceptible de mettre la
cage en rotation. Il faudrait que le courant des bobines auxiliaires soient déphasés de π/2
par rapport au courant des bobines principales (1 et 2). En effet, de manière semblable
au calcul précédent, sachant que : θ = π/2, z2 = iz1 ( e iθ = i ), w1 ( t) = z1 cos(ω t) et w2 ( t) =
z2 cos(ω t + θ ) = i z1 cos(ω t + θ ), on a

w1 ( t) + w2 ( t) = z1 (cos(ω t) + i cos(ω t + θ )) = z1 e− iωt

On obtient bien un champ tournant à la vitesse angulaire ω et dont la norme est constante.
Ce calcul est basé sur l’identité des 2 paires de bobines. Dans le cas où les bobines ne sont
pas les mêmes, le champ tourne, mais pas très rond....

Pour produire un déphasage entre les deux champs, on introduit un condensateur dans
le circuit auxiliaire. A l’aide de ce condensateur et des bobines, on s’arrange à obtenir
un déphasage d’environ π/2 entre les deux courants, donc aussi entre les deux champs.
Une fois la cage mise en rotation, un interrupteur coupe le circuit auxiliaire (de façon à
préserver la durée de vie du condensateur) et la cage continue de tourner sous l’effet du
couple moteur qui n’est plus nul.

123
25.3 Filtre passe-haut, filtre passe-bas

L’impédance d’un condensateur (1/(ω C )) diminue lorsque la fréquence du signal augmente.


Ainsi, en plaçant un condensateur en série avec le haut-parleur, cela réduit les basses
fréquences et laisse passer les hautes fréquences (filtre "passe-haut").

Pour une self, l’impédance (ω L) augmente avec la fréquence. Cela permet de laisser passer
les signaux de basses fréquences et de réduire ceux de hautes fréquences (filtre "passe-
bas").

25.4 Récepteur radio à modulation d’amplitude

Le circuit de base d’un récepteur radio en modulation d’amplitudes ( AM ) est donné ci-
dessous. Les électrons libres de l’antenne (couplée à la bobine L 1 ) se mettent en mouve-
ment lors du passage des ondes électromagnétiques de la station émettrice et engendrent
une tension induite variable aux bornes de L 1 . L’ensemble des deux bobines L 1 et L 2
constitue un transformateur. Un courant variable prend donc naissance dans le circuit
L 2 , C 1 qui forme un oscillateur.

On ajuste la capacité C 1 pour que l’oscillateur soit en résonance à la fréquence de l’onde


porteuse (entre 535 kHz et p1605 kHz) de l’émission que l’on souhaite recevoir (le "tuner").
Seule cette fréquence (2π L 2 C 1 )−1 ainsi que les fréquences voisines produisent un cou-
rant notable capable d’être transmis au circuit suivant, celui de la diode qui laisse passer
le courant que dans un seul sens. Le rôle du troisième circuit (C 2 , R ) est un filtre. Quand la
diode annule le courant, le condensateur se décharge dans R . La constante de temps de ce
circuit (RC 2 ) est longue comparée à la période de l’onde porteuse ; la décharge est lente et
la tension aux bornes de R ne diminue que légèrement avant que la diode ne laisse passer
le courant à nouveau : C 2 se recharge. Le résultat est une tension légèrement modulée
(de basse fréquence comparée à celle de la porteuse) aux bornes de R qui correspond ap-
proximativement à l’enveloppe de l’onde porteuse. Cette enveloppe est l’information, sous
la forme d’un signal électrique d’amplitude variable, qui correspond à l’onde sonore origi-
nelle du studio de la station émettrice. A ce stade, la forme de l’onde est bien celle que l’on
souhaite envoyer aux écouteurs (ou aux colonnes), à l’exception qu’elle est tout le temps

124
positive, comme si elle était décalée par une tension positive constante. Cette composante
continue (tension constante) est éliminée grâce au condensateur C 3 qui la bloque (1/(ωC )
est infinie pour ω = 0 qui correspond à une tension constante). Finalement, la tension
résultante (aux bornes des écouteurs) a des oscillations lentes qui ont presque la même
allure que celles de l’onde sonore émise par la personne parlant dans le micro du studio de
la radio.

26 Equations de Maxwell

Les phénomènes électriques et magnétiques sont réunis, dans leur description, par un
ensemble d’équations (équations de Maxwell ∼ 1870) qui en fournissent une synthèse re-
marquable.

Il faut distinguer deux paires de champs. Les sources, que sont les charges et les courants,
produisent dans leurs voisinages deux champs. L’un est noté D ~ et est appelé le champ
de déplacement électrique. L’autre, noté H ~ , est le champ magnétique. Le milieu ma-
tériel et aussi le vide situé dans le voisinage de ces sources réagit et exerce à son tour
deux champs de forces qui vont influencer les charges et les courants. Il s’agit du champ
électrique E ~ et du champ d’induction magnétique B ~.

~ et B
Dans le vide, les champs de force E ~ sont reliés aux champs créés par les sources D
~ et
~ par les relations suivantes (relations phénoménologiques).
H

~ = ǫ0 E
D ~ et ~ = µ0 H
B ~ relations phénoménologiques dans le vide

où ǫ0 et µ0 sont la permittivité du vide et la constante d’induction respectivement dont les

125
valeurs numériques sont connues (ǫ0 = 8, 85 · 10−12 [A s V−1 m−1 ] et µ0 = 4π · 10−7 [V s A−1
m−1 ]).

~ et B
Dans le cours, nous n’avons parlé que des champs de forces E ~ pour simplifier la
discussion.

Les équations de Maxwell proprement dites sont des relations structurelles entre les
champs et les sources. Elles sont valables dans tous les milieux. Elles sont numérotées
de (I) à (IV) ci-dessous.
Z
D ~ = Q enf
~ · dS (I)
S

Dans cette équation S est une surface fermée quelconque et Q enf est la charge contenue
à l’intérieur de S (charge enfermée). On reconnaît la loi de Gauss vue en électrostatique.
Notons qu’ici cette relation est encore valable si les grandeurs varient au cours du temps.
La loi de Gauss pour le champ d’induction magnétique s’écrit
Z
B ~=0
~ · dS (II)
S

où S est une surface fermée quelconque. La loi d’induction de Faraday est

d
Z Z
~
E · d~r = − B ~
~ · dS (III)
Γ dt Σ

où Γ est un chemin fermé quelconque et Σ une surface qui s’appuie sur Γ (Γ est le bord de
la surface Σ). La loi d’Ampère est modifiée par un terme dû à Maxwell (le deuxième terme
du membre de droite de (IV)) qui généralise la loi d’Ampère dans le cas où les courants
dépendent du temps. Cette loi porte le nom de loi d’Ampère-Maxwell et est donnée par

d
Z Z
~ · d~r = I enlacé +
H D ~
~ · dS (IV)
Γ dt Σ

Ici également Γ désigne un chemin fermé quelconque et Σ une surface qui s’appuie sur Γ.
La grandeur I enlacé désigne la somme algébrique des valeurs (positives ou négatives) que
prennent les courants au niveau de la surface Σ. Il s’agit donc uniquement des courants
enlacés par le chemin Γ. Les autres courants n’interviennent pas explicitement dans cette
expression.

Les équations (I) à (IV), ainsi que les relations phénoménologiques, constituent la base de
l’électromagnétisme classique. Une conséquence très importante de ces équations (jointes
aux relations phénoménologiques dans le vide) est qu’elles prédisent l’existence d’ondes
électromagnétiques, dont la lumière fait partie. La théorie de Maxwell permet donc de dé-
crire aussi les phénomènes ondulatoires de la lumière, dont l’optique géométrique consti-
tue un cas limite. On peut en effet montrer que les composantes des champs dans le vide
~ dont les composantes
satisfont à l’équation suivante (cas ici pour le champ électrique E
sont E 1 , E 2 et E 3 )

∂2 E i (~
µ 2
r, t) ∂2 E i (~r, t) ∂2 E i (~r, t)
∂ E i (~

r, t) 1
= + + i = 1, 2, 3
∂ t2 ǫ0 µ0 ∂ x2 ∂ y2 ∂ z2

126
Une telle équation s’appelle une équation d’onde (voir le cours sur les ondes), dans laquelle
la vitesse de propagation est donnée par
s
1
v= ≃ 3 · 108 m/s (vitesse de la lumière dans le vide)
ǫ0 µ0

De telles ondes électromagnétiques ont été mises en évidence pour la première fois par
Hertz vers la fin du XIX ème siècle en parfait accord avec la théorie de Maxwell.

27 Annexe I : Les nombres complexes

On sait que l’équation x2 + 1 = 0 n’admet pas de solution dans l’ensemble des nombres
réels IR. On souhaite construire un ensemble de nombres qui contienne IR comme sous-
ensemble et dans lequel l’équation précédente possède des solutions. Plus précisément,
cet ensemble doit satisfaire aux mêmes propriétés algébriques que IR, en d’autres termes :
être un corps commutatif. Ce terme regroupe les propriétés suivantes bien connues : •
associativité et commutativité des opérations + et · respectivement • existence de l’élément
0 (neutre pour +) et de l’élément 1 (neutre pour ·) • existence de l’opposé d’un élément
relativement à + • existence de l’inverse d’un élément (excepté pour 0) relativement à · •
distributivité de · sur +. Un tel ensemble existe (on va le construire) et il s’appelle le corps
des nombres complexes que l’on note par C.

Tout nombre complexe peut être représenté par un couple de nombres (a, b), avec a et b
appartenant à IR. Le nombre a est la partie réelle du nombre complexe et b est sa partie
imaginaire.

Deux nombres complexes (a, b) et (a′ , b′) sont dits égaux si

a = a′ et b = b′

On écrit alors

(a, b) = (a′ , b′ )

La somme et le produit de deux nombres complexes (a, b) et (a′ , b′) sont définis par les
règles suivantes

(a, b) + (a′, b′ ) = ( a + a′ , b + b ′ )
(a, b) · (a′ , b′) = (aa′ − bb′ , ab′ + a′ b)

On vérifie aisément que (0, 0) est l’élément neutre de + et que (1, 0) est l’élément neutre de
·. De plus, on constate que :

(−a, − b) est l’opposé de (a, b)


a −b
( , ) est l’inverse de (a, b) si a 6= 0 et b 6= 0
a2 + b 2 a2 + b 2

127
Le sous-ensemble de C constitué des éléments de la forme (a, 0) avec a ∈ IR peut être
identifié aux nombres réels IR. En effet, on constate en particulier que (a, 0) + (a′ , 0) = (a +
a′ , 0) et (a, 0) · (a′ , 0) = (a a′ , 0). L’inverse et l’opposé d’un nombre complexe de la forme (a, 0)
est encore un nombre complexe de cette forme. On a donc IR ⊂ C. Pour alléger l’écriture on
adoptera, pour cette raison, la notation suivante :

a = (a, 0)
b = ( b, 0)

On convient de noter par le symbole i l’élément particulier (0, 1) de C (qui n’est pas dans
IR).

i = (0, 1)

Donc

a + i b = (a, 0) + (0, 1) · ( b, 0)

ce qui donne, par les règles de la multiplication et de l’addition

a + i b = (a, b)

Ceci nous permet de renoncer désormais à la notation (a, b), et de représenter par a + i b
le nombre complexe de partie réelle a et de partie imaginaire b. D’autre part, pour profiter
pleinement des possibilités du calcul avec les nombres complexes, nous les désignerons le
plus souvent par une seule lettre (généralement z). On écrira par exemple

z = a+ i b

a et b (réels tous les deux) étant les parties réelles et imaginaires de z. On écrira

a = Re( z) b = Im( z)

Calcul de i 2 : On a

i 2 = (0, 1) · (0, 1) = (−1, 0) = −1

On peut donc en déduire ceci : Relativement aux opérations (+, ·), les nombres com-
plexes peuvent être traités comme les nombres réels, avec la seule règle supplé-
mentaire selon laquelle on remplace le carré de i par −1.

Par exemple, pour z = a + i b et z′ = a′ + i b′ , on a

z · z ′ = ( a + i b ) · ( a′ + i b ′ ) = a a′ + i 2 b b ′ + i a b ′ + i b a′ = a a′ − b b ′ + i ( a b ′ + a′ b )

Deux nombres complexes sont dits conjugués lorsqu’ils ont même partie réelle et des
parties imaginaires opposées ; ainsi

2 + 3i et 2 − 3i

128
sont conjugués. On note généralement par z̄ le conjugué de z.

z = a + ib ⇔ z̄ = a − ib

On voit immédiatement que

z + z̄ = 2a = 2 Re( z) et z z̄ = a2 + b2

La somme et le produit de deux nombres complexes conjugués sont donc réels. Pour deux
nombres complexes z1 et z2 on a aussi

z1 + z2 = z̄1 + z̄2 et z1 · z2 = z̄1 · z̄2

Représentation graphique : De même qu’il existe une correspondance biunivoque entre les
nombres réels et les points d’un axe, il est possible d’associer à tout nombre complexe,
d’une manière unique, un point du plan : ayant choisi un repère cartésien Ox y, on fait
correspondre au nombre a + ib le point de coordonnée (a, b) ; si l’on désigne par z le nombre
a + ib, on parlera même du point z correspondant (on dit aussi le point d’affixe z). Dans
une telle représentation, l’axe des abscisses (l’axe horizontal Ox) est dit axe réel, celui
des ordonnées (l’axe vertical O y) axe imaginaire. On parle également de plan complexe
pour l’ensemble C.

On appelle module de z le nombre (réel) noté | z|, qui représente la norme du vecteur (a, b)
dans cette représentation géométrique.
p
| z | = a2 + b 2 on a donc | z|2 = z · z̄

On appelle argument de z l’angle, défini à k2π près, formé avec l’axe réel par le vecteur
reliant l’origine au point z ; on le note arg( z).

Soient

r = | z| et θ = arg( z)

On a alors

a = r cos θ , b = r sin θ

donc

z = a + i b = r (cos θ + i sin θ )

129
c’est la forme trigonométrique du nombre complexe z.

Interprétation géométrique des opérations : Considérons deux nombres complexes z et z′


et les points correspondants dans la représentation graphique. On voit immédiatement
que le point z + z′ s’obtient en faisant la somme des vecteurs correspondant à z et z′ (c’est
la règle du parallélogramme en géométrie vectorielle).

L’interprétation géométrique de la multiplication se fait facilement si on utilise la forme


trigonométrique. Soient

z = r (cos θ + i sin θ ) et z′ = r ′ (cos θ ′ + i sin θ ′ )

On a
z z′ = rr ′ (cos θ + i sin θ )(cos θ ′ + i sin θ ′ )
= rr ′ cos θ cos θ ′ − sin θ sin θ ′ + i (sin θ cos θ ′ + sin θ ′ cos θ )
¡ ¢

= rr ′ (cos(θ + θ ′ ) + i sin(θ + θ ′ ))

d’où

| z · z′ | = | z| | z′ | et arg( z z′ ) = arg( z) + arg( z′ )

le module du produit est le produit des modules, l’argument du produit est la


somme des arguments.

On voit en particulier que si l’on multiplie le nombre z par un nombre de module un, le
module du produit est égal à celui de z ; géométriquement, cette multiplication revient
alors à une rotation. Par exemple, la multiplication par i correspond à une rotation d’un
angle π/2 (dans le sens positif).

Exponentielle et fonctions trigonométriques dans C

Pour x dans IR, l’exponentielle est définie par la série (qui converge pour tout x)

x2 xn
ex = 1 + x + + ... + + ...
2! n!
On montre en analyse que la série définie par le même développement mais en remplaçant
le nombre réel x par le nombre complexe z converge également pour tout nombre complexe
z
z2 zn
ez = 1 + z + + ... + + ...
2! n!
Cette fonction e z est aussi appelée la fonction exponentielle dans le plan complexe.

D’une façon analogue, on peut également étendre aux nombres complexes les fonctions
trigonométriques. On a pour sin z et cos z, le développement suivant (on remplace x ∈ IR
par z ∈ C dans les développements habituels valables dans IR)

z2 z4 z3 z5
cos z = 1 − + − ... et sin z = z − + − ...
2! 4! 3! 5!

130
Dans le plan complexe comme dans le cas réel, on a les égalités

cos(− z) = cos z et sin(− z) = − sin z

et pour la fonction exponentielle, on a encore les importantes propriétés

e z1
e z1 e z2 = e z1 + z2 et = e z1 − z2
e z2
Une relation remarquable qui existe entre l’exponentielle et les fonctions trigonométriques
dans le domaine complexe résulte directement des développements ci-dessus. Notamment,
en remplaçant z par i z dans le développement de l’exponentielle, on trouve que, pour tout
z

z2 z3 z4
ei z = 1 + i z − − i + + ...
2! 3! 4!
2 4
z3 z5
µ ¶ µ ¶
z z
= 1 − + − ... + i z − + − ... = cos z + i sin z
2! 4! 3! 5!
En remplaçant z par − z dans cette relation, on obtient encore

e− i z = cos z − i sin z

En particulier, pour un z réel, notons-le θ , on a l’importante relation

e i θ = cos θ + i sin θ

Comme tout nombre complexe z peut s’écrire sous sa représentation trigonométrique par
z = r (cos θ + i sin θ ), où r est le module de z et θ son argument, on peut aussi l’écrire

z = r ei θ

Remarque :
En posant θ = π dans la relation pour e iθ , on obtient, sachant que cos π = −1 et sin π = 0,
une formule bien intéressante due à Euler

ei π
= −1

Cette formule à fait dire au mathématicien Krylov : ≪ Cette formule remarquable d’Euler
symbolise l’unité des mathématiques : −1 y représente l’arithmétique ; i l’algèbre ; π la
géométrie et e l’analyse ≫ .

28 Annexe II : Méthode des moindres carrés

28.1 Problématique

Lorsqu’on effectue des mesures pour vérifier un modèle physique ou une théorie, on est
amené bien souvent à vouloir ajuster une courbe aux points de mesure effectués. Il existe

131
bien des méthodes différentes selon le contexte envisagé. Nous présentons ici l’une des
plus connues appelée la méthode des moindres carrés.

Supposons que pour N grandeurs physiques x i , pour i = 1, ..., N nous mesurions, lors d’une
série d’expériences, les N autres grandeurs physiques yi ( i = 1, ..., N ). Par exemple dans
le cas de la mesure de la chute libre d’une bille, les x i peuvent représenter les différents
temps de chute et les yi les hauteurs de chute correspondantes. On peut tout aussi bien
faire le choix inverse si l’on préfère. Admettons qu’une théorie donne le lien entre les
grandeurs physiques envisagées sous la forme d’une fonction connue f dépendant de un
ou plusieurs paramètres notés a 1 , a 2 , ..., a k que l’on notera collectivement par le vecteur ~
a,
pour simplifier. Ainsi

y = f (~
a, x)

Dans l’exemple de la chute des corps, la fonction qui associe le temps de chute t à la
hauteur de chute h est donnée par h = 1/2 gt2. Il s’agit ici d’une fonction du deuxième
degré qui dépend d’un seul paramètre g, la gravitation terrestre. Si, dans l’expérience de
la chutes des corps, il s’agit de déterminer l’accélération terrestre, il faut alors ajuster le
mieux possible une parabole aux points de mesure. De l’équation de cette parabole, on
obtient une estimation de g. Mais que signifie ≪ ajuster le mieux possible ≫ ?

Pour réaliser cet ajustement, la première méthode qui vient à l’esprit consiste à minimiser
l’écart entre le modèle (donné par la fonction f ) et les mesures (fournies par les couples
( x i , yi )). Autrement dit, il s’agit de trouver un jeu de paramètres a 1 , ..., a k qui minimise la
quantité suivante
N
a, x i ))2
X
S (~
a) = ( yi − f (~
i=1

Cette quantité est manifestement positive (c’est une somme de carrés) et elle serait nulle
si les mesures étaient idéales, sans incertitudes. Ainsi, un choix des paramètres ~
a qui rend
minimale la quantité ci-dessus permet un bon ajustement du modèle aux mesures. D’où le
terme ≪ moindres carrés ≫ pour cette procédure.

On peut affiner la procédure précédente en minimisant non pas la quantité ci-dessus, mais
la suivante :

a, x i ) 2
N µ y − f (~ ¶
2
X i
χ =
i=1 σi

dans laquelle σ i désigne l’incertitude connue de la mesure yi . On parle aussi d’écart-type


pour les σ i . On peut ainsi accorder un poids plus grand à une mesure dont on sait que
son incertitude est petite. Par exemple si l’incertitude σ1 associée à la mesure ( x1 , y1 ) est
a, x i ))/σ1 )2 sera grand par rapport aux autres termes. Il aura plus de
petite, le terme (( yi − f (~
poids dans la somme. Cette méthode s’appelle la méthode du χ2 (≪ chi-carré ≫ ), prononcez
khi-carré.

Dans le cas particulier où l’incertitude est la même pour chaque mesure (tous les σ i sont
les mêmes, disons σ), c’est-à-dire chaque mesure a le même poids, la méthode du χ2 se

132
réduit à celle des moindres carrés. En effet, puisqu’on peut mettre en facteur σ2 dans
l’expression du χ2 , la minimisation du χ2 donne le même jeu de paramètres ~a que dans
la méthode des moindres carrés. Dans la suite, nous nous limiterons aux applications de
cette dernière méthode.

Remarque concernant le χ2 :
Etant donné un ensemble de paramètre a 1 , a 2 , ..., a k qui définissent un modèle, on peut
se poser la question suivante. Quelle est la probabilité de mesurer ce modèle avec une
certaine tolérance en chaque point de mesure (la tolérance étant fixée à l’avance) ? On
définit ainsi la vraisemblance statistique du modèle (ou du jeu de paramètres) vis-à-vis
des mesures comme la probabilité d’obtenir ces mesures lorsque le modèle est choisi (et
donc supposé vrai). Il est important de garder à l’esprit qu’une vraisemblance statistique
n’est pas une preuve de la justesse d’une théorie physique !

On suppose que chaque mesure yi est entachée d’une erreur aléatoire, indépendante pour
chaque mesure, et distribuée selon une loi normale par rapport au modèle f (~ a, x) choisi
(distribution selon une courbe en cloche). L’incertitude (ou l’écart-type) σ i pouvant être
différente d’une mesure à l’autre. On peut alors montrer que le maximum de vraisem-
blance (c’est-à-dire la probabilité maximale de l’adéquation des mesures au modèle pour
une tolérance fixée) correspond aux paramètres que l’on obtient en minimisant le χ2 . Il
est difficile, voire impossible, de s’assurer que les mesures sont distribuées selon une loi
normale. Dans les faits, il s’agit d’une hypothèse assez forte et rarement vérifiable. Dans
la pratique, on écarte les mesures qui, manifestement, sont en dehors de ce qui semble
être la ≪ normalité ≫ et l’on applique la méthode du χ2 , ou celle des moindres carrés, sur la
base des mesures restantes.

28.2 Régression linéaire

Dans cette situation, le modèle qui relie les grandeurs est une fonction linéaire affine. La
relation entre les grandeurs x et y est de la forme :

y = ax + b

La fonction f dépend de deux paramètres a et b. Elle s’exprime par

f (a, b, x) = ax + b

Il faut donc ajuster une droite à un ensemble de N mesures ( x i , yi ) pour i = 1, ..., N . Dési-
gnons par S (a, b) la fonction de la méthode des moindres carrés à minimiser

N
( yi − ax i − b)2
X
S (a, b) =
i=1

Le problème consiste donc à trouver a et b qui minimise la fonction des deux variables a
et b. La condition est que la dérivée de S par rapport à a s’annule et aussi que la dérivée
de S par rapport à b s’annule également. Désignons par ∂a S la première dérivée et par

133
∂b S la seconde. On a donc, en utilisant la dérivée en chaîne :
N
X N
X
∂ a S = −2 x i ( yi − ax i − b) = 0 et ∂ b S = −2 ( yi − ax i − b) = 0
i=1 i=1

De la première relation, on tire :


N N N
x2i + b
X X X
a xi = x i yi
i=1 i=1 i=1

et de la deuxième relation on tire


1 XN 1 XN
b= yi − a xi
N i=1 N i=1

Ces deux dernières équations constituent un système linéaire pour les inconnues a et b
qu’il est facile de résoudre (2 équations à 2 inconnues). Les sommes présentes dans ces
équations sont des grandeurs connues, car elles s’expriment toutes à l’aide des x i et des
yi . On les calcule donc au départ.

Il est donc utile pour la suite de définir les nombres suivants que l’on calcule une fois pour
toutes à l’aide des couples de mesures ( x i , yi ).

1 XN
x= xi moyenne des x i
N i=1

1 XN
y= yi moyenne des yi
N i=1
1 XN
x2 = x2 moyenne des carrés des x i
N i=1 i

1 XN
xy = x i yi moyenne des produits x i yi
N i=1

A l’aide de ces constantes, les deux équations pour a et b s’écrivent

a x2 + b x = x y et b = y−a x

On obtient donc pour a et b, les paramètres qui permettent d’ajuster au mieux les points
de mesures par régression linéaire :

xy − x · y
a= et b = y−a x
x 2 − ( x )2

Remarque :
Dans la situation particulière où l’on sait que le modèle prédit une relation linéaire entre
x et y qui passe par l’origine, la fonction f ne dépend donc plus que d’un seul paramètre
a. Autrement dit f (a, x) = ax, la fonction S (a) ne dépend plus que de a également et il faut
minimiser la fonction S (a) suivante
N
( yi − a x i )2
X
S ( a) =
i=1

134
Il faut calculer la dérivée de S par rapport à a et imposer que cette dérivée s’annule. De
manière similaire au cas précédent, on trouve pour a dans ce cas

xy
a= régression linéaire passant par l’origine
x2

On obtiendrait le même résultat en imposant b = 0 dans la relation pour b ci-dessus.

28.3 Modèles linéaires à M paramètres

On peut généraliser la méthode de régression linéaire dans les situations où les courbes
d’ajustage ne sont pas des droites mais des fonctions quelconques. Supposons que le mo-
dèle soit caractérisé par une combinaison linéaire de M fonctions données que l’on désigne
par g 1 ( x), g 2 ( x),..., g M ( x). Autrement dit la fonction f s’exprime par

M
X
f (~
a, x) = a 1 g 1 ( x) + a 2 g 2 ( x) + ... + a M g M ( x) = a k g k ( x)
k=1

où ~
a = (a 1 , a 2 , ..., a M ) désigne le vecteur formé des paramètres a 1 ,...,a M . Dans l’expression
pour f , les fonctions g k ( x) sont définies par le modèle et peuvent être a priori quelconques.
Le problème des moindres carrés consiste donc à minimiser la fonction suivante
à !2
N
X M
X
S (~
a) = yi − a k g k (xi )
i=1 k=1

Rappelons que, dans cette expression, les couples ( x i , yi ) pour i = 1, ..., N désignent les N
mesures pour lesquelles nous souhaitons obtenir un jeu de paramètres a 1 ,...,a M qui per-
met d’ajuster au mieux la ≪ fonction-modèle ≫ f (~ a, x) à ces mesures. La condition nécessaire
pour obtenir un minimum consiste à exiger que la dérivée de S (~ a) par rapport à chaque
paramètre s’annule. Désignons par ∂l S (~ a) la dérivée de S (~ a) par rapport au paramètre a l
(dérivée partielle). On a donc
à !
N
X M
X
∂l S (~
a) = 2 yi − a k g k ( x i ) (− g l ( x i )) = 0 ∀ l = 1, ..., M
i=1 k=1

En regroupant les termes, on obtient les conditions suivantes sur les paramètres :
à !
XM X N N
X
g l (xi ) g k (xi ) a k = yi g l ( x i ) ∀ l = 1, ..., M
k=1 i=1 i=1

On obtient donc un système de M équations linéaires pour les M inconnues a 1 ,...,a M .


Ecrivons-le en notation matricielle. Pour ce faire, on définit la matrice B d’ordre M × M
dont les éléments de matrice sont les suivants

N
X
B lk = g l (xi ) g k (xi )
i=1

135
Cette matrice est symétrique (B lk = B kl ) et réelle et peut donc être mise sous forme diago-
nale. De plus, toutes ses valeurs propres sont strictement positives. La matrice B est donc
inversible. Désignons par B−1 la matrice inverse de B. Sous forme matricielle, le système
précédent s’écrit ainsi

B~
a = ~c ⇒ a = B−1 ~
~ c

Avec
   PN   
a1 i=1 yi g 1 ( x i ) B11 B12 ... B1 M
 a2   P N yi g 2 ( x i )   B21 B22 ... B2 M 
~
a= ~c =  i=1 . B=
     
.. 
..  .. .. .. 
 .     . . . 
PN
aM i=1 yi g M ( x i )
BM1 BM2 . . . BM M

Si le nombre de paramètres est grand, le problème principal consiste à inverser la ma-


trice B. Ce qui ne constitue pas une réelle difficulté avec les algorithmes utilisés dans les
ordinateurs actuels.

28.4 Applications

Exemple 1 :
Comme premier exemple pour se familiariser avec la méthode, reprenons le cas particulier
de la régression linéaire.

f (~
a, x) = a 1 x + a 2 ⇒ g 1 ( x) = x et g 2 ( x) = 1

Le vecteur ~c et la matrice B s’écrivent


µ PN ¶ µ ¶ µ PN 2 PN ¶ Ã !
i =1 yi x i xy i =1 xi i =1 xi x2 x
~c = PN =N et B = PN =N
i=1 yi
y i=1 x i N x 1

Le système linéaire prend alors la forme suivante (après avoir inversé la matrice B d’ordre
2 × 2)
à !µ à !µ
1 −x
¶ µ ¶ µ ¶ ¶
x2 x a1 xy a1 1 xy
= ⇒ = 2
x 1 a2 y a2 x 2 − ( x )2 − x x y

On obtient donc finalement (après avoir effectué le produit matricielle)


à !
xy − x · y
µ ¶
a1 1
= 2
a2 x 2 − ( x )2 x · y − x y · x
ce qui est bien ce que l’on avait obtenu.

Exemple 2 :
Considérons le cas où la fonction-modèle est une parabole de la forme

f (a, x) = a x2 ⇒ g 1 ( x) = x2

136
où seul est présent le monôme du deuxième degré (ce serait le cas par exemple pour l’ex-
périence de chute libre à vitesse nulle au départ). Dans cette situation à un paramètre, la
matrice B se réduit à un nombre et le vecteur ~c n’a qu’une seule composante. On obtient
N N
x4i yi x2i
X X
B11 = et c1 =
i=1 i=1

Le paramètre a s’écrit donc


PN
i=1 yi x2i yx2
a= P N 4
=
i=1 x i x4

où, dans la dernière expression, on a défini la moyenne des produits yi x2i et la moyenne
des x4i respectivement :

1 XN 1 XN
yx2 = yi x2i et x4 = x4
N i=1 N i=1 i

Exemple 3 :
Comme dernier exemple de modèle linéaire à M paramètres, considérons la situation où
la fonction f est également du type parabolique, mais, cette fois, le modèle n’impose pas la
nullité du monôme du premier degré, ni celle du terme constant.

f ( a 1 , a 2 , a 3 , x) = a 1 x2 + a 2 x + a 3 ⇒ g 1 ( x) = x2 , g 2 ( x) = x, g 3 ( x) = 1

La matrice B s’écrit, en suivant la même démarche que précédemment et avec les défini-
tions similaires pour les moyennes des puissances de x i
 
x4 x3 x2
B = N  x3 x2 x 
 

x2 x 1

Quant au vecteur ~c, il devient


 
yx2
~c = N  yx 
 
y

Il ne reste plus qu’à résoudre le système linéaire d’ordre 3 suivant (inversion d’une matrice
3 × 3) :
    
x4 x3 x2 a1 yx2
 3
 x x2 x   a 2  =  yx 
  

x2 x 1 a3 y

137
Remarque finale :
Dans bien des cas (la majorité), la fonction-modèle f (~ a, x) ne dépend pas linéairement des
paramètres ~ a. La méthode précédente ne peut donc pas s’appliquer telle quelle. Il faut
revenir à la définition générale de la fonction S (~
a), la dériver par rapport aux paramètres
et, finalement, imposer à ces dérivées de s’annuler. Une des difficultés des problèmes de
moindres carrés non-linéaires est l’existence fréquente de plusieurs minima locaux. Une
exploration systématique de l’espace des paramètres peut alors se révéler nécessaire.

Références
[1] "Cours de physique II : Electricité" par J.-A. Monard, Bienne (1992).
[2] "Physique" par Eugène Hecht, DeBoeck Université (1999).
[3] "Physique II : Electricité et Magnétisme" par D. Halliday, R. Resnick, J. Walker, Du-
nod (2003).
[4] "The Feynman lectures on physics" par R. Feynman, R. B. Leighton, M. L. Sands,
Addison-Wesley (1963).

138
29 Exercices

29.1 Série 1 : charge, loi de Coulomb, champ électrique

Exercice 1.1
Deux charges égales, situées à 1 m l’une de l’autre, se repoussent avec une force de 1 N.

Que valent ces charges ?

Exercice 1.2
Comparer les forces de gravitation et les forces de Coulomb qui s’exercent entre :
a) deux électrons ;
b) deux protons ;
c) un proton et un électron.

Exercice 1.3
Une boule de cuivre a un diamètre de 10 cm. On suppose qu’on lui arrache tous ces élec-
trons libres (1 électron libre par atome).

Que vaut la force qui serait exercée par cette boule sur une boule semblable située sur la
Lune (distance Terre-Lune : 384′000 km) ?

Exercice 1.4
Dans le modèle de l’atome d’hydrogène proposé par Bohr en 1913, l’électron décrit une
orbite circulaire autour du noyau (constitué que d’un seul proton). Le rayon de l’orbite est
de 0, 53 · 10−10 m.

Combien l’électron accomplit-il de rotations par seconde ?

Exercice 1.5
Deux charges électriques ponctuelles de 10µ C et −10µ C sont placées en deux points A et
B distants de 16 cm.

Déterminer le champ électrique (norme, direction, sens) qu’elles produisent :


a) au milieu du segment AB ;
b) en un point situé à 10 cm de chacune des charges.

Exercice 1.6
Dans un oscilloscope, les électrons sont accélérés sur 2 cm par un champ électrique de
5 · 105 N/C.

139
a) Quelle est leur vitesse lorsqu’ils sortent du champ ?

Ils traversent ensuite, sur une longueur de 5 cm, un autre champ électrique, uniforme,
dirigé perpendiculairement à leur vitesse initiale. Ils y subissent une déflection de 1 cm et
finissent par atteindre le bord de l’écran.

b) Quelle est la valeur du second champ ?

29.2 Série 2 : flux, tension, potentiel

Exercice 2.1
Le champ électrique qui se trouve juste au-dessus du tambour photorécepteur chargé d’un
photocopieur a une grandeur E = 2, 3 · 105 N/C. Ce tambour a une longueur de 42 cm et un
diamètre de 12 cm.

a) Quelle est la charge nette du tambour ?

Le fabricant prévoit de produire une version portable de l’appareil. Pour y arriver, il doit
réduire la taille du tambour pour qu’il ait 28 cm de long et 8 cm de diamètre. Le champ
électrique à la surface du photorécepteur doit demeurer inchangé.

b) Déterminer la charge du nouveau tambour.

Exercice 2.2
Au voisinage d’un conducteur chargé, le champ électrique a une intensité de 10′ 000 V/m.

Calculer la charge que porte une petite surface de 5 mm2 de ce conducteur.

Exercice 2.3
On a deux sphères métalliques creuses, d’épaisseur négligeable et de rayons R 1 et R 2 (R 1 <
R 2 ). La plus grande peut être divisée en deux hémisphères. On place sur la petite sphère
une charge positive Q 1 et sur l’un des hémisphères une charge Q 2 , également positive. On
place les deux hémisphères l’un contre l’autre autour de la petite sphère, de manière à
obtenir deux sphères concentriques.

Déterminer la valeur du champ électrique en tout point de l’espace.

Exercice 2.4
On relie les bornes d’une pile de 4, 5 V à un fil métallique ayant 1 km de long.

En admettant que le champ électrique ait la même intensité tout le long du fil, calculer la
force qu’il exerce sur les électrons libres et l’accélération que ceux-ci subissent entre deux
chocs consécutifs.

140
Exercice 2.5
Dans un champ électrique uniforme, on considère trois points A , B et C constituant les
sommets d’un triangle équilatéral de 10 cm de côté. Le vecteur E ~ a une grandeur de 15
◦ −−→ ◦ −−→
V/m. Il fait un angle de 20 avec le vecteur AB et un angle de 40 avec le vecteur AC .

Déterminer les tensions entre A et B, entre B et C et entre C et A .

Exercice 2.6
Quelle est la vitesse acquise par un électron initialement immobile, s’il est accéléré sur un
parcours entre les extrémités duquel il y a une tension de 1 V ?

Exercice 2.7
Un faisceau parallèle contient des électrons ayant des vitesses différentes. Pour sélection-
ner ceux qui ont une vitesse donnée, on utilise le dispositif de la figure.

d = 5 cm

M
α T1 T2
x = 6 cm

faisceau incident

Le faisceau est introduit à travers un trou T1 et sous un angle α = 60◦ dans l’espace
compris entre deux plaques conductrices parallèles M et N , distantes de 5 cm. Un second
trou T2 est percé dans la même plaque que T1 , à une distance de 6 cm. On établit entre M
et N une tension U .

Déterminer la valeur de U pour que les électrons sortant de T2 aient une vitesse de 1′ 000
km/s.

Exercice 2.8
Une boule pleine (isolante) de rayon R est uniformément chargée. Sa charge totale est Q
(disons Q > 0). Montrer que le champ électrique E à une distance r du centre de la boule
vaut :
Q
a) E = 4πǫ0 Rr3 si r ≤ R ;
Q
b) E = 4πǫ0 r12 si r > R .

141
Exercice 2.9
Deux plaques parallèles, horizontales, sont séparées d’une distance d = 5 cm. On charge
une plaque positivement et l’autre négativement. Le champ électrique E produit par ces
charges est homogène entre les plaques, mais loin des bords, en excellente approximation.
Ce système s’appelle un condensateur plan. Imaginons une petite boule de masse m = 1 g,
chargée d’une charge q = 1µC, qui reste en équilibre entre les plaques (la force électrique
compensant la pesanteur de la boule).
a) Indiquer sur un schéma la polarité des plaques et calculer la tension entre ces plaques.
b) Quelle serait l’accélération de la boule si l’on triplait la tension aux bornes de ces
plaques ?

Exercice 2.10
L’unité d’énergie la plus utilisée en physique nucléaire est le [MeV] = 106 [eV] (pour
≪ électron-volt ≫ ) qui est l’énergie acquise par une particule possédant la charge élémen-

taire ± e (électron, proton, etc.) qu’on accélère sous une tension de 1 V.


a) Exprimer 1 MeV en J.
b) Quelle est la vitesse d’un électron dont l’énergie cinétique est de 20 MeV ?

Exercice 2.11
A l’aide d’une instrumentation adéquate, on détecte les lignes équipotentielles sur une
certaine surface. On inscrit les résultats sur un graphique (voir figure).

y cm

50

3
30 25 0 V
2 V
15 0 V
10 V
10 5 V
V
0 10 30 50 x cm

a) Quelle est la direction et quel est le sens du champ électrique ?


b) Calculer la norme du champ électrique, ainsi que ses composantes E x et E y .

29.3 Série 3 : condensateurs

Exercice 3.1
Un condensateur plan est formé de deux plaques circulaires de 20 cm de diamètre, dis-
tantes de 2, 4 cm.

142
a) Quelle est la charge qu’il emmagasine lorsqu’on le relie à une pile de 12 V ?

b) On enlève les connexions et, sans changer la charge, on place les plaques à 1 mm l’une
de l’autre. Calculer la nouvelle tension.

Exercice 3.2
Calculer la capacité d’un condensateur formé de deux cylindres coaxiaux ayant des dia-
mètres de 19, 9 cm et 20, 1 cm, une épaisseur négligeable et une hauteur de 30 cm.

Exercice 3.3
Un condensateur plan a une capacité de 400 pF et porte une charge de 4 · 10−7 C. Deux
petits trous ont été faits dans les plaques. Un proton est lancé à travers les petits trous et
décrit une trajectoire perpendiculaire aux plaques.

Connaissant la vitesse initiale de la particule, 400 km/s, calculer sa vitesse après la tra-
versée du condensateur (une fois dans un sens et une fois dans l’autre).

Exercice 3.4
La pile a une tension de 10 V et la capacité de chacun des cinq condensateurs est de 10µF.

C2
C4
C3
C1 +
− U = 10 V
C5

Quelle est la charge du condensateur 1 et la charge du condensateur 2 ?

Exercice 3.5
Un condensateur a une capacité C 1 et possède une charge Q . On relie ses armatures à
celles d’un condensateur déchargé et de capacité C 2 .

Calculer la perte d’énergie que subit le système en passant de son état initial à son état
final.

Exercice 3.6
On augmente la distance entre les deux armatures d’un condensateur plan de 1 cm à 2
cm. Au départ, la tension aux bornes du condensateur valait 100 V.

a) Quelle est la nouvelle tension aux bornes du condensateur ?

143
b) Quelle est la variation de l’intensité du champ électrique dans le condensateur ?

c) De combien l’énergie emmagasinée a-t-elle changé ?

29.4 Série 4 : courants, circuits

Exercice 4.1
Un condensateur de 4µF est chargé sous une tension de 100 V. On joint ses armatures par
un conducteur de grandes résistance. 20 secondes plus tard, on constate que la tension
s’est abaissée à 99, 5 V.

Quel est le courant de décharge ?

Exercice 4.2
Un fil de maillechort a une section de 1 mm2 . Il y règne un champ électrique de 6 V/m. La
résistivité du maillechort est de 3 · 10−7 Ωm.

Calculer l’intensité du courant dans le fil.

Exercice 4.3
Un câble de cuivre de densité 8, 94 a une masse de 200 kg et sa résistance est de 0, 64 Ω.

Calculer sa longueur et sa section.

Exercice 4.4
Un condensateur de 1µF porte une charge de 10−3 C. On le relie à une résistance de 1 MΩ.

a) Calculer le courant au début de la décharge.

b) Expliquer pourquoi ce courant n’est pas constant.

c) En admettant qu’il soit à peu près constant pendant le premier centième de seconde de
la décharge, calculer la valeur de la charge et de la tension du condensateur après ce laps
de temps.

Exercice 4.5
Dans le circuit, la résistance de 3Ω est parcourue par un courant de 12 mA.

144
4Ω
1Ω
2Ω 5Ω
3Ω

− +

Calculer la tension aux bornes de la source.

Exercice 4.6
Calculer le courant dans chaque résistance et le potentiel de chaque fil de connexion. Pla-
cer le point de référence du potentiel au point O indiqué.

10Ω
6Ω
2Ω 4Ω
A b b
B C b b
D
40Ω
12Ω
+ − b

O
1V

Exercice 4.7
Déterminer le courant (avec son sens) dans chaque branche du circuit. Les tensions des
sources sont données, ainsi que leur résistance interne.

10 V
2Ω 10Ω
− +
V
6Ω 102Ω − + 2Ω
− + − +
10 V 40 V
2Ω 4Ω

Exercice 4.8
Calculer tous les courants du circuit ci-dessous. On néglige les résistances internes des
sources de tension.

145
5Ω
20 V 4Ω 7V

2Ω

29.5 Série 5 : puissance, générateur, récepteur

Exercice 5.1
On paie 1, 08 francs pour utiliser pendant 3 heures un radiateur électrique. Sachant que
le courant qui le traverse est de 5 A et que le prix du kWh est de 24 centimes, calculer :
a) la tension aux bornes de l’appareil.
b) la puissance électrique consommée.

Exercice 5.2
Une lampe de 60 W est branchée sur un générateur ayant une tension de 220 V entre ses
bornes. Que vaut sa résistance ?

Même problème pour une ampoule de phare de voiture, de puissance identique, mais ali-
mentée par un générateur de 12 V.

Pourquoi la lampe de voiture résiste-t-elle mieux aux chocs que la lampe du salon ?

Exercice 5.3
Il faut 5 minutes pour chauffer un litre d’eau de 20 à 100 degrés. La cuisinière électrique
est alimentée par un générateur dont la tension aux bornes vaut 220 V.

Sachant que les pertes de chaleur s’élèvent à 30% de l’énergie totale consommée, calculer
la résistance de la plaque.

Exercice 5.4
On dispose de trois lampes L 1 , L 2 et L 3 , sur lesquelles on peut lire :

L 1 : 220V/66W, L 2 : 220V/110W, L 3 : 150V/75W

On dispose également d’une série de résistances. Comment brancher ces trois lampes si-
multanément sur un générateur ayant 220 V entre ses bornes pour obtenir l’intensité
lumineuse maximale ?

Quelle sera alors la puissance électrique fournie par le générateur ? Dessiner un schéma
du montage.

146
Exercice 5.5
Lorsque l’on réunit les bornes d’une pile par une résistance de 3 Ω, elle débite un courant
de 0, 3 A. Si la résistance vaut 15 Ω, le courant n’est plus que de 0, 1 A.

On demande la tension électromotrice et la résistance interne de la pile.

Exercice 5.6
Un générateur a une tension électromotrice de 6 V et une résistance interne de 2 Ω.

a) Quel est le courant maximum qu’il peut débiter ?

b) On le branche sur une résistance de 10 Ω. Que vaut la chaleur dégagée en une minute
dans cette résistance ?

Exercice 5.7
Un moteur électrique est alimenté par un générateur de tension électromotrice égale à 140
V et de résistance interne négligeable. Il actionne une pompe qui élève l’eau d’un puits de
7, 5 m, à raison de 56 litres à la minute.

a) Quelle est la puissance utile délivrée par la pompe ?

b) Quelle est la puissance fournie par le moteur à la pompe si l’on admet que les 10% de
cette puissance sont perdus pour des causes diverses ?

c) En supposant que les frottements à l’intérieur du moteur font perdre les 20% de l’énergie
mécanique produite, calculer la tension contre-électromotrice et la résistance interne du
moteur, la valeur du courant étant de 0, 8 A.

d) Quelle est la quantité de chaleur dégagée dans le moteur en 5 minutes ?

e) Quelle valeur prendrait cette même quantité si, par suite d’un blocage de la pompe, le
moteur se trouvait dans l’impossibilité de tourner ?

Exercice 5.8
Une pile a une tension électromotrice Uem et une résistance interne r . On branche cette
pile à un rhéostat (résistance variable R ).

a) Calculer la puissance dissipée dans le rhéostat en fonction de Uem , r et R .

b) Quelle valeur R faut-il régler au rhéostat pour que la puissance dissipée dans celui-ci
soit maximale ?

147
29.6 Série 6 : ampèremètres, voltmètres

Exercice 6.1
Un ampèremètre dont la résistance est de 100 Ω voit son aiguille dévier de 100 divisions
lorsqu’il est parcouru par un courant de 10 mA.

Quelle résistance placer en dérivation (en parallèle) entre ses bornes pour l’utiliser à me-
surer des courants pouvant atteindre 1 A ?

Exercice 6.2
Un ampèremètre a une résistance de 5 Ω. La déviation totale de l’aiguille a lieu pour un
courant de 0, 1 A.

a) On met en série une résistance de 1000 Ω et on utilise cet ensemble comme voltmètre.
Quelle est la tension qui provoque la déviation totale de l’aiguille ?

b) On mesure la tension aux bornes d’une résistance de 500 Ω faisant partie d’un circuit
complexe. Que dire du résultat ?

Exercice 6.3
Un ampèremètre est shunté pour que sa sensibilité soit divisée par 50. La résistance équi-
valente de l’appareil est alors de 0, 2 Ω.

Quelle est la résistance propre de l’ampèremètre et quelle est la valeur du shunt ?

Exercice 6.4
Un générateur a une tension constante de 12 V à ses bornes. On le branche sur une résis-
tance de 125 Ω.

a) Que valent la tension et le courant dans la résistance ?

b) On complète le circuit avec un voltmètre aux bornes de la résistance et un ampère-


mètre à la sortie du générateur. Les résistances internes sont respectivement de 1000 Ω
et de 10 Ω. Déterminer la valeur indiquée par chaque instrument de mesure, ainsi que le
courant dans la résistance de 125 Ω.

29.7 Série 7 : particules dans un champ magnétique

Exercice 7.1
Un électron se déplace au-dessus de l’atmosphère à une vitesse de 1000 km/s, perpendi-
culairement aux lignes de champ magnétique terrestre. Dans la région où il se trouve, la
norme du champ vaut 10−6 T.

148
Déterminer le rayon de courbure de sa trajectoire.

Exercice 7.2
Une particule α ( q = 2 e, m = 4 u) suit une trajectoire circulaire d’un rayon de 4, 5 cm dans
un champ magnétique uniforme de grandeur B = 1, 2 T. Calculer
a) sa vitesse ;
b) sa période ;
c) son énergie cinétique en eV (électron-volt) ;
d) la différence de potentiel par laquelle il devrait être accéléré pour avoir cette énergie.

Exercice 7.3
Dans un spectromètre de masse, les ions sont accélérés entre deux électrodes par une ten-
sion de 1000 V. Ils sont défléchis ensuite dans un champ magnétique uniforme, d’intensité
égale à 0, 1 T. Ils parcourent un demi-cercle avant d’atteindre un détecteur.

Quelle distance séparera la position du détecteur récoltant des ions de Mn55 de celui ré-
coltant des ions de Mn56 ? Les ions sont doublement ionisés ( q = 2 e).

Exercice 7.4
Une particule de masse m et de charge négative q se déplace librement. Elle pénètre dans
une région de l’espace où règne un champ magnétique B~ uniforme de norme égale à 3 · 10−4
T. Elle décrit alors un cercle de 10 cm de rayon.

Si l’on avait judicieusement superposé à B ~ un champ électrique E~ uniforme de norme


égale à 1500 V/m, la particule n’aurait pas été déviée, mais aurait poursuivi sa trajectoire
initiale. On demande de déterminer :
a) les directions et sens de E~ et B
~ (faire un dessin) ;
b) le rapport | q|/ m de la particule.

Exercice 7.5
Un positron a une énergie cinétique de 2, 0 keV. Il est projeté dans un champ magnétique
uniforme B ~ de norme 0, 1 T. Sa vitesse forme un angle de 89◦ avec B
~ . Trouver :
a) la période ;
b) le pas ;
c) le rayon de sa trajectoire hélicoïdale.

Exercice 7.6
Le flux sanguin dans une artère transporte des ions positifs de sodium (Na+) et des ions
négatifs de chlore (Cl− ) à une vitesse inconnue. La figure représente la section d’une artère
cylindrique plongée dans un champ magnétique B ~ constant et perpendiculaire à l’axe du
cylindre.

149
~
B

Na+ Cl−
N + − S

a) Expliquer, à l’aide d’une figure, l’apparition d’une tension entre les bords de l’artère.

b) On mesure une tension U = 300 µV entre les bords d’une artère de diamètre d = 1, 2 cm
pour un champ B = 0, 1 T. Calculer la vitesse des ions et déterminer le débit du sang.

29.8 Série 8 : force de Laplace

Exercice 8.1
Deux rails métalliques parallèles, distants de d = 10 cm, sont inclinés de α = 20◦ par
rapport à l’horizontale. Leurs extrémités supérieures sont reliées à travers un générateur
de courant continu.

~
B
G

L’ensemble est placé dans un champ magnétique B ~ homogène, uniforme, vertical, dirigé
vers le haut et de norme égale à 0, 05 T. On pose un barreau conducteur cylindrique sur
les rails, perpendiculairement à leur direction. Le barreau reste immobile à l’endroit où il
est posé. Sa masse vaut 20 g.

a) Quel est le sens du courant dans le barreau ?

b) Quelle est sont intensité ? (Les forces de frottement sont négligeables.)

Exercice 8.2
Dans deux conducteurs horizontaux, distants de 2 mm, circulent des courants de 10 A en
sens inverses.

Déterminer la force subie par un tronçon de fil de 10 cm de long. La comparer avec le poids
du tronçon, sachant qu’il est en cuivre et que sa section est de 1 mm2 .

150
Exercice 8.3
Deux longs fils parallèles sont disposés perpendiculairement à un champ magnétique uni-
forme d’intensité égale à 2 · 10−7 T. Un courant de 1 A parcourt les deux fils en sens inverse.

Pour quelle distance d entre les deux fils n’y aura-t-il pas de force magnétique résultante
agissant sur l’un ou l’autre de ces fils ?

Exercice 8.4
Une bobine plate, rectangulaire, mobile autour de son grand axe qui est vertical, comporte
20 spires de 4 × 5 cm2 . Elle est placée dans un champ magnétique uniforme, horizontal,
d’intensité égale à 0, 1 T. Elle est parcourue par un courant de 5 A tout en étant initiale-
ment maintenue en sorte que son plan fasse un angle de α = 45◦ avec les lignes de champ.
On demande :

z
y
~
B x

5 cm bobine plate

m
4c

a) d’indiquer le sens du courant dans la bobine pour qu’elle tourne comme indiqué ;

b) de calculer la force qui s’exerce sur un des côtés horizontaux de la bobine et la force qui
s’exerce sur un des côtés verticaux de la bobine.

c) Que se passe-t-il si on laisse la bobine se déplacer librement autour de son axe ?

Exercice 8.5
La figure illustre un fil rectiligne parcouru par un courant I 1 = 30 A. Un cadre rectangu-
laire, placé dans un plan contenant le fil, est parcouru par un courant I 2 = 20 A.

I 1 = 30 A
c = 1 cm
I 2 = 20 A
b = 8 cm

a = 30 cm

a) Décrire les forces qu’exerce le fil sur chaque côté du cadre (direction et sens).

b) Déterminer la résultante de ces forces (norme, direction, sens).

151
Exercice 8.6
Pour cet exercice on se réfère à la figure de la section qui discute du moteur électrique à
courant continu. Un tel dispositif peut être utilisé comme une dynamo. On fait tourner
mécaniquement le rotor et l’on produit ainsi du courant. La bobine rectangulaire enroulée
sur le rotor a les dimensions 10 cm×8 cm (hauteur × diamètre du rotor) et contient N = 500
spires. On souhaite alimenter en courant ( I = 0, 4 A) une ampoule de lampe de poche pour
qu’elle fonctionne normalement (sa résistance est de R = 15 Ω lorsque un courant de 0, 4 A
la traverse). Le champ magnétique dans l’entrefer vaut B = 0, 01 T.

rotor
I noyau de fer
bobine rectangulaire

~
B
I

~
v
I

a) Expliquer le fonctionnement de cette dynamo.

b) A combien de tours par seconde la bobine doit-elle tourner pour que l’ampoule brille
normalement ?

Exercice 8.7
Deux barrettes de longueurs a et de masse m sont placées horizontalement l’une contre
l’autre. Elles sont suspendues par des fils souples de longueur L. On relie les fils à une
batterie comme l’indique le dessin. Ce système est utilisé comme ampèremètre.


+

Calculer le courant I en fonction de l’angle α formé par l’un des fils avec la verticale.
Application numérique : m = 3 g, α = 2◦ , a = 10 cm, L = 12 cm.

152
Exercice 8.8
Une spire circulaire de 10 cm de diamètre est suspendue horizontalement au fléau d’une
balance. Elle est parcourue par un courant de 20 A. Une autre spire, identique, est fixée
horizontalement au-dessous de la première. Les deux centres sont sur la même verticale,
séparés par 3 mm lorsque la balance est en équilibre et que le courant dans la seconde
spire est nul.

On fait passer un courant de 10 A dans la deuxième spire. Quelle masse faut-il enlever ou
ajouter dans le plateau de la balance pour rétablir l’équilibre ? Discuter les deux situations.

Exercice 8.9
Deux rails parallèles distants de L = 10 cm sont inclinés de α = 30◦ par rapport à l’hori-
zontale. Un petit barreau de masse m = 20 g peut rouler sur les rails sans frottement. Il
est appuyé sur deux butées au bas des rails. Rails et barreau forment un circuit électrique
traversé par un courant I = 10 A.

α +

On établit brusquement autour du circuit un champ magnétique uniforme, vertical, d’in-


tensité B = 0, 5 T. Le barreau monte le long des rails.

Quelle est sa vitesse v lorsqu’il a parcouru la distance d = 20 cm ? Résoudre le problème


de façon littérale avant d’en donner l’application numérique.

Exercice 8.10
On fait référence ici à l’expérience vue en classe sur le mouvement d’une tige métallique
(de masse m = 5 g) posée sur deux rails conducteurs horizontaux séparés d’une distance
d = 6 cm, le tout immergé dans un champ B = 0, 1 T homogène (produit par un aimant

153
permanent). En faisant passer un courant I dans la tige (via les rails branchés sur un
générateur), cette dernière se met alors en mouvement sous l’effet de la force de Laplace.
Imaginons qu’une force de frottement statique existe entre la tige et les rails (de coefficient
statique µ0 = 0, 6). Rappel : Ffr ≤ µ0 S avec S la force de soutien (des rails sur la tige).

a) Pour un champ B vertical, calculer le plus petit courant I qu’il faut faire passer dans la
tige pour la mettre en mouvement.

b) Peut-on trouver une autre orientation du champ B (quel angle α par rapport à la verti-
cale ?) pour que le courant I soit plus petit que celui trouvé en a) ? Donner alors la valeur
de ce courant.

29.9 Série 9 : champ magnétique produit par des courants

Exercice 9.1
Deux fils rectilignes parallèles sont placés à 20 cm l’un de l’autre. Ils sont parcourus par
des courants de sens contraires, mais de même intensité égale à 20 A. Déterminer la di-
rection, le sens et l’intensité du champ magnétique qu’ils produisent aux points suivants,
tous situés sur une perpendiculaire les reliant :

a) à mi-distance entre les fils ;

b) en un point situé à 5 cm d’un fil et à 15 de l’autre ;

c) en un point situé à 10 cm d’un fil et à 30 de l’autre.

Exercice 9.2
On a deux courants parallèles, de même sens, de même intensité I et situés à une distance
d l’un de l’autre.

Calculer l’intensité du champ magnétique qu’ils produisent en un point situé à la distance


d de chacun des fils (résolution littérale).

Exercice 9.3
Au centre d’un solénoïde comportant 10 spires par cm et dont l’axe est orienté est-ouest,
se trouve une aiguille aimantée mobile autour d’un axe vertical.

Sachant que la composante horizontale du champ magnétique terrestre est de 0, 2 · 10−4


T, on demande de calculer l’angle dont tourne l’aiguille lorsque le courant traversant le
solénoïde passe de 0 à 20 mA.

Exercice 9.4
Un fil conducteur est bobiné de manière à former une succession de spires circulaires

154
dont les centres définissent un cercle. Une autre manière d’obtenir le même enroulement
consiste à prendre un solénoïde de longueur finie et à le courber de telle manière que les
spires des extrémités se touchent, l’ancien axe devenant un cercle. Le volume entouré par
les spires est un tore.

b
b

R1 b

O b

R2 r b

Lorsqu’un courant I circule dans le conducteur, on demande de déterminer la norme, la


direction et le sens du champ magnétique B ~ , produit dans le plan contenant les centres
des spires, à l’intérieur et à l’extérieur du tore. On donnera B~ en fonction de la distance r
du point considéré au centre O du tore. On appelle R 1 le rayon intérieur du tore et R 2 le
rayon extérieur. N est le nombre total de spires. La figure ne montre que 8 spires en coupe
avec le sens du courant.

29.10 Série 10 : flux, induction

Exercice 10.1
Une spire circulaire de rayon 8 cm est mobile dans un champ magnétique homogène de
0, 5 T.

Calculer le flux du champ magnétique qui traverse la spire lorsque les lignes de champ
font avec la normale à son plan un angle de : a) 90◦ ; b) 60◦ et c) 0◦ .

Exercice 10.2
~ . Calculer le flux de
Dans les quatre circuits, le courant I 1 produit un champ magnétique B
ce champ à travers le circuit (2).

a) b) c) d)
a l >> R 1
l >> R 1
I1
r R2 R1
I1 (2) 2R 2
b R1 I1
I1
(2)
N spires (2) N spires
(2)

155
Exercice 10.3
Un objet à 5 faces est placé dans un champ magnétique uniforme (orienté selon 0 y) d’in-
tensité B.

a
~
B

c y

b
x

Trouver le flux magnétique à travers chacune des faces de cet objet.

Exercice 10.4
Un wagonnet est posé sur des rails horizontaux et établit entre eux un contact électrique.
Ces rails, séparés par une distance l , sont reliés par une résistance R . Le système se trouve
placé dans un champ magnétique B ~ dont les lignes sont verticales et dirigées vers le haut.
Le wagonnet se déplace à une vitesse ~ v qu’on suppose constante.

l
R
~
B

x b

b
~
v

~ à travers la boucle, formée par la


a) Exprimer en fonction du temps le flux du champ B
résistance, les rails et le wagonnet.

b) Déterminer la tension et le courant électrique induits dans cette boucle (les résistances
des rails et du wagonnet sont négligeables devant R ).

~ (grandeur, direction, sens) exercée par le champ sur le wagonnet.


c) Déterminer la force F

~ ′ qui com-
d) Pour maintenir la vitesse constante, on exerce sur le wagonnet une force F
~ . Que vaut le travail de F
pense exactement F ~ pendant un intervalle de temps ∆ t ?

~′ ?
e) Quelle est la puissance mécanique fournie par l’agent qui exerce la force F

f) Que vaut la puissance électrique dissipée dans la résistance ?

156
g) Quels commentaires les résultats obtenus sous e) et f) suggèrent-ils ?

h) On change le sens de la vitesse du wagonnet. Indiquer brièvement les modifications que


cela apporte aux résultats précédemment établis ?

i) Comment les résultats précédents sont-ils modifiés lorsqu’on impose au wagonnet un


mouvement uniformément accéléré (accélération ~a) ?

Exercice 10.5
Une boucle de fil rectangulaire a une longueur a, une largeur b et une résistance R . Elle
est placée près d’un fil très long parcouru par un courant I (le fil est dans le plan de la
boucle). La distance entre le fil et le centre de la boucle est r .

~
v

b
r
a

a) Calculer le flux magnétique à travers la boucle. Indication :


Zr +b/2 µ ¶
1 r + b/2
dx = ln
r −b/2 x r − b/2

b) Déterminer le courant dans la boucle lorsqu’elle s’éloigne du fil à la vitesse ~


v (la vitesse
est perpendiculaire au fil et est dans le plan de la boucle).

Exercice 10.6
Considérons un champ magnétique uniforme B ~ , contenu à l’intérieur d’un long cylindre
~ est nul à l’extérieur du cylindre.
de rayon R , de direction parallèle à l’axe du cylindre. B
Supposons que l’intensité croisse linéairement en fonction du temps : B = kt, où k est une
constante positive.

a) On demande de déterminer la direction, le sens et la norme du champ électrique induit


~ ind en un point situé à une distance r de l’axe du cylindre (0 ≤ r < ∞).
E

b) Même question lorsque l’intensité décroît linéairement selon la relation : B = B0 − kt.

Exercice 10.7
Deux conducteurs verticaux et parallèles placés dans un champ magnétique B homogène,
sont reliés par une résistance R . Une barre CD , de longueur l et de masse m, glisse sans

157
frottement le long des conducteurs, fermant ainsi le circuit. Le champ magnétique est
perpendiculaire au plan du circuit. La résistance des conducteurs et de la barre est négli-
geable devant R . Au départ, la barre est lâchée sans vitesse initiale. On peut négliger le
flux du champ induit par rapport au flux du champ B.

l
C D

~
B
R

a) Déterminer le sens du courant induit.

b) En négligeant le ralentissement produit par la force de Laplace, déterminer la valeur


du courant I dans la résistance R lors des premiers mètres de chute. On donnera I en
fonction du temps ( t = 0 : début de la chute).

c) Lorsque la vitesse de la barre devient importante, la force de Laplace n’est plus négli-
geable. Montrer que cette vitesse va tendre vers une valeur limite que l’on calculera.

d) Plus difficile : déterminer l’intensité du courant I en fonction du temps t, pour chaque


instant t.

Exercice 10.8
Une boucle circulaire de conducteur est disposée dans un champ magnétique uniforme
parallèle à la normale à la boucle. L’intensité de B~ est de 3, 0 T. L’aire de la boucle se réduit
au taux de 0, 4 m2 /s (il s’agit d’un conducteur élastique).

Quelle est la tension induite dans la boucle ?

Exercice 10.9
Un solénoïde est constitué de Ns = 600 spires. Il a une longueur l = 30 cm et un diamètre
d s = 4 cm. Il est entouré en son centre par une bobine circulaire plate contenant Nb = 20
spires de d b = 10 cm de diamètre. Les axes de la bobine et du solénoïde sont confondus.

158
La bobine est reliée à un voltmètre (de résistance interne infinie) qui indique les valeurs
suivantes en fonction du temps t : U = 2 · 10−5 t pour 0 ≤ t < 5 s et U = 0 pour t > 5 s.

Comment varie au cours du temps le courant I dans le solénoïde ?

29.11 Série 11 : auto-induction

Exercice 11.1
Exprimer en termes de la constante de temps TL le temps qu’il faut pour que le courant
dans un circuit RL atteigne 99, 9% de sa valeur d’équilibre.

Rép. : 6, 91 TL .

Exercice 11.2
Dans un circuit RL, le courant augmente jusqu’au tiers de sa valeur finale en 5 s.

Trouver la constante de temps du circuit.

Rép. : 12, 3 s.

Exercice 11.3
Dans un circuit RL, le courant chute de 1 A à 10 mA dans la première seconde après qu’on
a retiré la pile du circuit.

Si L a une valeur de 10 H, trouver la résistance R du circuit.

Rép. : 46 Ω.

Exercice 11.4
A l’instant t = 0, une tension de 45 V est soudainement appliquée à une bobine pour la-
quelle L = 50 mH et R = 180Ω.

Quel est le taux de croissance du courant à t = 1, 2 ms ?

Rép. : 12 A/s.

Exercice 11.5
Une bobine est reliée en série à une résistance de 10 kΩ. Une pile de 50 V est raccordée

159
aux deux éléments et le courant atteint une valeur de 2 mA après 5 ms.

a) Trouver la self L de la bobine.

b) Quelle quantité d’énergie est emmagasinée dans la bobine au même instant ?

Rép. : a) 97, 9 H b) 0, 196 mJ.

Exercice 11.6
Le courant qui traverse une self de 4, 6 H et de résistance 12Ω varie en fonction du temps
comme suit : de 0 à 7 A entre 0 et 2 ms, puis de 7 A à 5 A entre 2 et 5 ms, puis de 5 A à 0
A entre 5 et 6 ms.

Déterminer la tension induite durant ces intervalles de temps.

Rép. : 16 kV ; 3, 1 kV ; 23 kV.

29.12 Série 12 : circuits RLC

Exercice 12.1
Soit une bobine dont la résistance vaut 12Ω et le coefficient L est de 35 mH. On la connecte
à un générateur dont la tension aux bornes est donnée par U ( t) = 2 sin(ω t).

a) Pour quelle valeur de la fréquence le courant efficace traversant la bobine sera-t-il de


50 mA ?

b) Quel sera alors le déphasage entre U et I ?

c) Que vaudrait Î pour une fréquence f = 1000 Hz ?

Rép. : a) f ≃ 116, 5 Hz ; b) φ ≃ 65◦ ; c) Î ≃ 9, 1 mA

Exercice 12.2
a) Quelle résistance faut-il placer en série avec un condensateur de 2µF pour que le cou-
rant soit en avance de 30◦ sur la tension lorsque la fréquence est de 1500 Hz ?

b) Même question, pour une résistance placée en parallèle avec le condensateur.

c) Quelle est l’impédance équivalente de chacun des circuits ci-dessus ?

160
Rép. : a) R ≃ 91, 9 Ω ; b) R ≃ 30, 6 Ω ; c) | z| ≃ 106 Ω et | z| ≃ 26, 5 Ω

Exercice 12.3
On branche un générateur donnant une tension sinusoïdale ( f = 500 Hz, Ueff = 5 V) aux
bornes d’une self de 5 · 10−2 H, de résistance négligeable.

a) Calculer le courant I eff .

b) Déterminer la capacité du condensateur qu’il faut placer en série avec la self pour que
le courant I eff augmente d’un facteur 3.

Rép. : a) I eff ≃ 32 mA ; b) C ≃ 3, 03 µF

Exercice 12.4
On place une self de 45 mH en série avec une résistance de 10 Ω. Le tout est en parallèle
avec un condensateur de capacité de 4 · 10−6 F et branché aux bornes d’un générateur a.c.
de fréquence 500 Hz.

Que vaut l’impédance du circuit ?

R 2 +ω2 L2
Rép. : | z| ≃ 180 Ω ou littéralement : | z| = p
R 2 +ω2 (ω2 L2 C +R 2 C −L)2

Exercice 12.5
Un appareil caractérisé par L = 0, 1 H et R = 8Ω (en série) est branché sur le réseau.
Cet appareil déplaît aux Services Industriels (S.I.) car il utilise un courant important, ce
qui chauffe inutilement la ligne d’alimentation (énergie non facturée au client), sans pour
autant consommer beaucoup d’énergie (petite facture !).

a) Calculer le module de l’impédance de l’appareil, le courant efficace et la puissance


consommée par l’appareil.

b) Pour diminuer autant que possible I eff , les S.I. suggèrent au client d’ajouter un conden-
sateur bien choisi, en parallèle avec l’appareil, de manière à rendre réelle l’impédance
équivalente du circuit. Trouver la capacité de ce condensateur et la nouvelle valeur de I eff
dans la ligne. Que dire de la facture des S.I. ?

p
2
Rép. :a) | z| = R 2 + (ωL)2 , I eff = Ueff /| z|, P̄ = Ueff cos φ/| z| avec cos φ = R /| z| ;
p
2 2
b) C = L/(R + (ωL) ) et I eff (nouveau) = I eff R / R 2 + (ωL)2 < I eff mais P̄ ′ = P̄ donc même

facture (le courant est plus faible, mais le déphasage est nul cette fois).

161
Exercice 12.6
Une génératrice a une tension alternative U ( t) = Û sin ω t avec Û = 25 V et ω = 377 rad/s.
Elle est branchée à une self de 12, 7 H et de résistance négligeable.

a) Quelle est la valeur maximale du courant ?

b) Quelle est la tension lorsque le courant est maximal ?

c) Quelle est l’intensité du courant lorsque la tension est de −12, 5 V et que cette tension
est croissante ?

Rép. : a) 5, 22 mA ; b) 0 ; c) −4, 51 mA

Exercice 12.7
Une génératrice a une tension alternative u( t) = Û sin(ω t − π/4), où Û = 30 V et ω = 350
rad/s. Le courant produit dans un circuit branché à cette génératrice est I ( t) = Î sin(ω t −
3π/4), où Iˆ = 620 mA.

a) A quel instant après t = 0, la tension de la génératrice atteindra-t-elle pour la première


fois sa valeur maximale ?

b) Idem, mais pour le courant.

c) Il n’y a qu’un seul élément branché à la génératrice. S’agit-il d’un condensateur, d’une
self ou d’une résistance ?

d) Quelle est la valeur de C , L ou R , le cas échéant ?

Rép. : a) 6, 73 ms ; b) 11, 2 ms ; c) une self ; d) 138 mH

Exercice 12.8
Une résistance de 10 Ω est en série avec un condensateur de 47 µF. On place une self de
45 mH en parallèle sur ces deux éléments. On ferme le circuit sur un générateur a.c. de
fréquence 1′ 000 Hz et de tension efficace 10 V.

a) Calculer l’impédance du circuit.

b) Calculer le déphasage entre la tension et le courant.

c) Calculer la puissance moyenne consommée par ce système.

Rép. : a) 10, 7 Ω ; b) −16, 7◦ ; c) 9 W

162
Exercice 12.9
On considère un circuit oscillant LC non-amorti dont le condensateur a une capacité C =
1, 5 µF et la bobine une self L = 12 mH. On charge d’abord le condensateur sous une tension
de 57 V, puis on ferme l’interrupteur.

a) Calculer le courant maximal qui passe dans la bobine.

b) Combien de temps après la fermeture de l’interrupteur le courant est-il le plus grand ?

Exercice 12.10
Dans le schéma ci-dessous, le condensateur C 1 = 900 µF est chargé sous une tension de
100 V alors que le condensateur C 2 = 100 µF est déchargé. La bobine a une self L = 10 H.

S1 S2

C2 L C1

Comment actionner les interrupteurs S 1 et S 2 au cours du temps pour charger au maxi-


mum C 2 ? Que vaut alors la tension à ses bornes ?

Exercice 12.11
On souhaite allumer une ampoule de lampe de poche dont les caractéristiques sont U0 =
3, 5 V pour I 0 = 0, 2 A (valeurs efficaces). Cette ampoule se trouve à l’extrémité d’une ligne
constituée d’un fil de constantan de 0, 2 mm de diamètre et de 2 m de long (aller-retour).
Un transformateur de rapport de transformation N2 / N1 = 3 est placé en début de ligne et
un autre de rapport N1 / N2 = 3 se trouve en fin de ligne (voir la figure).

a) Calculer la tension et la puissance (efficaces) à imposer au générateur de manière à ce


que l’ampoule s’allume normalement (taux de luminosité de 100%).

163
b) Calculer la puissance perdue dans la ligne.

On impose maintenant la même puissance au générateur, ce dernier est connecté à la


même ligne qu’avant mais elle est dépourvue cette fois de transformateurs.

c) Calculer la tension qu’il faut mettre au générateur pour obtenir la même puissance qu’à
la question a).

d) Calculer la tension aux bornes de l’ampoule.

e) Calculer (en %) le taux de luminosité de l’ampoule.

f) Calculer la puissance perdue dans la ligne.

Rép. : a) 4, 193 V et 0, 839 W b) 0, 139 W c) 6, 390 V d) 2, 297 V e) 43, 06 % f) 0, 538 W

164
30 Solutions des exercices

30.1 Solutions Série 1

Corrigé 1.1
On applique la loi de Coulomb. Chaque charge est repoussée par l’autre par la même force
F = 1 N, les charges
q étant q
les mêmes q 1 = q 2 ≡ q. Donc
| q|·| q| F d2 1·12
F=k d2
⇒| q |= k
= 9·109
= 1, 05 · 10−5 C ≃ 10µC.

Corrigé 1.2
| q|·| q|
Pour la gravitation : FG = G m1d·2m2 , pour l’électrostatique : FE = k d2 . On veut évaluer
combien de fois la force électrique est plus intense que la force gravitationnelle pour ces
k| q || q |
particules. On va calculer FFE = Gm11 m22 .
G

FE k| q 1 || q 2 | 9·109 ·(1,6·10−19 )2
a) FG = Gm 1 m 2 = 6,67·10−11 ·(9,11·10−31 )2
= 4 · 1042 ;
FE k| q 1 || q 2 | 9·109 ·(1,6·10−19 )2
b) FG
= Gm 1 m 2
= 6,67·10−11 ·(1,67·10−27 )2 = 1, 2 · 1036 ;
FE k| q 1 || q 2 | 9·109 ·(1,6·10−19 )2
c) FG = Gm 1 m 2 = 6,67·10−11 ·1,67·10−27 ·9,11·10−31 = 2, 3 · 1039.
Dans tous ces cas de figure, FG est parfaitement négligeable devant FE . C’est la raison
pour laquelle on peut oublier la force gravitationnelle en chimie, en physique nucléaire,
etc.

Corrigé 1.3
La masse de la boule de cuivre est m = ρ cu · V où V = 43 πR 3 est le volume de la boule (de
rayon R = 0, 1 m) et ρ cu = 8, 92 · 103 kg/m3 est la masse volumique du cuivre. On obtient
une masse de m = 4, 67 kg. Pour connaître combien il y a d’atomes de cuivre (et donc
d’électrons libres) dans la boule, il faut connaître la masse d’un atome de cuivre. Elle vaut
(voir tableau périodique des éléments) m a = 63, 546 [u] où l’unité de masse atomique est 1
[u] = 1, 66 · 10−27 kg. Par conséquent, m a = 1, 05 · 10−25 kg et le nombre n d’électrons libres
4,67
dans la boule est n = mma = 1,05·10−25 = 4, 43 · 1025 électrons libres.
La charge (positive) de la boule après extraction de tous les électrons libres est Q = 4, 43 ·
1025 · 1, 6 · 10−19 = 7, 1 · 106 C. Finalement, la force de répulsion entre deux telles boules
Q2 (7,1·106 )2
séparées de la distance Terre-Lune ( d = 3, 84 · 108 m) est FE = k d2 = 9 · 109 · (3,84·108 )2 =
3, 1 · 106 N (une force équivalente au poids d’une masse de 300 tonnes !).

Corrigé 1.4
Le proton ( p) est environ 1800 fois plus massif que l’électron ( e). On peut donc considérer
que p est fixe et que e lui tourne autour selon un MCU. La seule force F ~ qui agit sur
l’électron est d’origine électrostatique. Par l’équation de Newton
F = ma c , où m est la masse de l’électron et a c = ω2 R est son accélération centripète (R =
0, 53 · 10−10 m). On cherche le nombre de tours que e effectue par seconde, c’est-à-dire la

165
fréquence f = T1 (où T est la durée par tour, appelée aussi la période. Unité de f : [s−1 ] ≡
[Hz] pour hertz).
q
ω F q2
~v Comme ω = 2π · f ⇒ f = 2π avec ω = mR et F = k R2 . La charge
du proton et de l’électron sont les mêmes en valeur absolue ( q =
e b
1, 6 · 10−19 C). On r
obtient donc
q
~
F kq 2 9·109 ·(1,6·10−19 )2
p f = 21π mR3 = 21π 9,11·10−31 ·(0,53·10−10 )3 = 6, 6 · 1015 Hz. Dans ce mo-
dèle, l’électron effectue 6, 6 · 1015 tours par seconde.

Corrigé 1.5

~1
E a) x = AB = 16 cm et cos α = 0, 8 ⇒ α = 36, 9◦.
~
E Le champ E ~=E ~1 + E
~ 2 et ici E = E 1 + E 2 avec
~2
E kq 1
E 1 = E 2 = ( x/2)2 .
kq 9 −5
Donc E = 2 ( x/2)12 = 2 9(0
·10 ·10
,16/2)2
= 2, 8 · 107 N/C.
α ~1
E
A b ~
E b B
q 1 = 10µC ~2
E q 2 = −10µC

kq 1 kq 1
b) E = E 1 cos α + E 2 cos α = 2E 1 cos α car E 1 = E 2 = d2
avec d = 0, 1 m. Donc E = 2 d2
cos α =
9·109 ·10−5
2· 0,12
cos 36, 9 = 1, 44 · 107 N/C.

Corrigé 1.6
a) La charge de l’électron est − e ( e = 1, 6 · 10−19 C est la charge élémentaire) et sa masse est
m = 9, 11 · 10−31 kg. La force qu’il subit est F = eE ⇒ ma = eE ⇒ a = eE m
= 8, 79 · 1016 m/s2 . Il
p
s’agit ici d’un MRUA, donc v = 2ad = 5, 93 · 107 m/s (environ 60′ 000 km/s).

~
E
l = 1 cm
−e b
~ = −eE
F ~

~′
E
d = 2 cm
d ′ = 5 cm


b) Le temps de passage est (c’est un MRU horizontalement) t = dv = 8, 43 · 10−10 s. Or,
le mouvement vertical est un MRUA. Après la durée t, l’électron parcourt la distance
l = 0, 01 m donc l = 21 a′ t2 ⇒ a′ = 2t2l = 2, 81 · 1016 m/s2 . Connaissant l’accélération a′ , on
ma′
peut déterminer le champ E ′ qui a provoqué cette accélération à l’électron : E ′ = e =
1, 6 · 105 N/C.

166
30.2 Solutions Série 2

Corrigé 2.1
a) Le champ électrique E~ qui se trouve juste en-dessus de la surface (latérale) du cylindre
est perpendiculaire à l’élément de surface dS (voir la figure). Sa norme (E ) est la même
en chaque point (équidistant de l’axe du cylindre). Prenons comme surface de Gauss S
(surface fermée) un cylindre de rayon à peine plus grand (de manière infinitésimale) que
r et de hauteur h, contenant le tambour en question. Le calcul du flux Φ (à travers S ) du
champ électrique est facile à calculer vu la symétrie du problème.

~
E ~
E

r dS dS
r′

h′
h

Comme E ~ = E dS (car E
~ · dS ~ ∥ dS~ ) et que E est constant (en norme) sur toute la surface
latérale
H du cylindre S , on aH
~ ~
H
Φ = S E · d S = S E dS = E s dS = E (2πrh). Sur les disques de S , le flux est nul (car E = 0
sur ces disques, le tambour étant un cylindre creux). On applique maintenant la loi de
Gauss : (Q est la charge enfermée, autrement dit la charge nette du tambour)
Φ = ǫQ0 ⇒ Q = ǫ0 Φ = ǫ0 2πrhE = 8, 86 · 10−12 · 2π · 0, 06 · 0, 42 · 2, 3 · 105 = 3, 22 · 10−7 C = 0, 322µC.

b) On réduit la surface du tambour, mais le champ E doit rester le même qu’avant. Il faut
donc réduire la charge nette du nouveau tambour (Q ′ ). Comme
Q Q′ ′ ′
E = 2πǫ0 rh = 2πǫ0 r ′ h′ ⇒ Q ′ = Q rrhh = 0, 44 · Q = 0, 144µC.

Corrigé 2.2

~
E ~
E champ perpendiculaire On applique la loi de Gauss pour une surface
~ fermée (de Gauss) qui a la forme d’une capsule
~
dS E
(cylindre) dont la partie inférieure est située
h
dans le conducteur (là où E = 0) et la partie su-
dS
+ + + périeure en-dessus de la surface du conducteur.
+ + + Les charges (+ ici) que l’on a mises se placent à
charge à la surface + la surface du conducteur. La capsule a une hau-
E = 0 dans le conducteur teur dh et une surface dS (les couvercles) très
petites (infinitésimales).

Le flux d Φ (infinitésimal) du champ à travers la capsule n’est dû qu’au flux du champ sur
la surface supérieure. Comme E ~ , on obtient d Φ = EdS . La loi de Gauss affirme alors
~ ∥ dS
que la charge d q (infinitésimale) enfermée dans la capsule (à la surface du conducteur ici)
vaut d q = ǫ0 d Φ. Donc d q = ǫ0 EdS . Connaissant le champ E et la portion de la surface du

167
conducteur (d’aire dS ), on obtient donc la charge d q située sur cette portion de surface. Si
le champ E n’est pas trop inhomogène sur une certaine portion S de surface, la charge q
sur cette surface peut se calculer par q = ǫ0 ES . Avec l’application numérique :
q = 8, 86 · 10−12 · 104 · 5 · 10−6 = 4, 43 · 10−13 C.
dq
Remarque : Le résultat trouvé peut s’écrire : dS = ǫ0 E . On définit la densité superficielle de
dq
charge d’un conducteur (chargé) par l’expression σ ≡ dS . La grandeur σ (se dit sigma) se
mesure en [C/m2 ] et exprime la charge par unité de surface (m2 ) qui se trouve à la surface
·10−13
d’un conducteur chargé. Donc on a σ = ǫ0 E . Dans l’exemple : σ = 4,43 5·10−6
= 8, 86 · 10−8
C/m2 .

Corrigé 2.3
En joignant les deux hémisphères, la charge Q 2 se répartit de manière uniforme sur toute
la grande sphère. De même, la charge Q 1 est répartie de manière uniforme sur toute
la petite sphère. Le système possède la symétrie sphérique. Le champ E ~ est radial et a
la même intensité pour tous les points situés à égale distance du centre O des sphères
concentriques. En appliquant la loi de Gauss pour une surface fermée qui est une sphère
de centre O et de rayon r , on obtient facilement l’expression du champ partout dans l’es-
pace.
Pour r < R 1 , par Gauss : Φ = 0 car il n’y a pas de charge enfermée à l’intérieur de la petite
sphère. Donc E ( r ) = 0 pour r < R 1 .
Q
Pour R 1 < r < R 2 , par Gauss : Φ = ǫ01 car Q 1 est la charge enfermée (Q 2 se trouvant à
l’extérieur n’est pas enfermée). Or, le calcul de Φ est très simple vu la symétrie sphérique :
Q1
Φ = E 4π r 2 . Donc on den déduit E ( r ) = 4πǫ r2
pour R 1 < r < R 2 .
0
Q 1 +Q 2
Pour R 2 < r , par Gauss : Φ = ǫ0
car Q 1 + Q 2 est maintenant la charge enfermée. Or,
Q 1 +Q 2
comme avant, Φ = E 4π r 2 . Donc E ( r ) = 4πǫ0 r 2
pour R 2 < r .
Pour r = R 1 ou r = R 2 , E = 0 puisque le champ est nul à l’intérieur des conducteurs (lors-
qu’il n’y a pas de mouvement de charge).

Corrigé 2.4
La tension entre A et B est de 4, 5 V (U AB =
4, 5 V). Or, le champ électrique E ~ a la même
A B norme (E ) tout le long du fil métallique (sa
~
E ~
E
~
E section est toujours la même), donc U AB =
U 4,5 −3
El ⇒ E = AB l = 1′ 000 = 4, 5 · 10 V/m.
l
La force électrique (en norme) qui s’exerce sur chaque électron libre est F = eE et donc
l’accélération qu’il subit entre chaque collision est
F eE 1,6·10−19 ·4,5·10−3
a= m = m = 9,11·10−31
= 7, 91 · 108 m/s2 .

Corrigé 2.5
Par définition de la tension, on a :

168
~ ·−
−→ −−→
A b

U AB = E AB = E || AB || cos 20 = 15 · 0, 1 · cos 20 = 1, 41 V ;
40◦ UBC = E~ ·−
−→ −−→
BC = E || BC || cos 100 = 15 · 0, 1 · cos100 = −0, 26 V ;
20◦ ~ ·−
UC A = E
−→ −−→
C A = E || C A || cos 140 = 15 · 0, 1 · cos 140 = −1, 15 V.
b b

B
100 ◦ C On constate U AB + UBC + UC A = 1, 41 − 0, 26 − 1, 15 = 0 V (la
~
E tension sur un chemin fermé est nulle).

Corrigé 2.6
On applique le thm. de l’énergie cinétique. Le travail de la force électrique, WAB = (− e)U AB ,
est égal à la différence des énergies cinétiques. r
q
1 2 1 2 1 2 2(− e )U AB 2(−1,6·10−19 )·(−1)
(− e)U AB = 2 mvB − 2 mv A = 2 mvB + 0 ⇒ vB = m = 9,11·10−31
= 5, 93 · 105 m/s.
Pour que l’électron accélère en direction de B, il faut que U AB soit négatif ! Si la tension
entre les extrémités est de 1 V, cela signifie que c’est UBA = 1 V et donc U AB = −UBA = −1 V.

Corrigé 2.7

Un tel dispositif s’appelle un sélecteur de vitesse.


N
y
A la sortie du trou T2 tous les électrons doivent
~
E
d = 5 cm avoir la même vitesse v = 1′000 km/s. Ces élec-
x
trons avaient déjà cette vitesse lorsqu’ils se trou-
M
T1 T2
vaient dans le faisceau incident (car il s’agit ici
α
d’un problème de balistique puisque le champ E ~
x = 6 cm
est homogène : MRU selon Ox et MRUA selon O y).
faisceau incident Le faisceau sortant par T2 a le même angle α que
le faisceau incident.

La plaque M doit être positive et la plaque N négative pour que l’électron sorte par T2 . Le
champ E~ va donc dans le sens indiqué et U M N > 0.

Le temps t de passage de l’électron entre les plaques est (MRU selon Ox) : t = vxx = v cos
x
α
et
t
2 est le temps qu’il met pour parvenir au sommet. L’accélération (dirigée de N vers M ) peut
∆v y 0− v y 2 sin α cos α 6 )2 ·sin 60·cos60
se calculer a y = ∆t = t/2 = − 2v sin
t
α
= − 2v x = − 2·(10 0,06 = −1, 44 · 1013 m/s2 .

Connaissant l’accélération, on peut alors trouver la force que l’électron subit et donc le
champ électrique E entre les plaques :
−ma 9,11·10−31 ·1,44·1013
F y = − eE ⇒ ma y = − eE ⇒ E = e y = 1,6·10−19
= 82 V/m. La tension entre les
plaques vaut donc U M N = E d = 82 · 0, 05 = 4, 1 V. Une simple pile connectée aux plaques
suffit pour ce sélecteur de vitesse.

Corrigé 2.8
On applique la loi de Gauss en prenant comme surface de Gauss une sphère de rayon r
concentrique à la boule (pleine et isolante) chargée uniformément et dont la charge est
Q > 0. Par symétrie sphérique, le flux Φ du champ électrique est simple à calculer. On a :

169
~ ~= 2
H H H
Φ= S E · dS S E dS = E S dS = E 4π r .
~ et que E est constant (en norme) sur la sphère de Gauss.
~ ∥ dS
On a utilisé que E

S = surface de Gauss (sphère) a) La loi de Gauss dit que le flux Φ calculé est proportionnel
Q
boule pleine ~ à la charge enfermée dans la surface de Gauss : Φ = ǫenf. .
(isolante) E 0
r Lorsque r < R , la charge enfermée est plus petite que Q (la
b
charge totale de la boule). La charge qui se trouve dans la
R boule mais à l’extérieur de la surface de Gauss ne contribue
pas au flux Φ.

La fraction de la charge totale qui se trouve enfermée est simplement égale au rapport des
3 3 3
volumes : 44ππRr 3/3
/3
= Rr 3 . Donc Q enf. = Q Rr 3 . On a donc :
Q enf. Q 3 Q
E 4π r 2 = ǫ0
= ǫ0 Rr 3 ⇒ E = 4πǫ0 Rr3 . L’intensité croît linéairement avec r .

b) Lorsque r ≥ R , la charge enfermée est Q (Q enf. = Q ). Donc


Q Q Q
E 4π r 2 = ǫenf.
0
= ǫ0 ⇒ E = 4πǫ0 r12 . L’intensité décroît avec le carré du rayon.

Corrigé 2.9
mg 10−3 ·9,81
a) A l’équilibre, mg = qE ⇒ E = q = 10−6 = 9′ 810 V/m.
~
qE
y Donc la tension entre les plaques est U = E d = 9′ 810 ·

0, 05 = 490, 5 V.
~
E
+ b) On triple maintenant la tension, donc U = 1471, 5 V. Le
m~
g champ est triple aussi, il vaut E = 29′430 V/m.

qE −m g 10−6 ·29′ 430−10−3 ·9,81


Comme qE − mg = ma y ⇒ a y = m
= 10−3
= 19, 62 m/s2 (vers le haut).

Corrigé 2.10
a) 1 eV = eU pour U = 1 V. D’où, 1 eV = 1, 6·10−19 ·1 = 1, 6·10−19 J. Donc 1 MeV = 1, 6·10−13 J.
r
−12 −12 1 2·3,2·10−12
b) 20 MeV = 3, 2 · 10 J. Donc 3, 2 · 10 = 2 mv ⇒ v = 9,11·10−31 = 2, 65 · 109 m/s. Ce qui
2

est impossible car alors v > c (la vitesse de la lumière) ! Il y a une faute quelque part... La
mécanique classique n’est plus applicable pour décrire les électrons à ces énergies-là ! Il
faut utiliser la relativité restreinte.

Corrigé 2.11

y cm
a) Le champ électrique est perpendiculaire aux lignes
50 équipotentielles et les lignes de champ E~ sont orientées
ligne de champ
~
E dans le sens des potentiels décroissants (on ≪ descend
30 C équipotentielle
b

b
B les potentiels ≫ en suivant une ligne de champ).
10 b
A
0 10 30 50 x cm

170
En effet, prenons deux points A et B situés sur une ligne de champ (voir figure). Ici,
U AB = VA − VB = 10 − 20 = −10 V (tension négative). Or, U AB = E ~ ·−
−→
AB. Par conséquent,
~ est dans le sens opposé au vecteur −
E
−→
AB. La ligne de champ est donc orientée comme
indiqué. Les lignes de champ coupent l’axe Ox à 45◦ (et donc aussi l’axe O y). Les équipo-
tentielles étant également espacées, le champ électrique est constant en norme.
Pour voir qu’une ligne de champ traverse perpendiculairement une équipotentielle, consi-
dérons le point C sur la même équipotentielle que B et très proche (infiniment proche) de
~ ·−
B. On a donc UBC = VB − VC = 0. Or, UBC = E
−→ ~ et −
BC , donc E
−→
BC sont perpendiculaires.
−−→
b) La distance entre A et B vaut || AB ||= 20cm · cos 45 = 14, 14 cm.
−−→ U AB
Comme U AB = E || AB || cos 180 ⇒ E = − −−→ = 0,10
141 = 70, 9 V/m. Finalement, E x = E y =
|| AB||
−E cos 45 = −70, 9 · cos45 = −50, 1 V/m.

30.3 Solutions Série 3

Corrigé 3.1
a) On calcule d’abord la capacité C du condensateur plan : C = ǫ0 Sd avec S = πR 2 = π · 0, 12
est l’aire d’une plaque et d = 0, 024 m est la distance entre les plaques. Donc
π·0,12
C = 8, 86 · 10−12 0,024 = 1, 16 · 10−11 F. La charge accumulée sur une plaque est Q = CU =
1, 16 · 10−11 · 12 = 1, 39 · 10−10 C.
b) On réduit d de 24 mm à 1 mm sans changer Q puisque les plaques ne sont plus reliées
au générateur. Donc C devient 24 fois plus grande alors que la tension, elle, devient 24 fois
Q
plus petite (U = C ), donc U = 0, 5 V.

Corrigé 3.2
Par Gauss, calculons le champ E ~ (qui est ra-
dial) entre les deux cylindres. Pour surface
vue de dessus de Gauss, on prend un cylindre de rayon r
2R 2 = 20, 1 cm
− compris entre R 1 et R 2 (R 1 < r < R 2 ) et de
~
E hauteur h (coaxial aux cylindres). Le flux Φ
+
vaut :
h = 30 cm Φ = ES avec S = 2πrh ⇒ E = 2πǫQ0 h 1r avec Q
la charge enfermée. Elle est positive et est
~ pointe vers
répartie sur le petit cylindre (E
2R 1 = 19, 9 cm l’extérieur dans la figure).

Calcul approximatif : On voit que le champ E décroît avec r . Cependant, R 1 et R 2 sont


très proches l’un de l’autre et E ne varie pas beaucoup entre les cylindres. On le considère
constant E 0 (valeur du champ pour r = R̄ ≡ (R 1 + R 2 )/2 = 10 cm). La tension U entre les
cylindres est U = E 0 d avec d = R 2 − R 1 = 0, 1 cm la distance entre les cylindres. Donc
Q Q 2π·8,86·10−12 ·0,3·0,1
C = U = E 0 d = 2πǫd0 hR̄ = 0,001 = 1, 67 · 10−9 F = 1, 67 nF.

171
Calcul exact : il faut calculer la tension U en prenant en compte que le champ décroît avec
r . Par
RRla définition
R : ³ ´
~ · d~r = R2 E dr = Q
RR 2 1 Q R2 Q
U = R12 E R1 2πǫ0 h R1 r dr = 2πǫ0 h ln R1 . Avec C = U , on a
2πǫ
³ 0 h´ 2π·8,86·10−12 ·0,3
C= R2 = ln(20,1/19,9) = 1, 67 · 10−9 F = 1, 67 nF. (Le calcul approché est exact au cen-
ln R1
tième).
Si R 1 diffère peu de R 2 , la capacité d’un condensateur cylindrique peut être approchée par
celle d’un condensateur plan (comme si l’on déroulait les cylindres). C’est généralement ce
qui se passe. En électronique, les condensateurs sont fabriqués à l’aide de 2 feuilles conduc-
trices isolées l’une de l’autre (par des feuilles isolantes) que l’on enroule sur elles-même.
D’où une grande surface (donc grande capacité) dans un petit volume (peu d’encombre-
ment).

Corrigé 3.3
Q 10−7
a) La tension entre les plaques vaut U = C = 44··10 ′
−10 = 1 000 V. Considérons le cas où le
proton (chargé +) entre par la plaque +. Il se fait expulser vers l’autre plaque, son énergie
cinétique augmente. Le travail de la force électrique est W = eU > 0. Par le thm. de l’éner-
gie cinétique (au départ, v0 = 4 · 105 m/s)
r :
q
2·1,6·10−19 ·103
eU = 21 mv2 − 12 mv02 ⇒ v = v02 + 2meU
= (4 · 105 )2 + 1,67·10−27 = 5, 93 · 105 m/s.

b) Le proton est maintenant envoyé dans le condensateur par la plaque −. Il se fait donc
freiner. Le travail de la force électrique vaut W = − eU (le champ E ~ et la force F
~ = eE
~ sont
opposés cette fois au déplacement
q du proton).
p p
− eU = 21 mv2 − 12 mv02 ⇒ v = v02 − 2m eU
= 1, 6 · 1011 − 1, 92 · 1011 = −3, 2 · 1010 ⇒ impossib-
le. Cela signifie que le proton n’arrive pas à atteindre la plaque +. Il va alors rebrousser
chemin et ressortir par la plaque −. Calculons l’endroit entre les plaques où le proton re-
brousse chemin. Il s’arrête à une certaine distance d ′ (de la plaque −) qui correspond à
une tension U ′ (entre le point d’arrêt et la plaque −). On a
− eU ′ = 0 − 21 mv02 ⇒ U ′ = 21e mv02 = 835 V. La tension entre les plaques étant de 1′ 000 V,
cela signifie que le proton s’arrête aux 1835′ 000 = 0, 835 = 83, 5% de la distance qui sépare les

plaques du condensateur (en partant de la plaque −).

Corrigé 3.4

La pile de U = 10 V est reliée aux condensateurs par


Q2 > 0
des câbles conducteurs. Une répartition des charges
Q1 > 0 se produit sur les plaques des condensateurs (de ma-
C2
C4
C3
C1
nière quasi-instantanée) et un équilibre s’établit. Un
+
− U = 10 V champ électrique différent est engendré entre les arma-
C5
tures des condensateurs. En revanche, dans les câbles
le champ est nul.

On cherche Q 1 > 0 et Q 2 > 0 les charges portées par les armatures (+) des condensateurs
1 et 2 respectivement. Pour Q 1 c’est facile car U = 10 V est aussi la tension aux bornes de

172
C 1 . Donc Q 1 = C 1U = 10−5 · 10 = 10−4 C = 100µC. Calculons les capacités équivalentes :
1
C23
= C1 + C1 ⇒ C 23 = 5µF et C 234 = C 23 + C 4 = 15µF. D’où, C 1 = C 1 + C1 ⇒ C 2345 = 6µF.
2 3 2345 234 5
L’armature négative de C 2345 est la même que celle de C 5 . Donc Q 5 = C 2345U . En effet, car
la tension aux bornes de C 2345 est aussi la tension U = 10 V. On obtient donc Q 5 = 60µC.
Ainsi, la tension aux bornes de C 5 est U5 = Q 5
C5
= 6 V. On en déduit que la tension aux bornes
de C 23 (et également aux bornes de C 4 ) vaut U23 = 10 − 6 = 4 V (U4 = 4 V). Or, la charge
sur C 23 est la même que la charge sur C 2 . Donc Q 2 = C 23U23 = 20µC et Q 3 = Q 2 = 20µC.
Finalement Q 4 = C 4U4 = 40µC.
En résumé : Q 1 = 100µC ; Q 2 = 20µC ; Q 3 = 20µC ; Q 4 = 40µC ; Q 5 = 60µC.
Le résultat est bien cohérent car la somme des charges des armatures négatives de C 3
et C 4 doit être la même (en valeur absolue) que la charge de l’armature positive de C 5
(20 + 40 = 60).

Corrigé 3.5
On charge le condensateur C 1 (Q sur
avant après une armature et −Q sur l’autre). On
C1 C1 le branche ensuite à un second conden-
~
E ~1
E sateur (C 2 ) non chargé via un double-
interrupteurs (d’ailleurs un seul interrup-
Q Q1
teur aurait suffi. Pourquoi ?). Au départ,
double interrupteurs
Q2 l’énergie contenue dans C 1 vaut W1 =
1
C U 2 où U = CQ1 est la tension appli-
2 1
~2
E quée aux bornes de C 1 pour le charger.
C2 C2 Q2
W1 = 2C1 .

En fermant l’interrupteur, une nouvelle répartition des charges se fait sur les deux conden-
sateurs : Q 1 sur l’un et Q 2 sur l’autre, mais Q = Q 1 + Q 2 (aucune charge ne quitte le sys-
tème). La tension aux bornes de chaque condensateur est la même et vaut maintenant U ′
que l’on peut déterminer immédiatement car la capacité de l’ensemble est C = C 1 + C 2 .
Q Q
Donc U ′ = C1 +C2 qui est plus petite que U = C1 . L’énergie contenue dans l’ensemble des
Q2
condensateurs vaut W2 = 2(C1 +C2 )
. On constate que W2 < W1 . L’énergie perdue vaut donc
Q2 C
Wperte = W1 − W2 = 2C1 (C1 +2 C2 ) . Lors de la fermeture de l’interrupteur, un mouvement de
charges s’est produit (un courant). Ce déplacement a engendré un échauffement des fils de
connexion dont l’énergie thermique dissipée est égale à Wperte .

Corrigé 3.6
a) Si l’écart entre les plaques double, alors la capacité est divisée par 2 et donc la tension
double. La tension vaut donc 200 V.
b) On a vu (Ex. 2.2) que le champ s’exprime par E = ǫσ0 où σ désigne la densité de surface
de charge (en C/m2 ). σ ne change pas lors du processus, donc le champ E non plus. Ceci
n’est vrai que si le champ peut être considéré comme homogène. C’est bien le cas tant que
l’espacement entre les plaques est petit par rapport à la taille des plaques.

173
2
c) L’énergie s’exprime par W = 12 CU 2 = 12 C (Ed )2 = E2 Cd 2 ( d est la distance entre les
plaques et E le champ qui ne change pas). Comme C est divisé par 2 et que d 2 est multiplié
par 4, l’énergie W est donc doublée. Il faut fournir de l’énergie pour séparer les plaques
d’un condensateur car ces dernières s’attirent.

30.4 Solutions Série 4

Corrigé 4.1
Au départ, la charge du condensateur est Q 1 = CU = 4 · 10−6 · 100 = 4 · 10−4 C. Après ∆ t = 20
s, la charge vaut Q 2 = 4 · 10−6 · 99, 5 = 3, 98 · 10−4 C. Une charge égale à ∆ q = Q 1 − Q 2 =
0, 02 · 10−4 C s’est déplacée d’une plaque à l’autre durant ∆ t, d’où un courant moyen de
∆q 0,02·10−4
I= ∆t = 20 = 10−7 A.

Corrigé 4.2
Un champ électrique E = 6 V/m règne dans le fil dont la résistivité est ρ = 3 · 10−7 Ω m.
Comme U = EL (L est la longueur du fil) et que l’on a aussi (loi d’Ohm) U = ρ SL I , on en
−6
déduit I = ES
ρ
= 63··10
10−7
= 20 A.

Corrigé 4.3
La masse volumique du Cu est ρ m = 8, 94 · 103 kg/m3 et sa résistivité est ρ = 1, 68 · 10−8Ωm.
On connaît la masse m = 200 kg et la résistance R = 0, 64Ω du câble de cuivre. On cherche
sa section s et sa longueur l .rDonc m = ρ m sl et R = ρ l / s ⇒
200·1,68·10−8
q

m = ρ m Rs2 /ρ ⇒ s = ρ m R = 8,94·103·0,64 = 2, 42 · 10−5 m2 = 24, 2 mm2 .
La longueur l = ρm = 8,94·103200
·2,42·10−5
= 924, 4 m.
ms

Corrigé 4.4
Q −3 Q −3
a) U = C = 10
10−6
= 103 V ⇒ I = U
R = RC = 1010
6 ·10−6 = 10
−3
A.
b) On voit sous a) que I est proportionnel à la charge Q qui reste sur le condensateur.
Comme Q diminue (de façon exponentielle, voir cours) donc I diminue aussi (de façon
exponentielle).
c) ∆Q = I ∆ t = 10−3 · 10−2 = 10−5 C. Il reste sur le condensateur une charge Q de Q =
10−3 − 10−5 = 9, 9 · 10−4 C et la tension aux bornes de ce condensateur est maintenant
Q 9,9·10−4
U = C = 10−6 = 990 V.

Corrigé 4.5
On désigne par U1 la tension aux bornes de la résistance de 1Ω et par I 1 le courant qui
traverse cette résistance. De même pour les résistances de 2, 3, 4 et 5Ω. On applique la loi
d’Ohm (U = R I ) et les lois de Kirchhoff successivement.

174
4Ω U3 = 3 · 12 = 36 mV ⇒ U1 = 36 mV ⇒ I 1 = 36 1 = 36 mA
1Ω ⇒ I 2 = 36+12 = 48 mA ⇒ U2 = 2·48 = 96 mV ⇒ U4 = 96+
2Ω 5Ω 36 = 132 mV. Donc, I 4 = 132 = 33 mA ⇒ I 5 = I 1 + I 3 + I 4 =
3Ω 4
48+33 = 81 mA et donc U5 = 5·81 = 405 mV. Finalement,
la tension aux bornes du générateur est U = U4 + U5 =
− +
132 + 405 = 537 mV.
U

Corrigé 4.6

10Ω Les câbles de connexion étant considérés comme idéaux


6Ω (résistance nulle et donc pas de tension entre deux
2Ω 4Ω points d’un même câble), le potentiel V est le même en
A b b
B C b b
D
40Ω chaque point d’un câble reliant deux résistances. Tous
12Ω les câbles qui se rejoignent au point A sont par consé-
+ − b
quent au même potentiel (idem pour les points B, C , D
O
1V et O).

On décide de poser égal à zéro le potentiel au point O (VO = 0 V). Aux bornes de la ré-
sistance de 40Ω la tension étant nulle (pas de différence de potentiel), aucun courant ne
la traverse et, par conséquent, elle ne joue pas de rôle. Le courant passe par le câble de
connexion qui relie D et O. D’ailleurs le potentiel VD est le même que VO , donc VD = 0 V.
D’autre part VA = 1 V puisque la tension au générateur est 1 V (VA − VO = 1 ⇒ VA = 1 + VO =
1 V). La tension U10 (aux bornes de la résistance de 10Ω) vaut 1 V, donc I 10 = 0, 1 A. La ré-
1
sistance équivalente dans la branche entre A et D vaut R = 2 + 4 + ( 12 + 16 )−1 = 10Ω. Comme
U AD = 1 V (car VA − VD = 1 − 0 = 1 V), le courant dans cette branche vaut I = 1/10 = 0, 1 A.
On en déduit donc I 2 = I 4 = 0, 1 A et aussi U2 = 2 · 0, 1 = 0, 2 V ⇒ VB = VA − 0, 2 = 0, 8 V. De
même, U4 = 4 · 0, 1 = 0, 4 V ⇒ VC = VD + 0, 4 = 0, 4 V. Finalement U6 = U12 = UBC = VB − VC =
0,4 0,4
0, 8 − 0, 4 = 0, 4 V, on a I 6 = 6 = 0, 067 A et I 12 = 12 = 0, 033 A.

Corrigé 4.7
Dans un circuit où il y a plusieurs sources
de tension, on ne sait pas a priori dans quels
10 V sens vont les courants. Il faut utiliser les
2Ω 10Ω I 1 − I 2 deux règles de Kirchhoff pour déterminer
+ les courants avec leurs sens. On place des
6Ω − I1 − I2 − I3
V sens aux courants (arbitrairement) et l’on
1 0Ω +
I1 I2 2 − 2Ω exploite directement la 1ère loi de Kirchhoff
− + − + (loi des noeuds). Dans notre cas, 3 courants
I1 I3 sont indépendants.
10 V 40 V
2Ω 4Ω
Il y a 3 mailles indépendantes auxquelles on attribue un sens de parcours (arbitrairement
également). Il y aura 3 équations (1 par maille) pour les 3 inconnues : I 1 , I 2 et I 3 . Par la
deuxième règle de Kirchhoff (loi des mailles) : (en suivant les sens de parcours)
1) −10 + 2 · I 1 + 6 · I 2 = 0
2) 10 + 2 · ( I 1 − I 2 ) − 6 · I 2 + 2 · ( I 1 − I 2 − I 3 ) − 10 + 10 · ( I 1 − I 2 ) = 0

175
3) −40 + 4 · I 3 + 2 · I 3 + 10 − 2 · ( I 1 − I 2 − I 3 ) = 0
Après simplifications, chaque équation s’écrit :
1) I 1 + 3 · I 2 = 5
2) 7 · I 1 − 10 · I 2 − I 3 = 0
3) − I 1 + I 2 + 4 · I 3 = 15
De 2), on tire I 3 = 7 · I 1 − 10 · I 2 que l’on remplace dans 3). On résout ensuite le système de
2 équations pour I 1 et I 2 qui ne présente aucune difficulté.
On obtient finalement : I 1 = 2 A, I 2 = 1 A et I 3 = 4 A. Les solutions étant toutes positives,
cela signifie que les courants vont bien dans le sens qu’on leur avait attribué a priori.

Corrigé 4.8
On résout comme à l’ex. 4.7. Il y a 2 équations (associées
aux 2 mailles) pour les 2 courants I 1 et I 2 . La 1ère loi
de Kirchhoff est déjà exploitée. On a donc
I2 5Ω I1 1) 20 − 2 · I 2 − 4 · ( I 1 + I 2 ) = 0
20 V 4Ω 7V 2) −7 + 5 · I 1 + 4 · ( I 1 + I 2 ) = 0
I1 + I2 Après simplifications :
2Ω
1) 2 · I 1 + 3 · I 2 = 10
2) 9 · I 1 + 4 · I 2 = 7

En multipliant 1) par 4 et 2) par −3 puis en additionnant les équations obtenues, on a :


8 · I 1 − 27 · I 1 = 40 − 21 ⇒ −19 · I 1 = 19 ⇒ I 1 = −1 A. Puis dans 2), on obtient : I 2 = 4 A.
Le courant I 1 va dans le sens opposé à celui indiqué et vaut 1 A. Le sens de I 2 est celui
indiqué.

30.5 Solutions Série 5

Corrigé 5.1
108 4,5
L’énergie consommée = 24 = 4, 5 kWh = 1, 62 · 107 J. La puissance P = 3 = 1, 5 kW =
3 P 1′ 500
1, 5 · 10 W. Comme P = U I ⇒ U = I = 5 = 300 V.

Corrigé 5.2
2 2 2 U ′2 122
P UR ⇒ R = UP = 220 ′
60 = 806, 7 Ω. Pour l’ampoule de phare de voiture, on a R = P = 60 =
2, 4 Ω. On voit que R ′ < R car le diamètre de R ′ est plus gros que celui de R pour une
meilleure résistance au choc.

Corrigé 5.3
L’énergie utile pour chauffer de l’eau : E utile = m · c · ∆T = 1 · 4180 · 80 = 334′400 J. c est
la chaleur massique de l’eau et ∆T l’écart de température. Le rendement est de η = 0, 7
E
(70%), donc l’énergie consommée vaut E cons = utileη
= 4, 78 · 105 J. La puissance de l’appareil
E cons 4,78·105
est alors de P = t = 300 = 1′ 590 W. Finalement, la résistance de la plaque est de
2 2
R = UP = 1220
′ 590 = 30, 4 Ω.

176
Corrigé 5.4
Les ampoules L 1 et L 2 se branchent directement sur
le générateur car elles supportent une tension de 220
R V. La puissance qu’elles développent alors est P1 = 66
W pour L 1 et P2 = 110 W pour L 2 . Pour qu’une ten-
U = 220 V G L1 L2 L3 sion de 150 V apparaissent aux bornes de L 3 (tension
maximale supportée), il faut alors placer une résis-
tance R en série avec L 3 pour faire chuter la tension
de 220 V à 150 V.

Donc, la tension aux bornes de R doit être de UR = 220 − 150 = 70 V. Le courant qui circule
P3 75
dans L 3 est I 3 = U 3
= 150 = 0, 5 A et c’est le même courant qui traverse la résistance R .
UR
70
Cette dernière vaut donc R = 0,5 = 140 Ω. Finalement, la puissance
I3 = que débite le
générateur est P = P1 + P2 + P3 + R I 32 = 66 + 110 + 75 + 140 · 0, 52 = 286 W.

Corrigé 5.5

Une pile de tension électromotrice Uem et de résis-


pile Uem r tance interne r est connectée à une résistance R . La
tension U aux bornes de la pile est U = Uem − rI et,
par la loi d’Ohm, R I = Uem − rI . Donc
1) 3 · 0, 3 = Uem − r · 0, 3
R 2) 15 · 0, 1 = Uem − r · 0, 1.

On résout ce système pour Uem et r et l’on obtient : Uem = 1, 8 V et r = 3 Ω.

Corrigé 5.6
a) U = Uem − rI et le courant maximal I max est débité par la pile lorsque la tension à ses
bornes (U ) est nulle. Donc 0 = Uem − rI max ⇒ I max = Urem = 26 = 3 A.
Uem 6
b) R I = Uem − rI ⇒ I = R +r = 10+2 = 0, 5 A. Donc l’énergie thermique dissipée par la résis-
tance R pendant t = 60 s est E th. = R I 2 t = 10 · 0, 52 · 60 = 150 J.

Corrigé 5.7
Le moteur M reçoit une puissance électrique Pélec
qu’il transforme partiellement en puissance mé-
bilan de puissance

canique Pméc (le reste est perdu sous forme ther-
Pélec Pméc Pméc Putile
M ··· P mique : dissipation par effet Joule due aux cou-
rants). Une deuxième cause de perte thermique
Ptherm Ptherm Ptherm provient des frottements mécaniques à l’intérieur
du moteur (engrenage, etc.).

m gh 56·9,81·7,5
a) La puissance utile délivrée par la pompe est Putile = t
= 60
= 68, 67 W.

′ Putile
b) La puissance mécanique à la sortie du moteur (et à l’entrée de la pompe) est Pméc = η1

avec η 1 = 0, 9 (10% de perte d’origines diverses dans la pompe). Donc Pméc = 76, 3 W.

177
c) La puissance mécanique produite par le moteur (avant les pertes dans les engrenages)
P′
est Pméc = ηméc
2
avec η 2 = 0, 8 (20% de pertes à ce niveau). Donc Pméc = 95, 4 W. Cette puis-
sance mécanique est directement liée à la tension contre-électromotrice Ucem du moteur
et de son courant de fonctionnement I = 0, 8 A : Pméc = Ucem I ⇒ Ucem = PmécI = 119, 3 V. La
tension aux bornes du moteur est plus élevée, elle vaut U = 140 V. La résistance interne r
du moteur est responsable de cet écart. On a U = Ucem + rI ⇒ r = U −UI cem = 25, 9 Ω.

d) L’énergie thermique globalement perdue dans le moteur pendant 5 minutes ( t = 300 s)


est due à l’effet Joule d’une part : rI 2 t et aux frottements dans les engrenages d’autre part :

(Pméc − Pméc ) t. On a : E th. = r · I 2 · t + (Pméc − Pméc

) · t = 4′973 + 5′730 = 10′703 J = 10, 7 kJ.

e) Si la pompe est bloquée et que le moteur ne tourne plus, alors aucune puissance mé-
canique n’est transformée et, par conséquent, Ucem = 0. La tension aux bornes du moteur
U = 140 V se trouve directement aux bornes de la résistance interne r = 25, 9 Ω. Un cou-
140
rant beaucoup plus grand en résulte I ′ = Ur = 25 ,9 = 5, 41 A (au lieu de 0, 8 A) et les fils
surchauffent (voire fondent). L’énergie thermique dissipée devient E ′th. = rI ′ 2 t = 227 kJ
(soit environ 21 fois plus qu’en condition normale de fonctionnement).

Corrigé 5.8
Uem
a) Les résistances R et r étant en série, on a I = R +r . La puissance thermique dissipée
dans le rhéostat est P = R I 2 = Uem
2
· (R+Rr )2 .
b) Cette puissance P dépend de la résistance variable R . On dérive P comme fonction de
R et l’on impose à cette dérivée de s’annuler. On trouve alors le R qui rend P extrémale
(maximale ici).
2 2 2
dP
dR = Uem · 1·(R+r()R−+2r·)(4R+r )·R = Uem · (rR− R
+r )4
⇒ dP
dR = 0 ⇒ R = r.
La puissance est maximale si la résistance du rhéostat est la même que la résistance
interne de la pile. La tension de fonctionnement est alors U = U2em et le courant débité par
la pile est la moitié de son courant maximal.

30.6 Solutions Série 6

Corrigé 6.1
On veut que la plus grande partie du courant soit
déviée (passe dans R plutôt que dans l’ampère-
I i r mètre) afin de mesurer des courants plus impor-
A tants (ici : Ii = 100) sans endommager l’instru-
ment. La résistance équivalente de r et R (en pa-
R rallèle) est RRr
+r . Par la loi d’Ohm :

R ·r
r·i = R +r · I ⇒ Ii = R +r
R ⇒ 100 = R +r
R
r
⇒ R = 99 = 1, 01 Ω.

178
Corrigé 6.2

a)

i r R U AB = (R + r ) · i = (1000 + 5) · 0, 1 = 100, 5 V est la tension


b
A b

maximale.
A B

b)
Un bon voltmètre ≪ prélève ≫ un très petit courant pour
R′ faire sa mesure. Il faut donc que I 2 soit donc très petit
I A I1 b
B
b

par rapport à I 1 . Pour comparer I 2 à I 1 , on calcule U AB


I2
r R
de deux manières différentes. Soit par R ′ · I 1 , soit par
A (R + r ) · I 2 . On a donc

I2 R′ 500
R ′ · I 1 = (R + r ) · I 2 ⇒ I1 = R +r = 1′ 000 +5 ≃ 0, 5. C’est donc un mauvais voltmètre.

Corrigé 6.3
1
Comme pour l’ex. 6.1 (même schéma), on a : i = RR+r · I ⇒ RR+r = Ii = 50 . D’autre part, la
Rr
résistance équivalente de R et r qui est R+r a pour valeur 0, 2 Ω. Donc
0, 2 = RRr
+r
r
= 50 ⇒ r = 10 Ω et donc 50R = R + 10 ⇒ R = 10
49
= 0, 204 Ω.

Corrigé 6.4
a) Dans le cas d’instruments idéaux, (résistance interne infinie pour le voltmètre et nulle
12
pour l’ampèremètre), on mesurerait : U = 12 V et I = 125 = 0, 096 A = 96 mA.

b)
On désigne par r A = 10 Ω et r V = 1′000 Ω les résistances in-
rA ternes de l’ampèremètre et du voltmètre respectivement. Ces
A G résistances sont en série avec les instruments. Donc r V et R
rV forment un groupement en parallèle et r A est en série avec
V ce groupement. La résistance équivalente du circuit est R équ =
R r A + rr V+·RR = 121, 1 Ω.
V
12
Le courant I qui sort du générateur vaut I ′ = 121

,1 = 0, 099 A = 99 mA. La tension aux

bornes de l’ampèremètre étant r A · I , la tension UR mesurée par le voltmètre aux bornes
de la résistance R vaut UR = 12 − r A · I ′ = 11, 01 V (au lieu de 12 V dans le cas idéal). Le
courant mesuré par l’ampèremètre est I ′ = 99 mA (au lieu de 96 mA dans le cas idéal) et
U
le courant qui passe dans la résistance est I R = RR = 88 mA. Il y a donc 8 mA qui passe
dans le voltmètre.

179
30.7 Solutions Série 7

Corrigé 7.1
On applique ici la force de Lorentz qu’exerce le champ B = 10−6 T sur l’électron de charge
− e et de masse m = 9, 11 · 10−31 kg. La vitesse de l’électron v = 106 m/s est perpendiculaire
au champ, donc la trajectoire est un MCU (de rayon R ) et donc l’accélération est purement
~ ||=|| (− e)~
centripète. Il vient : (vu que || F ~ ||= evB sin 90 = evB)
v∧B
v2 mv
evB = m R ⇒ R = eB = 5, 69 m.

Corrigé 7.2
a) La masse m d’une particule α est m = 6, 644 · 10−27 kg et sa charge est 2 e. Comme pour
2
l’ex. 7.1, on a 2 evB = m vR ⇒ v = 2 eBR
m
= 2, 59 · 106 m/s.
b) Le temps T qu’elle met pour faire un tour (c’est un MCU) est T = 2πvR = 1, 09 · 10−7 s.
c) Son énergie cinétique vaut E c = 12 mv2 = 2, 24 · 10−14 J = 1, 4 · 105 eV = 0, 14 MeV.
d) E c = 2 eU où U est la tension (différence de potentiels) à appliquer.
0,14 MeV 0,14 MV
Donc U = E2 ec = 2e = 2 = 0, 07 MV = 70′ 000 V (ou 70 kV).

Corrigé 7.3
Dans un spectromètre de masse (voir le cours), les ions de charges q > 0 (ici) sont d’abord
accélérés sous une tension U = 1′ 000 V, puis déviés selon une trajectoire circulaire par un
champ B = 0, 1 T. La vitesse des ions à la sortie du canon se calcule ainsi : qU = 21 mv2 ⇒ v =
q q
2 qU v2 2 mU
m . Comme qvB = m R , on en déduit le rayon de la trajectoire : R = qB2
. Dans notre
cas, les ions sont deux isotopes du manganèse : Mn55 et Mn56 doublement ionisés ( q = 2 e).
Les masses de ces ions sont m55 = 55 · u et m56 = 56 · u. Les électrons étant environ 1′ 800
fois plus légers que les protons et les neutrons, on peut ici négliger leur masse devant celle
des noyaux.
q On a donc q q
R 55 = 22meB
55 U
2 = 0, 2396 m et R 56 =
2 m 56U
2 eB2
= 0, 2418 m. (En fait, on a R56
R
55
= 56
55 ). Les
détecteurs sont donc séparés de 2 · (R 56 − R 55 ) = 2 · (241, 8 − 239, 6) = 4, 4 mm.

Corrigé 7.4
Il faut placer E ~ comme indiqué. La
~
B ~
B force électrique F ~E = q E~ ( q < 0) vers
q<0 q<0
le haut compense la force de Lorentz
~
v ~ L = q~ ~ (orientée vers le bas). Donc
b

~
v
b
F v∧B
b
| q|E = | q|vB ⇒ E = vB (1). En absence
~
E
de champ E ~ on a (1ère figure) | q|vB =
v2 | q| v E
m R ⇒ m = RB = RB 2 par (1). Donc on a
| q| ′
m = E
RB2
= 0,1·1(3500
·10−4 )2
= 1, 67 · 1011 C/kg.
Pour un électron : me = 1, 76 · 1011 C/kg.
La particule est sûrement un électron.

180
Corrigé 7.5
Un positron (ou positon) est un anti-électron (particule de même masse que l’électron mais
de charge opposée). On a vu au cours que la trajectoire de cette particule (dans un champ
~ homogène) est une hélice (ou un cercle si ~
B v⊥B ~ ). Le mouvement du positron est la
composition d’un MRU dans la direction de B ~ et d’un MCU dans le plan perpendiculaire à
~
B. L’énergiercinétique du positron est E c = 2 keV = 3, 2 · 10−16 J. On en déduit sa vitesse :
q
2·3,2·10−16
v = 2m Ec
= 9,11·10−31 = 2, 65 · 107 m/s.

a) La composante de la vitesse perpendiculaire au champ B ~ vaut


v⊥ = v sin α = 2, 65 · 105 · sin 89 = 2, 65 · 107 m/s (comme v, au arrondis près). La période T
mv2
du MCU se calcule ainsi : ev⊥ B = R ⊥ ⇒ v⊥ = eBRm
. Donc T = 2Vπ⊥R = 2eB
πm
= 3, 58 · 10−10 s
= 358 ps. (La période est indépendante de v. Elle ne dépend que du type de particule ( e,
m) et du champ B).
b) La composante de la vitesse parallèle au champ B ~ (V∥ = v cos α avec α = 89◦) ne change
7 5
pas. Elle vaut V∥ = 2, 65 · 10 · cos 89 = 4, 62 · 10 m/s. le pas p de l’hélice est la distance
parallèle à B ~ que le positron parcourt en une période. Le pas est donc p = v∥ T = 4, 62 · 105 ·
−10
3, 58 · 10 = 1, 65 · 10−4 m = 0, 165 mm.
mv⊥
c) Finalement, le rayon R du cercle est (voir a) : R = eB
= 1, 51 · 10−3 m = 1, 51 mm.

Corrigé 7.6
a) Admettons que le sang entre dans la
feuille (la vitesse des ions entre dans la
~
B feuille). Les ions subissent la force de Lo-
b

−− − − − C rentz : les Na+ migrent vers le bas de l’artère


et les Cl− vers le haut. Il y a donc apparition
Na+ Cl−
N + − S d’un champ électrique vertical dont le sens
~
E va de bas en haut (voir la figure). Une force
++ + + +
b
D électrique (de sens opposé à la force de Lo-
d rentz) prend naissance sur les ions.

~E = q E
Lorsque la force électrique F ~ s’oppose exactement à la force de Lorentz F
~ L = q~ ~
v∧B
sur chaque ion, la migration cesse et un régime stationnaire est créé (les ions qui cir-
culent dans cette portion d’artère vont tout droit à vitesse constante v). On a FE = FL ⇒
| q|E = | q|vB ⇒ E = vB. Le champ électrique est alors déterminer par le champ B et la
vitesse du sang v. Pour simplifier, supposons que la vitesse des ions est uniforme dans
l’artère, le champ électrique est alors homogène. On mesure la tension (Hall) entre C et D
(UDC = 300 µV) au moyen d’un voltmètre très sensible. Cette tension est liée au champ E
par UDC = Ed .
Remarque : En réalité, dans un fluide visqueux (comme le sang), la distribution des vi-
tesses n’est pas uniforme sur une section de l’artère. Le sang va plus vite au centre et de
moins en moins vite proche des bords de l’artère. Le champ électrique n’est pas homogène
et le calcul du débit est plus complexe.

b) La vitesse du sang est alors déterminée directement par la mesure de U ≡ UDC :


·10−4
v= EB
= dU·B = 0,3012 ·0,1
= 0, 25 m/s.

181
Le débit D est directement lié à la vitesse du sang v et à la section S (en m2 ) de l’artère.
2 π·0,0122 ·0,25
En effet, on a vu : D = v · S = πd4 v = 4
= 2, 83 · 10−5 m3 /s = 28, 3 cm3 /s = 28, 3 ml/s.
Remarque : Ce dispositif, utile pour mesurer des débits sanguins, peut aussi fonctionner
en sens inverse pour controller la vitesse du sang dans une artère. On applique une ten-
sion U (la portion de l’artère est dans le champ B comme avant). Le régime stationnaire
est atteint (quasiment instantanément) dès que la vitesse des ions est telle que FE = FL .

30.8 Solutions Série 8

Corrigé 8.1 a) Si le barreau est immobile, la force


de Lorentz F~ L = I~
L∧B~ doit être dirigée
~ y vers la droite et donc le courant I doit
B ~
B
G entrer dans la feuille (figure de droite).
~
S x
~L ~
S b) Le système est à l’équilibre. Selon x :
F
I
~L S sin α = FL et selon y : S cos α = mg.
~ F FL
P D’où, tan α = m ⇒
~ g
α α P m g tan α 0,02·9,81·tan20
I= Bd = 0,05·0,1 = 14, 3 A.

Corrigé 8.2
Les deux conducteurs se repoussent si les courants sont de sens opposés. La force de La-
place est FL = I LB avec B le champ créé par un des courants à l’endroit où se trouve
l’autre. Par la loi d’Ampère (appliqué à un fil rectiligne) : B = µ0 2πI d avec d = 2 mm la dis-
tance entre les deux fils. Chacun des tronçons à une longueur de L = 10 cm. On a donc
µ0 I 2 L µ0 ·102 ·0,1
FL = 2π d = 2π·0,002 = 0, 001 N.
Le poids d’un tronçon de fil de cuivre est (masse volumique du cuivre : ρ = 8′ 920 kg/m3 ;
section du fil : S = 10−6 m2 et la longueur L = 10 cm = 0, 1 m)
P = ρ SL g = 8′ 920 · 10−6 · 10−1 · 9, 81 = 8, 75 · 10−3 N. Le poids P est 8,75 fois plus grand que
la force de Laplace FL .

Corrigé 8.3
Dans cette situation, sur chaque fil s’exercent deux
forces opposées. Le courant I 2 crée le champ B ~ 2 à l’en-
~2
B ~1
B droit du fil (1), ce qui provoque sur lui la force F2 =
I1 ~ ~ I2 I 1 · L · B2 . Le courant I 1 crée le champ B~ 1 à l’endroit
~2 F F ~1
F b
F
du fil (2), ce qui provoque sur lui la force F1 = I 2 · L · B1 ,
d
où L est la longueur des fils.
~
B ~
B ~
B
D’autre part, le champ B ~ exerce sur chaque fil la force d’intensité identique F = I · L · B (car
µ I µ I µ I I L µ I I L
I 1 = I 2 ≡ I = 1 A). Comme B1 = 20πd1 et B2 = 20πd2 , il vient F1 = 02π1 d2 et F2 = 02π1 d2 (donc
F1 = F2 ). A l’équilibre on a F = F1 et aussi F = F2 ce qui donne l’égalité
µ I I L µ I
I LB = 02π1 d2 ⇒ d = 2π0B = 1 m.

182
Corrigé 8.4

vue de dessus
z y
y
~ da
F
équilibre
~
B d x x
~ cd ~ cd
F
a F
α
b
b
d
d
I c a b
a b

b ~ bc ~ ab
F ~ ab
F
F

La première figure donne le sens du courant (règle du tire-bouchon pour les forces). On a
représenté la force F ~ da qui s’exerce sur le côté da et F
~ bc qui s’exerce sur le côté bc. Ces
deux forces ne produisent pas de rotation du cadre, mais elles ont tendance à l’étirer. La
deuxième figure montre une vue de dessus avec les forces qui s’exercent sur les montants
verticaux (F~ cd et F
~ ab ) et qui produisent la rotation du cadre. Cette rotation s’arrête à la
position d’équilibre (3ème figure).
Numériquement : F da = F bc = N I [ da]B sin α = 20·5·0, 04·0, 1·sin 135 = 0, 28 N (avec N = 20,
le nombre de spires).
Fab = F cd = N I [ab]B sin 90 = 20 · 5 · 0, 05 · 0, 1 = 0, 5 N

Corrigé 8.5
a) Le champ créé par le fil entre dans la
feuille du côté du fil où se trouve le cadre.
I 1 = 30 A Les forces de Laplace sont donc comme in-
0
c = 1 cm diquées (les flèches ne sont pas à l’échelle).
I 2 = 20 A µ0 I 1
b = 8 cm Le champ décroît en 1/ x (B( x) = 2π x ) donc la
force aussi décroît en 1/ x.
b) Le cadre tend à s’approcher du fil, la ré-
x a = 30 cm sultante F est dirigée vers le fil.

F = F1 − F2 avec F1 = I 2 aB( c) est la force qui s’exerce sur le côté du cadre proche du fil et
F2 = I 2 aB( b + c) est la force sur ³le côté le plus ´ éloigné. On a donc
µ0 I 1 I 2 a ¡ 1 1
¢ µ0 ·30·20·0,3 1 1
F = 2π c
− b+ c = 2π 0,01
− 0,09 = 3, 2 · 10−3 N.

Remarque : Pour calculer la force F ′ sur un des côtés du cadre perpendiculaire au fil, il
faut R
intégrer la Rforce sur chaque tronçon infinitésimal dx ¡ b+. cOn
¢ obtient :
b+ c b+ c µ0 I 1 I 2 Rb+ c dx µ0 I 1 I 2 µ0 ·30·20
′ ′
F = c dF = c I 2 B( x) dx = 2π c x
= 2π ln c = 2π · ln(9) = 2, 6 · 10−4 N.

Corrigé 8.6
a) En faisant tourner le rotor, les fils verticaux du cadre tournent à une certaine vitesse v
autour de l’axe du rotor. Leurs électrons libres se déplacent alors dans le champ B~ (radial)
de l’entrefer et subissent la force de Lorentz (− e)~v∧B~ qui les met en mouvement le long

183
des fils verticaux (chaque électron libre acquière une petite vitesse v∥ parallèle au fil),
engendrant ainsi le courant I (de sens contraire au mouvement des électrons). Or, cette
vitesse v∥ donne lieu à une force de Lorentz supplémentaire (− e)~ ~ qui, cumulée à celle
v∥ ∧ B
de tous les électrons libres en mouvement, constitue ce que l’on a appelé la force de Laplace
qui s’exprime simplement par FLap = I LB et dont le sens est opposé à v (L = 10 cm est ici
la longueur de chaque fil vertical et donc aussi la hauteur du rotor).

rotor
I noyau de fer
bobine rectangulaire
~ext
F
I
~
B ~Lap
F I ~
I FLap
~ext
F
~
v
I

b) If faut donc exercer une force F ~ext pour compenser les F ~Lap (Fext = N · FLap ), afin d’as-
surer la rotation du rotor à vitesse constante. Plus précisément, c’est un couple de forces
qu’il faut exercer (voir la figure). Le nombre N = 500 est le nombre de spires (donc le
nombre de fils verticaux de chaque côté du cadre). Finalement, la puissance mécanique
Pméc que l’on développe est intégralement transformée en puissance thermique (et donc
en lumière) dans l’ampoule. Cette puissance thermique s’exprime par Pth = R I 2. On a
Pméc = 2Fext v = 2Fext ω r , avec ω la vitesse angulaire du rotor et r = 4 cm le rayon du rotor.
On peut donc déterminer le lien entre ω et le courant I souhaité. De Pméc = Pth il vient :
RI 15·0,4
2Fext ω r = R I 2 ⇒ 2 N I LBω r = R I 2 ⇒ ω = 2 NLBr = 2·500·0,1·0,01·0,04 = 150 rad/s. Le nombre
de tours par seconde (la fréquence f ) qu’il faut faire effectuer au rotor pour obtenir un
courant de I = 0, 4 A est donc f = 2ωπ = 1502π
= 24 Hz (24 tours par seconde).

Corrigé 8.7
Le courant est entrant dans la barrette (1) et
sortant dans la barrette (2) et donc les deux
vue en coupe barrettes se repoussent. B ~ 1 est le champ
que la barrette (1) engendre à l’endroit où
O µ I
se trouve la barrette (2) : B1 = 4πL0sin α . La
fil
barrette (2) subit de la barrette (1) la force
α x µ I2a
L F2 = I aB1 = 4πL0 sin α (a est la longueur de la
I
x si ~ ( x) barrette). Le moment (par rapport à O) MO
n (2 dF
α) b

~ ( x) sortant
B de F2 est MO = L cos α · F2 (car L cos α est le
(1)
b

~2
F bras de levier de F2 ). On obtient pour MO :
2L sin α (2) µo I 2 a cos α
~1
MO = 4π sin α .
B
D’autre part, les fils souples se repoussent également. Pour évaluer le moment de la force
qu’exerce le fil (1) sur le fil (2), considérons une portion dx (située en x) du fil (2) et dési-
gnons par B( x) le champ en x (qui sort de la feuille) créé par le courant dans le fil (1). Le
µ0 I
champ B( x) vaut : B( x) = 2π x sin(2 α)
. La portion dx de fil subit alors la force infinitésimale

184

dF ( x) = IB( x) dx. Le moment infinitésimal (par rapport à O) de dF ( x) est dMO = x · dF ( x)
′ 0 µ I2
(car x est le bras de levier de dF ( x)). On obtient dMO = 2π sin(2 α)
dx (qui est indépendant
µ I 2
′ ′
R L ′ 0
RL
de x). Le moment MO total est l’intégrale : MO = 0 dMO = 2π sin(2 α) 0
dx ⇒
′ µ I2L
0 µ I2L
MO = 2π sin(2 α)
= 4π sin0 α cos α . Finalement, le moment total (des forces de Laplace) qui s’exerce
sur la barrette (2) et sur les deux fils qui la soutiennent est :
µ I2 ¡

= 40π sin α2Lcos α + asin cos α
¢
MO + 2 MO α
. A l’équilibre, ce moment total doit équilibrer le moment
du poids de la barrette qui vaut mgL sin α. On en déduit l’égalité suivante qui fournit la
relation entre le courant I et l’angle α :
µ0 I 2 ¡ 2L a cos α
¢
4π sin α cos α + sin α = mgL sin α.
Remarque : En fait, le point où l’on calcule les moments est choisi entre les points d’at-
tache des deux fils qui soutiennent la barrette (2). On obtient, pour le moment total des

forces qui s’exercent sur les fils qui soutiennent la barrette, le même résultat 2 MO obtenu
ci-dessus (car des termes se compensent). De même pour MO et pour le moment du poids
de la barrette.
L’expression entre I et α n’est en fait valable que si α est très petit car on a utilisé l’expres-
µ I
sion B = 2π0 r obtenue pour un fil rectiligne infini parcouru par un courant I . Pour un fil fini,
cette expression reste valable seulement pour le champ dans la région située proche du fil
(et loin des bords). Si α est petit, on peut faire les approximations : sin α ≃ α (en radians)
et cos
q α ≃ 1. On obtient
q finalement le résultat suivant ( I est proportionnel à α) :
4π m gL 4π·0,003·9,81·0,12
I≃ µ0 (2L+a)
·α = µ0 (2·0,12+0,1)
· α = 322, 3 · α.
π
Pour un angle α = 2◦ = 2 · 180 = 3, 49 · 10−2 rad, on a I = 322, 3 · 3, 49 · 10−2 = 11, 25 A.

Corrigé 8.8
Si le courant I 1 = 20 A de la spire suspendue au fléau circule dans le même sens que I 2 =
10 A de la spire fixe, les deux spires vont s’attirer et, pour compenser, il faut alors ajouter
une charge sur le plateau de la balance. Les spires étant proches, le champ qu’exerce
µ I
la spire (1) sur l’autre est B1 = 20πd1 avec d = 3 mm (distance entre les spires). La force
d’attraction est donc F = I 2 LB1 , avec L = 2πR (R = 5 cm est le rayon des spires). On a
µ I I L µ I I R µ I1 I2 R
donc F = 02π1 d2 = 0 1d 2 . La masse à ajouter est m = Fg = 0 gd = 0, 43 g. Si les courants
circulent dans des sens opposés, les spires se repoussent et il faut alors retrancher 0, 43 g
du plateau de la balance.

Corrigé 8.9 Le mouvement du barreau s’obtient en appliquant la


2ème loi de Newton. Selon x :
ILB cos α−m g sin α
FL cos α − mg sin α = ma ⇒ a = m
. L’accé-
y ~
B lération étant constante, il s’agit d’un MRUA, donc
x
~
S v = v0 + at et d = v0 t + 12 at2 . De ces relations, on tire
q
v = v02 + 2ad . Ici v0 = 0 et d = 20 cm.
~L
F b
I r ³ ´
ILB cos α−m g sin α
α Finalement : v = 2 d m =
r ³ ´
10·0,1·0,5 cos30−0,02·9,81 sin30
~
P 2 · 0, 2 0,02 = 2, 59 m/s.

185
Corrigé 8.10
a)
La tige est vue en coupe, le courant I entre dans la
y ~
B feuille. Selon y : S = mg ⇒ F f ≤ µ0 S = µ0 mg. Lorsque I
x ~ croît, FL croît et F f croît aussi de sorte que F f compense
S
FL . Or, cette compensation n’est possible que lorsque
~f
F ~L
F F f < µ0 S . Il y a donc mouvement de la tige dès que
I
FL = µ0 S = µ0 mg ⇒ IBd = µ0 mg. Il faut donc un cou-
µ mg 0,6·0,005·9,81
~
P rant au moins égal à : I = 0Bd = 0,06·0,1 = 4, 91 A.

b) ~ vers l’arrière, la force F


~ L pos-
En inclinant le champ B
sède maintenant une composante positive selon O y de
y ~
B sorte que la force de soutien S ~ qu’exerce le sol dimi-
α
x ~ nue et donc aussi la force de frottement statique maxi-
S
male µ0 S . Selon O y : S + FL sin α − mg = 0 ⇒ S = mg −
~f ~
F I α FL FL sin α. Le mouvement a lieu dès que FL cos α = µ0 S ⇒
FL cos α = µ0 ( mg − FL sin α) avec FL = IBd . De cette re-
~
P lation, on tire FL , puis I comme fonction de α.
µ0 m g 1
On obtient : I (α) = Bd cos α+µ0 sin α
. La fonction I (α) possède un minimum que l’on trouve
µ m g sin α−µ cos α
en la dérivant et en cherchant le zéro de la dérivée. On obtient I ′ (α) = 0Bd (cos α+µ 0sin α)2 .
0
Cette fonction s’annule lorsque sin α − µ0 cos α = 0 c’est-à-dire lorsque tan α = µ0 = 0, 6.
Ainsi α = tan−1 (0, 6) = 31◦ . En calculant I correspondant à cet angle, on trouve I = 4, 21 A
(qui est plus petit que le courant trouvé en a). C’est le plus petit courant nécessaire au
mouvement de la tige, pour autant que l’on incline le champ de 31◦ vers l’arrière.

30.9 Solutions Série 9

Corrigé 9.1

d = 20 cm
I1

r1
r ′1 ~
B
b

r2 I1 I2

r ′2 I2
b

~′
B

µ I 2µ0 I
a) B = 2 2π0d/2 = = 8 · 10−5 T.
πd
³ ´
µ I
b) B = B1 + B2 = 20π r11 + r12 = 10−4 T (avec r 1 = 0, 15 m et r 2 = 0, 05 m).
³ ´
µ I
c) B′ = B′2 − B′1 = 20π r1′ − r1′ = 2, 67 · 10−5 T (avec r ′1 = 0, 3 m et r ′2 = 0, 1 m).
2 1

186
Corrigé 9.2

~1
B

~
B
Les champs B ~ 2 ont même intensité : B1 = B2 = µ0 I
~ 1 et B
~2
B 60 ◦ 2π d
~ 1 et B
et l’angle entre B ~ 2 est de 60◦. On a donc
µ I
B = B1 cos 30 + B2 cos 30 = 2 2π0 d cos 30.
p p
d d Comme cos 30 = 3 3 µ0 I
2 , on a B = 2 πd .

60◦ 60◦
1 b b
2
I d I

Corrigé 9.3
Le champ à l’intérieur d’un solénoïde par-
couru par un courant I et comprenant n
~
B spires par mètre vaut : B = µ0 nI . Ici, n =
solénoïde
~T
α B 10 cm−1 = 103 m−1 et I = 20 mA. Comme le
champ terrestre B T = 2 · 10−5 T, on a
ouest est µ nI
tan α = BB = B0 = 1, 26 ⇒ α = 51, 5◦. L’ai-
T T
guille tourne de 51, 5◦.

Corrigé 9.4
~ est
On applique la loi d’Ampère où la circulation du champ B
prise sur un chemin fermé (lacet) qui est un cercle (dans le
I ~ (r)
B
b
b
plan de la coupe) de rayon r et centré en O.
b
Pour r < R 1 ou r > R 2 : B( r ) = 0 car le chemin fermé n’enlace
d~r b aucun courant.
r b
Pour R 1 < rH< R 2 : La circulation donne
~ · d~r = B( r ) dr = B( r )
H H
O b
Γ B Γ cercle dr = B( r ) 2πr . On a orienté
R2 R1 Γ dans le sens du champ B ~ et B ~ ( r ) ∥ d~r et B( r ) est constant
b

b
sur le cercle Γ. Par la loi d’ampère : B( r ) 2πr = µ0 N I car N
µ NI
courants I sont enlacés. Donc B( r ) = 20πr . Le champ est plus
intense proche de O que loin de O.

Remarque : Toujours par la loi d’Ampère (et par symétrie cylindrique autour de l’axe du
tore), B( r ) calculé est le même pour tous les points à l’intérieur du tore situés à la distance
r de l’axe du tore. De même, le champ est nul partout à l’extérieur du tore.

187
30.10 Solutions Série 10

Corrigé 10.1 Le vecteur surface S ~ est perpendiculaire à la


surface (plane ici) et sa norme est l’aire (en m2 )
de la surface (le disque ici). Le champ étant
~
S homogène, le flux se calcule par définition :
α ~·S~ = BS cos α. Rappelons que si B ~ était le
Φ=B
~ b champ de vitesse d’un courant d’eau, Φ repré-
B
senterait le débit (en m3 /s) de l’eau à travers la
surface.

Donc : a) Φ = BπR 2 cos α = 0, 5 · π · 0, 082 · cos 90 = 0 ; b) Φ = 0, 5 · π · 0, 082 · cos 60 = 5 · 10−3 Tm2 ;


c) Φ = 0, 5 · π · 0, 082 · cos 0 = 0, 5 · π · 0, 082 = 10−2 Tm2

Corrigé 10.2

a) b) c) d)
a l >> R 1
l >> R 1
I1
r R2 R1
I1 (2) 2R 2
b R1 I1
I1
(2)
N spires (2) N spires
(2)

a) Le champ entre dans le circuit (2) à la droite de I 1 et ressort à la gauche. Comme I 1


coïncide avec un axe de symétrie de (2), il y a parfaite compensation du flux. Le flux est
positif d’un côté et négatif de l’autre, la somme est donc nulle : Φ = 0.
~ ), le flux est donc
~⊥S
b) Le champ créé par I 1 ne traverse pas la surface du circuit (2) (B
nul : Φ = 0.
c) Le champ a la même direction que S ~ (mais pas nécessairement le même sens, cela
~
dépend du choix que l’on fait pour S ). Comme le champ est homogène dans un solénoïde et
πR 2 N I
que son intensité est B = µ0 Nl I , le flux à travers le circuit (2) vaut Φ = ±BπR 12 = ±µ0 1l
(il n’y a pas de lignes de champs à l’extérieur du solénoïde), avec + si S ~ et B~ ont même
sens et − si leurs sens sont opposés.
πR22 N I
d) Idem que pour c), sauf que maintenant : Φ = ±BπR 22 = ±µ0 l
.

Corrigé 10.3
On oriente les 5 faces de l’objet vers l’extérieur du volume que ces faces délimitent, comme
~ 1 . Le flux de B
pour S ~ sur les 2 faces situées dans le plan O yz (ou parallèle à lui) est nul
~ ~
(S ⊥ B pour ces deux faces). De même pour la face située dans le plan Ox y. Il ne reste que
deux faces où le flux n’est pas nul. Pour la face (disons S 2 ) située dans le plan Oxz, on a
Φ2 = −BS 2 = −Bac. Pour la surface S 1 , on a Φ1 = B ~·S~ 1 = BS 1 cos α.

188
p
z Or, cos α = p c et S 1 = a b2 + c2
vue depuis Ox b2 + c2
p
a
~1
S donc Φ1 = Ba b2 + c2 p 2c 2 = Bac. On
b +c
~1
S
α ~
B
constate donc que Φ1 = −Φ2 . Autre-
y α α ~ sur toute la sur-
ment dit, le flux de B
c ~
B
c face fermée (la surface du prisme) est
b b nul.
x

Il s’agit d’un cas particulier d’une situation générale pour le champ B~ : le flux d’un champ
~ quelconque est toujours nul à travers une surface fermée (loi de Gauss pour B). C’est une
B
grande loi de l’électromagnétisme qui est une traduction du fait qu’il n’existe pas de charge
magnétique (on n’en a encore jamais observée) contrairement au champ électrique où les
sources du champ électrique sont les charges électriques. Pour cette raison, les lignes du
champ magnétique sont refermées sur elles-même (ou vont à l’infini).

Corrigé 10.4
a) On peut résoudre ce problème en invoquant la loi de
Faraday. Avec le sens de parcours choisi (s.p.), le vecteur
l surface S ~ est orienté vers le haut et le flux est alors posi-
~
S
R tif : Φ( t) = Blvt > 0. On a supposé qu’à t = 0, le wagonnet
s.p. ~
B est à x = 0.
I ind ~L
F b) La surface croît et donc le flux aussi. Par la règle de
x Lenz, le courant induit I ind va dans le sens indiqué et
b

b
~
v la force de Laplace FL s’oppose au mouvement.

Uind = −Φ̇ = −BLv (Uind est opposé au sens de parcours, donc le champ électrique dans la
résistance aussi et donc aussi le courant induit (en accord avec la règle de Lenz), I ind = Blv
R .
B2 l 2 v
c) FL = I ind lB = R
.
2 2 2
l v ∆t
d) F ′ s’oppose à FL ; le travail de F ′ est ∆W = F ′ ∆ x avec ∆ x = v∆ t. Donc ∆W = B R qui
est positif (on doit fournir du travail pour maintenir la vitesse v au wagonnet).
B2 l 2 v2
e) La puissance fournie est P = ∆∆Wt = R
.
2 B2 l 2 v2
f) La puissance dissipée dans la résistance est Pth = R I ind = R .
g) Toute la puissance fournie P est dissipée en chaleur (par unité de temps). Il y a conser-
vation de l’énergie (et donc de la puissance P = Pth ).
h) v change de sens, Φ aussi (de signe), I ind aussi, FL et F ′ aussi. Il faut toujours fournir
le travail ∆W et donc P et Pth sont les mêmes qu’avant.
i) La vitesse croît v = at et x = 21 at2 (partant au repos à x = 0 disons). Donc Φ( t) = 12 Blat2
et Uind = −Blat etc.. Il suffit de remplacer v par at dans les expressions où v apparaît.

189
Corrigé 10.5
a) Le champ créé par le courant entre dans le plan
µ I
bande d’épaisseur dx du circuit. Par Ampère, il vaut B( y) = 2π0 y , où y est la
~
v distance du fil à l’endroit où on mesure le champ. On
I ind
oriente la surface (du circuit) dans le même sens que
~
B ~ , le flux Φ est alors positif. Pour le calculer, il faut in-
B
b x tégrer sur toutes les bandes d’épaisseur infinitésimale
r
a dx. Sur une bande située à la cote x, le flux infinitési-
mal d Φ( x) vaut :
I d Φ( x) = B( x)adx pour x tel que r − b2 ≤ x ≤ r + b2 .
µ ¶
Rr + 2b b
µ0 Ia Rr + 2 1 µ0 Ia r + 2b
Le flux total est Φ = B( x)adx = 2π r − x dx = 2π ln .
r − 2b b
2
r − 2b

b) Le mouvement du cadre engendre une diminution du flux et donc un courant induit


prend naissance dans le circuit. Par la règle de Lenz, ce courant tend à s’opposer à cette
diminution (le champ magnétique induit est dans le même sens que B) et circule donc dans
|U |
le sens indiqué. Sa valeur est I ind = Rind . Comme r = r 0 + vt, ( r 0 est la position initiale du
cadre) le fluxµdépend ¶du temps :
µ0 Ia r 0 +vt+ b2
Φ( t ) = ln . La tension induite est la dérivée par rapport au temps (petit exer-
2π r 0 +vt− b2
cice de dérivation) :
b
µ0 Ia r 0 +vt− 2 −bv µ0 Iabv 1 µ0 Iabv 1
Φ̇ = 2π r 0 +vt+ b (r 0 +vt− b )2 =− 2π (r 0 +vt)2 −b2 /4 . Finalement, I ind = R 2π (r 0 +vt)2 −b2 /4 .
2 2

Corrigé 10.6

a)
Les lignes de champ électrique induit sont des cercles au-
B croît tour du cylindre (symétrie cylindrique). Prenons pour chemin
R fermé Γ un de ces cercles (de rayon r ) et choisissons un sens
de parcours (s.p.). L’orientation de S~ (surface sous-tendue par
~
B ~ et le flux est ainsi positif (avec ce
Γ) est alors dans le sens de B
choix d’orientation). Comme B = kt, on peut calculer le flux.
s.p.
~ ind
Pour r > R : Φ( t) = kπR 2 t ⇒ Uind = − kπR 2 < 0 et donc E
I ind
est orienté dans le sens indiqué. On en déduit par |Uind | =
2
E ind ( r )2πr que E ind ( r ) = kR
2r .
kr
Pour r < R : Φ( t) = kπr 2 t ⇒ Uind = − kπr 2 ⇒ 2πrE ind ( r ) = kπr 2 ⇒ E ind ( r ) = 2 .

b) Si maintenant B( t) = B0 − kt diminue partant d’une valeur initiale B0 , le champ élec-


trique induit est dans l’autre sens mais de même norme qu’en a).

190
Corrigé 10.7
a) La barre tombe, la surface délimitant le circuit diminue, le
flux diminue (en valeur absolue) donc, par Lenz, le courant
~L
F 0 induit va dans le sens indiqué.
b) Au début de la chute, la vitesse de la barre v est petite, le
C D courant induit aussi et donc la force de Laplace FL également.
~ Dans les tous premiers instants, on peut dire que la barre
~
v P
b
tombe en chute libre : v = gt (pour t petit). Pour ∆ t petit,
I ind la barre tombe de ∆ x = v∆ t et la surface du circuit diminue
~
B
de vl ∆ t. Donc le flux (en valeur absolue) diminue de ∆Φ =
R Bvl ∆ t. On en déduit |Φ̇| = Blv avec v = gt. Ainsi, le courant
z
Bl g
induit vaut I ind = |Φ̇
R
|
= R · t (au tout début).
l
c) Plus la vitesse augmente, plus le courant induit croît et donc il en va de même de
FL . La barre atteint asymptotiquement une vitesse limite vmax lorsque FL = mg. On a
m gR
I ind lB = mg et I ind = |Φ̇|
R =
Blvmax
R . Donc vmax = B2 l 2 .

d) Pour déterminer la solution exacte (valable à chaque instant t), il faut résoudre l’équa-
tion de Newton (projetée selon Oz) : mg − FL = ma, avec l’accélération a = v̇ (dérivée de la
2 2
vitesse). La force de Laplace dépend de la vitesse selon : FL = I ind lB = |Φ̇R|lB = B Rl v . L’équa-
tion de Newton devient :
2 2
mv̇ + BRl v = mg. Il s’agit d’une équation différentielle pour v dont on connaît la solution
générale (voir le cours de math sur ce thème). La solution est (sachant qu’à t = 0 la vitesse
de la barre³est nulle) : ´
2 2
v( t) = vmax 1 − e−B l t/Rm , vérifier cela en remplaçant cette expression dans l’équation de
Newton. Le courant induit ³ vaut donc ´ :
( t) −B2 l 2 t/Rm mg
I ind ( t) = Blv
R = I max 1 − e avec I max = Bl . C’est une fonction croissante avec
asymptote horizontale à I max .

Corrigé 10.8

L’aire de la boucle est fonction du temps : S ( t) = πr 2 ( t) et elle di-


~
B minue au taux de Ṡ = − k = −0, 4 m2 /s. Le flux Φ( t) = BS ( t) ⇒ Φ̇ =
B Ṡ = − kB. La tension induite Uind = −Φ̇ = kB = 0, 4 · 3 = 1, 2 V. Elle
r ( t) est positive (et constante durant la décroissance de la surface). Le
champ électrique induit est donc dans le même sens que le sens de
s.p. parcours choisi.

Corrigé 10.9
Uind = at avec a = 2 · 10−5 V/s pour 0 < t < 5 s. Par Faraday, Uind = −Φ̇ où Φ( t) = Nb B( t)S s
µ N I ( t) µ N N S µ N N S
(S s = πd 2s /4) et où B( t) = 0 ls . Donc Uind = 0 bl s s İ ⇒ at = 0 bl s s İ . Par intégration,
on en déduit le courant I ( t) :
2
I ( t) = µ0 N lNs S s · at2 . Le courant varie comme t2 .
b

191
30.11 Solutions Série 11

Corrigé 11.1
Dans un circuit RL¢ (série) branché sur un générateur de tension U , le courant satisfait
I ( t) = I max 1 − e−t/T avec I max = U
¡
R , à compter du moment où l’on ferme l’interrupteur. On
cherche ¡ t pour ¢ que I ( t) = 0, 999 · ¡I max. −Donc
− t/T
= 0, 999 · I max ⇒ 1 − e t/T = 0, 999 ⇒ e−t/T = 0, 001 ⇒ t = − ln(0, 001) · T =
¢
I max 1 − e
6, 91 · T . Il faut presque attendre 7 constante de temps.

Corrigé 11.2
I (5) = I max −5/T I max
⇒ e−5/T = 32 ⇒ − T5 = ln(2/3) ⇒ T = − ln(2/3)
5
¡ ¢
3 ⇒ I max 1 − e = 3 = 12, 3 s.

Corrigé 11.3
I (0) I max 1
Le courant décroît selon I ( t) = I max e−t/T . Or, I (1) = I (1) = 10−2 = 100, avec I (1) = I max e
−1/T
.
I max 1s L L 10
Donc 100 = e−1/T ⇒ T = ln(100) = 0, 217 s. Mais T = R ⇒ R = T = 0,217 = 46 Ω.

Corrigé 11.4
45 0,05
Le courant satisfait I ( t) = I max 1 − e−t/T avec I max = U L
¡ ¢
R = 180 = 0, 25 A et T = R = 180 =
2, 77 · 10−4 s. Il faut calculer I˙( t) pour t = 1, 2 ms. On dérive et l’on obtient :
0,25 −3 −4
İ ( t) = I max
T e
− t/T
⇒ İ (1, 2 · 10−3 ) = 2,77·10−4 e−1,2·10 /2,77·10 = 11, 9 A/s.

Corrigé 11.5
U 50
a) Comme avant, I ( t) = I max 1 − e−t/T avec I (5 · 10−3) = 2 · 10−3 A et I max =
¡ ¢
R
= 104
= 5·
( t) −3 −3 −3
10−3 A. D’où e−t/T = 1 − IImax ⇒ e−5·10 /T
= 1 − 25··10
10 3 5·10
−3 = 5 = 0, 6 ⇒ T = − ln(0,6) = 9, 79 · 10
−3
s.
Donc L = RT = 97, 9 H.
b) L’énergie contenue dans la self est E = 12 LI 2 = 0, 196 mJ.

Corrigé 11.6
On utilise la loi de Faraday dans une situation où il y a de l’autoinduction : |Uind | = L| İ |
avec L = 4, 6 H (sachant que İ = ∆ I
∆ t ).
a) | I˙| = | ∆ I 7 ′ ′
∆ t | = 2·10−3 = 3 500 A/s ⇒ |Uind | = 4, 6 · 3 500 = 16, 1 kV.

b) | I˙| = | ∆ I 7−5
∆ t | = 3·10−3 = 666, 7 A/s ⇒ |Uind | = 4, 6 · 666, 7 = 3, 07 kV.

c) | İ | = | ∆ I 5 ′ ′
∆ t | = 1·10−3 = 5 000 A/s ⇒ |Uind | = 4, 6 · 5 000 = 23 kV.

192
30.12 Solutions Série 12

Corrigé 12.1
On a vu au cours, pour un circuit RL (série) en régime alternatif, la tension aux bornes du
générateur et le courant sont de la forme :pU ( t) = Û sin(ω t + φ1 ) et I ( t) = Î sin(ω t + φ2 )
ωL
avec tan(φ1 − φ2 ) = R et Û = Z Î pour Z = R 2 + (ωL)2 .

a) On cherche ω pour que pÎ = 0, 05 A. D’où Z = Û = 28, 3 Ω (car Û = 2 V) et donc on tire ω


2 Î
qui vaut ω = 732, 3 rad/s ⇒ f = ω/2π = 116, 5 Hz.
b) tan(φ1 − φ2 ) = ωRL = 2, 136 ⇒ φ1 − φ2 = 65◦ . Le courant est ≪ en retard ≫ de 65◦ par rapport
à la tension.
c) Si f = 1000 Hz, ω = 2π f = 6, 28 · 103 rad/s. On calcule la nouvelle impédance Z qui vaut
2
Z = 220, 2 Ω et donc Î = Û
Z = 220,2 = 9, 1 mA.

Corrigé 12.2
1 1 1
a) tan(−30◦ ) = − ωRC ⇒ R = ωC tan 30 = 2π·1500·2·10−6·tan30 = 91, 9 Ω.

b) Utilisons la notation complexe pour les impédances : 1z = R1 + i ωC et l’argument de z


(l’angle par rapport à l’axe réel) est de −30◦ , d’où l’argument de 1/ z est de φ = 30◦ . Donc
tan φ
tan φ = Im(1/ z)
Re(1/ z) = R ωC ⇒ R = ωC = 30, 6 Ω.
q
c) Pour a) : | z| = R 2 + (ω1C )2 = 106 Ω ;
q
1 1
pour b) : | z| = | z | = R12 + (ωC )2 = 0, 0377 Ω−1 ⇒ | z| = 26, 5 Ω.

Corrigé 12.3
U
a) I eff = | zeff| = ω5L = 2π·5005·5·10−2 = 32 mA.

b) L’impédance devient | z| = |ωL − ω1C | et le courant devient I eff = 96 mA.


5 1
Donc 0,096 = |157, 1 − 3′ 142 C
| ⇒ C = 3, 03 · 10−6 F = 3, 03 µF.

Corrigé 12.4

L’impédance équivalente (complexe) de ce groupement est :


L R 1 1 1−ω2 LC + iωRC 2
z = R + i ωL + i ω C = R + i ωL = 1−ω RLC + iωRC R − iωL
+ i ωL R − i ωL =
R + iω(ω2 L2 C +R 2 C −L)
R 2 +ω2 L2
.
C
R 2 +ω2 L2
On obtient | z| en calculant l’inverse de | 1z |. Il vient | z| = p = 180 Ω.
R 2 +ω2 (ω2 L2 C +R 2 C −L)2

Corrigé 12.5
a) L’impédance est z = R + i ωL où la fréquence du réseau est f = 50 Hz et p ω = 2π f . La
tension efficace du réseau est Ueff = 220 V. Le module de l’impédance est | z| = R 2 + (ωL)2
U
(RL série). Le courant efficace est I eff = | zeff| .

193
Im
ωL 2
Ueff
La puissance moyenne consommée est P̄ = Ueff I eff cos φ = | z|
cos φ
φ avec cos φ = |Rz| .
Re
R
b) En plaçant une capacité C en parallèle, la nouvelle impédance ³ complexe
´ z′ se calcule :
1 1 1 R − i ωL R ωL
z′ = z + i ωC = R + iωL + i ωC = R 2 +(ωL)2 + i ωC = R 2 +(ωL)2 + i ωC − R 2 +(ωL)2 .
Pour que l’impédance z′ soit réelle, sa partie imaginaire doit donc être nulle, c’est-à-dire
aussi celle de z1′ . Il faut donc que la capacité C soit de C = R2 +L(ωL)2 pour rendre nulle
la partie imaginaire ci-dessus. Le nouveau déphasage est alors nul (φ′ = 0). La nouvelle
2
impédance est alors z′ = | zR| . Le nouveau courant I eff ′
devient :
′ Ueff Ueff | z| R
I eff = | z′ | = | z| | z′ | = I eff p 2 2
qui est manifestement plus petit que I eff puisque
R +(ωL)
p R < 1. La facture, elle, ne change pas car la puissance moyenne consommée est
R 2 +(ωL)2
identique à celle d’avant. En effet,
2 cos φ 2 R /| z| 2 R 2 1
P̄ = Ueff | z|
= Ueff | z|
= Ueff | z|2
= Ueff z′
= P̄ ′ (puisque cos φ′ = 1).

Corrigé 12.6
25
a) Î = |Ûz| où | z| = ωL = 377 · 12, 7 = 4788 Ω. Donc Î = 4788 = 5, 2 mA.

b) Comme φ = π2 ⇒ U = 0 lorsque I = Î .
c)
On cherche t 1 pour que U ( t 1 ) = −12, 5 V
U [V] et U ( t 1) sur le flanc croissant. On résout
U ( t) = Û sin(ω t)
25 25 sin(ω t 1 ) = −12, 5. Donc
sin(ω t 1 ) = −0, 5 ⇒ ω t 1 = − π6 + 2π = 116π . D’où
t1
t [s] l’on tire t 1 = 611 π
·377 = 15, 3 ms. Le courant I ( t)
−12, 5 étant déphasé de π2 par rapport à la tension
−25 U ( t) (car le circuit est une self pure),
¡ 11π
− π2 = 0, 0052 · sin 116π − π2 = 0, 0052 · sin 43π = −4, 52 mA.
¢ ¡ ¢ ¡ ¢
on obtient I ( t 1 ) = Î sin 6

Corrigé 12.7
a) sin(ω t − π/4) est maximale lorsque ω t − π/4 = π/2 ⇒ t = 43ωπ = 6, 73 ms.
b) I est maximal si sin(ω t − 3π/4) = 1 (ou −1) ⇒ ω t − 3π/4 = π/2 (ou −π/2) ⇒ ω t = 5π/4 (ou
π/4). Donc pour t = 4πω = 2, 2 ms, le courant vaut − Î et pour t = 45ω
π
= 11, 2 ms, le courant
vaut Î .
c) Comme I ( t) = Î sin(ω t − 3π/4) = Î sin(ω t − π/4 − π/2), le déphasage φ entre la tension et
le courant vaut φ = π/2 > 0. Le courant est en retard sur la tension, il s’agit d’un élément
sans résistance (car déphasage de π/2), c’est donc une self pure.
| z|
d) | z| = Û = 030
,62 = 48, 4 Ω. Comme | z | = ω L ⇒ L = ω = 0, 138 H = 138 mH.

194
Corrigé 12.8
1 1 1
a) L’impédance complexe du circuit est z = zR + zC + zL avec,
1
C R zR = R , zC = et zL = i ωL. Sachant que ω = 2π f avec
i ωC
f = 1000 Hz, on obtient 1z = 8, 97 · 10−2 + i · 2, 68 · 10−2 ⇒
z = 10, 23 − i · 3, 06. Le module de l’impédance est donc | z| =
L p
10, 232 + 3, 062 = 10, 7 Ω.

a.c.
3,06
b) tan φ = − 10,23 ⇒ φ = −16, 7◦. Le courant est en avance sur la tension.
Ueff 10 2
c) La puissance moyenne consommée est P̄ = Ueff | z| cos φ = 10 ,7 cos(−16, 7) = 9 W.

Corrigé 12.9
a) La charge Q portée par le condensateur au départ est Q = CU = 1, 5 · 10−6 · 57 = 8, 55 ·
10−5 C. Comme le circuit est sans résistance, l’énergie électrique-magnétique est conser-
vée. Lorsque l’énergie du condensateur E C est maximale, l’énergie dans la bobine E L est
nulle et inversement. Donc,
Q2 Q
E max
C
= E max
L
⇒ 2C
= 12 LI max
2
⇒ I max = p = 0, 64 A.
LC
ω 1
b) La fréquence d’oscillation du circuit est f = 2π = p = 1′186 Hz. La période T est
2π LC
T = 1f = 8, 43 · 10−4 s. Or, après un temps de T /4 l’énergie devient maximale dans la bobine
(et nulle dans le condensateur), donc c’est à ce moment-là que le courant est le plus grand
( T4 = 2, 1 · 10−4 s).

Corrigé 12.10

Il faut d’abord fermer l’interrupteur S 2 pour que


S1 S2 l’énergie croisse dans la bobine (qui va ensuite
jouer le rôle de générateur pour charger C 2 ). Le
circuit oscillant ainsi formé à une fréquence f 1 =
C2 L C1 p1 = 1, 678 Hz, d’où une période T1 = f1 =
2π LC1 1

0, 596 s. L’énergie est maximale dans la bobine


après un temps de T1 /4 = 0, 15 s.

C’est à ce moment là qu’il faut fermer S 1 et rouvrir S 2 . Le second circuit oscillant a une
fréquence f 2 = p1 = 0, 199 Hz, d’où une période T2 = f1 = 5 s. Il faut finalement ouvrir
2
2π LC2
T2
S 1 après 4 = 1, 25 s pour que toute l’énergie de la bobine se trouve dans le condensateur
C2 .
L’énergie qu’il y avait au départ : 21 C 1U 2 (U = 100 V), se retrouve finalement sur le conden-
sateur C 2 (sous une tensionqU2 ). Doncq
1 2 1 2 C1 900
2 C 2U 2 = 2 C 1U ⇒ U2 = U C2 = 100 · 100 = 100 · 3 = 300 V.

195
Corrigé 12.11
a) Pour que l’ampoule brille normalement, la tension (efficace) à ses bornes (notée UR )
doit être égale à UR = U0 = 3, 5 V. Le courant qui la traverse est donc de I 0 = 0, 2 A. Notons
k = N2 / N1 = 3 le rapport de transformation du premier transformateur et donc 1/ k est le
rapport du second transformateur. La tension aux bornes du primaire du second trans-
formateur est alors de U1 = kU0 et le courant dans la ligne (dont la résistance est r ) est
I 1 = I 0 / k. Par Kirchhoff, la tension U2 aux bornes du secondaire pour le premier transfor-
mateur est : U2 = U1 + rI 1 . Ainsi, la tension UG aux bornes du générateur est UG = U2 / k. On
obtient donc UG = U0 + rI 0 / k2 = 4, 193 V. Comme I G = kI 1 = I 0 , la puissance au générateur
est PG = UG I G = U0 I 0 + rI 02 / k2 = 0, 839 W.

b) La puissance perdue dans la ligne est Pth = rI 12 = rI 02 / k2 = 0, 139 W.

c) La puissance PG calculée s’exprime maintenant par PG = ( r + R ) I ′ 2 où I ′ est le courant


qui passe dans la ligne et dans l’ampoule (de résistance R ). On en déduit I ′ = 0, 131 A. La
nouvelle tension au générateur sera donc de UG′ = PG / I ′ = 6, 39 V.

d) La tension aux bornes de l’ampoule est de UR′ = R I ′ = 2, 297 V.

e) Le taux de luminosité est de : UR′ I ′ /(U0 I 0 ) = 0, 43, soit 43% de la puissance nominale de
l’ampoule.


f) Maintenant, la puissance perdue dans la ligne est Pth = rI ′ 2 = 0, 537 W.

196
31 Quelques constantes

Vitesse de la lumière dans le vide c = 2, 998 · 108 m/s


Constante de la gravitation universelle G = 6, 673 · 10−11 N m2 /kg2
Charge élémentaire e = 1, 602 · 10−19 C
Constante de Planck h = 6, 626 · 10−34 Js
Masse au repos de l’électron me = 9, 109 · 10−31 kg
Masse au repos du proton mp = 1, 673 · 10−27 kg
Masse au repos du neutron mn = 1, 675 · 10−27 kg
Masse au repos d’une particule α (2 p + 2 n) mα = 6, 644 · 10−27 kg
Permittivité électrique ǫ0 = 8, 854 · 10−12 A s/V m
Perméabilité magnétique µ0 = 4π · 10−7 V s/A m

Unité de masse atomique 1u = 1, 661 · 10−27 kg


Unité d’énergie : l’électron-volt 1 eV = 1, 602 · 10−19 J

197

Vous aimerez peut-être aussi